├── 1.png ├── 1.tex ├── 2.tex ├── 3.tex ├── 4.tex ├── 5.tex ├── 6.tex ├── 7.tex ├── README.md ├── main - 1A.pdf ├── main - 1A.tex ├── main - 1B.pdf ├── main - 1B.tex ├── main.pdf ├── main.tex └── preface.tex /1.png: -------------------------------------------------------------------------------- https://raw.githubusercontent.com/jamesfang8499/math1/df2bca2d15cbb1afb9567be4f2d5c70b6e0ca1cb/1.png -------------------------------------------------------------------------------- /2.tex: -------------------------------------------------------------------------------- 1 | \chapter{一次方程式} 2 | 3 | 4 | 在第一章,复习小学算术的基础之上,又引进了 5 | 负数,建立了对于加、减、乘、除以及乘方运算通行 6 | 无阻的有理数系,还着重讨论了对任何有理数都适用 7 | 的“运算通性”.这样一来,当我们用字母表示数 8 | 时,就应该注意领会:这个字母既可以表示所指范围 9 | 内的任一个数,同时也具有数的运算通性这两层意 10 | 义;例如:矩形的面积公式$S=a\cdot b$中,字母$a, b, S$ 11 | 分别表示矩形的长度、宽度与面积.它们可以表示所 12 | 有正数中的任一个;在它们的运算中,同样具有“运 13 | 算通性”. 14 | 15 | 本章将在字母表示数的基础上,由应用问题入 16 | 手,引进一次方程式,并应用代数方法去讨论解应用 17 | 题的一般途经. 18 | 19 | \section{算术解法与代数解法} 20 | 这一节我们将以应用题为目标,分别用算术解法 21 | 与代数解法来解决,从中领会代数解法的要点,并通 22 | 过比较分析,初步认识代数解法的一普遍性与优越性. 23 | 进而去掌握代数解法的原理,熟练运用其方法解决应 24 | 用问题. 25 | 26 | \subsection{两种解法的分析、对比} 27 | 从两个实例谈起: 28 | 29 | \begin{example} 30 | 某农场计划播种小麦与大豆共138亩,要求 31 | 种小麦的面积是大豆的四倍,试问:该农场应种小麦 32 | 与大豆各多少亩? 33 | \end{example} 34 | 35 | 【算术解法分析】由题目所给条件,可知:播种 36 | 总面积就是种大豆面积的$(4 + 1)$倍.因此, 37 | \[\begin{split} 38 | \text{(种大豆亩数)}&=\text{(总亩数)}\div (4 + 1)\\ 39 | \text{(种小麦亩数)}&=\text{(总亩数)}-\text{(种大豆亩数)} 40 | \end{split}\] 41 | 即: 42 | \begin{align*} 43 | 138\div(4 + 1)&=27.6\text{(亩)}\tag{大豆亩数}\\ 44 | 138-27. 6 &= 110.4\text{(亩)}\tag{小麦亩数} 45 | \end{align*} 46 | 47 | 当然,求出种大豆的亩数以后,也可根据题目已 48 | 知,4倍这个数,就是种小麦亩数. 49 | 50 | 【代数解法分析】可以用一个字母$x$表示我们所 51 | 求的一个数量,例如在此题中,$x$可以表示“种大豆 52 | 的亩数”;再由题目所给条件,显然可知“种小麦的 53 | 亩数”就应该用$4x$表示;因此,只要根据“(总亩数) 54 | $=$(种小麦亩数)$+$(种大豆亩数)”的关系式,和 55 | 题目给出“总亩数$=13$”的条件,就可以直截了当 56 | 地得到一个等式: 57 | \[4x+x=138\] 58 | 59 | 对于这个等式,比起算术中的算式来说,只是多 60 | 了一个“表示要求的数的字母$x$”,尽管还不知道$x$ 61 | 究竟是多少,但终归是一个数,就必然要具有“数的 62 | 运算通性”,特别是在运算中同样可以有效地对它使 63 | 用加、乘的交换、结合和分配律;又因为所列是一个 64 | 等式,就必然具有等式的基本性质.因此,我们可以 65 | 作如下的变形: 66 | \begin{align*} 67 | (4+1)x=138 \tag{分配律} 68 | \end{align*} 69 | 即:$5x=138$ 70 | \begin{align*} 71 | \therefore\quad x&=138\div 5 \tag{等式两边同除以5}\\ 72 | x&=27.6\text{(亩)} \tag{种大豆亩数}\\ 73 | 4x&=4\x 27.6=110.4\text{(亩)} \tag{种小麦亩数} 74 | \end{align*} 75 | 76 | 【对比】算术解法中,要求对题意进行思考,说 77 | 明每一个算式的意义.如$(4+1)$表示“总亩数是种 78 | 大豆亩数”的倍数等;而代数解法只要求用字母$x$表 79 | 示所求的一个数量(例2.1中,可以自己练习设$x$表示 80 | “种小麦亩数”的解法),将$x$与已知的数量一起考 81 | 虑它们之间的关系,根据题意能够直接了当地把关系 82 | 列成一个等式然后应用“运算通性”及“等式性 83 | 质”求出$x$应有的值. 84 | 85 | \begin{example} 86 | 小明用六角四分人民币买面值为4分、8分 87 | 两种邮票一共12张.试问:小明买了4分、8分邮票各几张? 88 | \end{example} 89 | 90 | 【算术解法分析】首先假定小明所买12张都是 91 | 9分邮票,就应该花钱$12 \x 0. 04=0.48$(元).而这 92 | 与实际所花的钱$0. 64$(元)还相差$(0. 64 - 0.48) 93 | =0.16$(元). 94 | 95 | 其次,为了使所花钱数增加,且邮票张数不变, 96 | 就要用8分邮票去换取4分邮票,每换取一张,钱数 97 | 增加$(0. 08-0. 04)=0. 04$(元),而邮票张数还保持 98 | 不变.这样,我们的问题就可以转化为:换几次(用 99 | 几张8分邮票)后,才能使所花钱数正好能补上所差 100 | 的0.16(元)呢? 101 | 102 | 由此可以得出本题算术解法是: 103 | \begin{equation} 104 | \begin{split} 105 | \text{“8分邮票的张数”}&=\text{“换取的次数”} \\ 106 | &=[0.64-(12\x 0.04)]\div (0.08-0.04) \\ 107 | &=0.16\div 0.04=4\text{(张)} 108 | \end{split} 109 | \end{equation} 110 | \[ \text{“4分邮票的张数”}=12-4=8 \text{(张)} \] 111 | 112 | 【代数解法分析】首先仍可以用一个字母$y$表 113 | 示所求的“8分邮票的张数”;由题目显然可以 114 | 知道“4分邮票的张数”就应该用$(12-y)$表示 115 | 了. 116 | 117 | 其次,由于$\text{(每张邮票的价钱)}\x\text{(张数)}=\text{(这 118 | 种邮票所用的钱数)}$,因而可以知道小明买8分、4分 119 | 邮票分别使用的钱数为:$(0,08\cdot y)$元与$0.04(12-y)$元. 120 | 121 | 再次,由于两种邮票总共花钱六角四分,因此就 122 | 得到: 123 | \begin{equation} 124 | (0.08)y+0.04(12-y)=0.64 125 | \end{equation} 126 | 127 | 最后,由(2.2)出发,运用数系运算通性和等式 128 | 的性质(特别是分配律),就可求出$y$应取的值来, 129 | 即: 130 | \begin{align*} 131 | 0.08y+0.04 \x 12-0.04y&= 0.64 \tag{分配律}\\ 132 | (0.08y-0.04y)+0.48&=0.64 \tag{交换、结合律}\\ 133 | (0.08-0.04)y+0.48&=0.64\tag{分配律}\\ 134 | 0.04y&= 0.64-0.48\tag{两边同减0.48}\\ 135 | 0.04y&=0.16 136 | \end{align*} 137 | \begin{align*} 138 | \therefore\quad y&=\frac{0.16}{0.04} \tag{两边同除以0.04}\\ 139 | y&=4\text{(张)} \tag{8分邮票张数}\\ 140 | 12-y&=12-4=8\text{(张)} \tag{4分邮票张数}\\ 141 | \end{align*} 142 | 143 | 【对比】算术解法中,我们要得出关键的算式 144 | (2.1)是不容易的,必须要经过一番思考,化费一定 145 | 心血才能得出的.而且每一步都要给出必要的说明, 146 | 显得“拐弯抹角”、“道路曲折”;而代数解法从思 147 | 路到计算都是比较直截了当,平铺直叙的.再加上我 148 | 们只要由例2.1、例2.2两题的代数解法中,不难发现, 149 | 它的基本路子、格式是相类似的.也就是说,代数解 150 | 法是有普遍性的.特别是在解一些较复杂的应用题 151 | 中,这种普遍可行的解法,更能显示出它的优越性 152 | 来. 153 | 154 | \begin{example} 155 | 有两所图书馆,自建馆以来每年各进图书五 156 | 千册;如果今年甲馆藏书23万册,乙馆藏书11万册, 157 | 今后仍然是每年各进书5千册.试问:由今年起,几 158 | 年后甲馆所藏图书的册数是乙馆的三倍? 159 | \end{example} 160 | 161 | 我们采用直接了当的代数解法,从中进一步明确 162 | 这种解法的普遍性、一贯性,并领略它的优越性. 163 | 164 | \begin{solution} 165 | 首先,设由今年起$x$年后甲馆藏书册数为乙 166 | 馆的三倍. 167 | 168 | 其次,由于今后每年两图书馆都是仍然进书5千 169 | 册,因此,$x$年后: 170 | \begin{itemize} 171 | \item 甲馆藏书册数为:$(23 + 0. 5x)$万册; 172 | \item 乙馆藏书册数为:$(11+0.5x)$万册. 173 | \end{itemize} 174 | 175 | 再次,由于$x$年后,甲所藏图书的册数是乙的三 176 | 倍,所以就有等式: 177 | \[(23 + 0. 5x)=3(11+0.5x) \] 178 | 179 | 以下只须用“通性”及等式性质进行变形: 180 | \begin{align*} 181 | 23 + 0. 5x& =33+1.5x \tag{分配律}\\ 182 | 23&=33+1.5x-0.5x \tag{两边同减$0.5x$}\\ 183 | 23-33&=1.5x-0. 5x \tag{两边同减33}\\ 184 | 23-33&=(1.5-0.5) x \tag{分配律}\\ 185 | -10&=x \tag{减法法则}\\ 186 | x&=-10 \tag{两边调换位置}\\ 187 | \end{align*} 188 | 189 | 这就是说:从今年起,$-10$年后(也就是10年前) 190 | 甲馆藏书册数就是乙馆的三倍. 191 | \end{solution} 192 | 193 | 可见,代数解法因为启用了字母表示所求的数, 194 | 所得结果用有理数的意义很容易做出问题的答案,如 195 | 果还用算术解法,就要经过一番周折,多费一些心血 196 | 去说明了.这也是代数解法较算术解法优越的表现之一. 197 | 198 | \begin{ex} 199 | \begin{enumerate} 200 | \item 试用算术、代数两种解法解下列各题: 201 | \begin{enumerate} 202 | \item 某数的4倍减去3,恰等于13,求某数. 203 | \item 现有一堆小球,分给若干儿童,若每人平均分给5个, 204 | 最后缺2个小球,若每人平均分给4个,又多余3个小 205 | 球.试问:有几个儿童?几个小球? 206 | \end{enumerate} 207 | 208 | \item 你能说说代数解法有几个步骤吗? 209 | \end{enumerate} 210 | \end{ex} 211 | 212 | \subsection{未知数和方程式} 213 | 由前边三个应用题的代数解法,不难看出,这种 214 | 解法的一般做法要点是: 215 | 216 | 用一些字母$x, y,\ldots$来表示所要求的数量,我 217 | 们称为“\textbf{未知数}”.就是用来表达“未知量”的符 218 | 号. 219 | 220 | 用含有未知数的算式,表达问题中所涉及到的其 221 | 它数量. 222 | 223 | 把问题中的数量关系,平铺直叙、直截了当地用 224 | 等式表示出来,这个等式我们称为\textbf{方程式}. 225 | 226 | 运用“通性”与等式性质,由所列方程式求出\textbf{未 227 | 知数应有的值}. 228 | 229 | 在以上做法中,由引进未知数到列出等量关系式 230 | (方程),只是解应用题的准备工作;而有效地应用 231 | “通性”、等式性质却是代数解法的关键所在.但要 232 | 注意:准备工作是基础,必须做得充分、熟练、正 233 | 确.否则,是谈不上能够正确解决问题的. 234 | 235 | 因此,我们还要首先讨论方程式的意义及怎样列 236 | 方程式,然后再进一步探讨求出“未知数应有的值” 237 | 的原理和方法. 238 | 239 | 正如前面三个例题的代数解法中,通过引入未知 240 | 数,我们列出了等式: 241 | \[4x+x=138\] 242 | \[0.08y+(12-y)\x 0.04=0.64\] 243 | \[23+0.5x=3(11+0.5x)\] 244 | 245 | 我们就把这些\textbf{含有未知数的等式,叫做方程式}. 246 | 简称\textbf{方程}.又如:$x\cdot y-3x=5$也是方程式. 247 | 248 | 在一个方程中,所含未知数,又称为\textbf{元};被“$+$” 249 | “$-$”号隔开的每一部分(包括这部分前边的“$+$”、 250 | “$-$”号在内)称为\textbf{一项};在一项中,数字或表示已 251 | 知数的字一母因数叫做未知数的\textbf{系数}.并且在各项中, 252 | 所含有的未知数的\textbf{次数和},称为\textbf{这一项的次数},如: 253 | $+ 0. 08x$, $x$, $-1$, $5x$等项的次数都是1,而$xy$项的 254 | 次数就是2;不含未知数的项,称为\textbf{常数项},它的次 255 | 数是0,因此也称为0次项. 256 | 257 | 在一个方程的各项中,最高次项的次数,就称为 258 | 这个\textbf{方程的次数}. 259 | 260 | 例如:方程$4x+x=138$中,共有三项:$4 x$, $+x$ 261 | 及138;各项的系数分别是4, $+1$和常数项138;各 262 | 项的次数分别是1、1及0;方程的次数是1. 263 | 264 | 又如:方程$xy-3x=5$中,共有三项:$xy$, $-3x$ 265 | 及5;各项的系数分别是$+1$,$-3$及常数项5;各项 266 | 的次数分别是2,1及0;方程的次数是2.等 267 | 等. 268 | 269 | 实际上,方程就是表达已知数与未知数之间的一 270 | 种等式关系,这种关系式是解决问题的基础,必须在 271 | 明确量与量之间的正确关系的前提下,适当引入未知 272 | 数,才能表达出来. 273 | 274 | \begin{example} 275 | 引入未知数,正确表达出以下问题中的等量 276 | 关系: 277 | \begin{enumerate} 278 | \item 某数的五分之一等于九,求某数. 279 | \item 某数与它的一半之和,恰好是24,求某数. 280 | \item 某两班学生总数是101人,而这两班的人 281 | 数相差三人,求这两班各多少人? 282 | \end{enumerate} 283 | \end{example} 284 | 285 | \begin{solution} 286 | \begin{enumerate} 287 | \item 设某数为$x$,则$\frac{x}{5}=9$ 288 | \item 设某数为$y$,则$y+\frac{y}{2}=24$ 289 | \item 设一个班有$x$人,另一班就有 290 | $(101-x)$人,则 $x-(101-x)=3$ 或 $(101-x)-x=3$. 291 | 292 | 也可以这样解: 设一个班有$x$人,另一个班就有$(x+3)$人, 293 | 则$$x+(x+3)=101$$ 294 | 295 | 还可以引入两个未知数: 296 | 设一个班有$x$人,另一个班有$y$人, 297 | 则由题目可知,应该同时有两个关系成立: 298 | \[\begin{cases} 299 | x+y=101\\ 300 | x-y=3 301 | \end{cases}\] 302 | \end{enumerate} 303 | \end{solution} 304 | 305 | 306 | \begin{ex} 307 | \begin{enumerate} 308 | \item 试说明下列各方程式有哪些项组成?各项的系数是什么? 309 | 方程的次数是几? 310 | \begin{multicols}{2} 311 | \begin{enumerate} 312 | \item $\frac{1}{2}x-5x=7$ 313 | \item $3x^2-5=2x$ 314 | \item $-0.05+7.2x=x$ 315 | \item $x-xy=0$ 316 | \end{enumerate} 317 | \end{multicols} 318 | 319 | \item 引入一个未知数,列出方程式: 320 | \begin{enumerate} 321 | \item 用某数的2倍去乘$-\frac{1}{3}$,正好是$-10$,求某数. 322 | \item 用2元钱去买回若干本书,每本书两角钱,还找回了六角 323 | 钱,问买回几本书? 324 | \item 高为4米,长比宽多2米的长方体体积为140米$^3$.求这 325 | 个长方体的长和宽. 326 | \item 两数之和为36,若已知一数是$m$,试求另一数.(注意: 327 | $m$表示已知数). 328 | \item 浓度为20\%的糖水300克中,含有多少克糖?多少克水? 329 | \[\text{浓度}=\frac{\text{糖}}{\text{水+糖}}\] 330 | \item 浓度为20\%的糖水150克中,加入50克水,浓度就变成 331 | 百分之几? 332 | \end{enumerate} 333 | \end{enumerate} 334 | \end{ex} 335 | 336 | \subsection{方程的解与解方程的原理} 337 | 根据应用题所给条件,引入未知数,正确的列出 338 | 方程式,只是为解决问题提供了基础,作好了必要的 339 | 准备.要使问题得到完满解决,就要从所列方程式出 340 | 发,求出未知数应有的值来,并加以验证. 341 | 342 | 在这里所说的\textbf{未知数应取的值是指:把所列方程 343 | 中的未知数换成这个值以后,就使它变成一个恒等 344 | 式}.也就是说:把未知数应取的值代入原方程中,能 345 | 使原方程成为“真正等式”.例如: 346 | \begin{itemize} 347 | \item 把$x = 49$代入$x+ (x+3)=101$, 得$101=101$; 348 | \item 把$y=7$代入$2-0.2y=0.6$, 得$0 .6=0. 6$; 349 | \item 把$x=1$与$y=8$同时代入$xy-3x=5$, 得$5=5$,等等. 350 | \end{itemize} 351 | 352 | 象这样,\textbf{能使方程式成为真正等式的未知数的 353 | 值,叫做方程的解}.或简单说:\textbf{使方程成立的未知数 354 | 的值},叫做\textbf{方程的解}.(一元方程的\textbf{解},也叫做\textbf{根}).例如: 355 | \begin{itemize} 356 | \item $x = 49$是方程$x+ (x+3)=101$的解(或根); 357 | \item $y=7$是方程$2-0.2y=0.6$的解(或根); 358 | \item $x=1$与$y=8$是方程$xy-3x=5$的解(或根)等. 359 | \end{itemize} 360 | 361 | 362 | \begin{ex} 363 | 观察一下,下列方程的解各是什么?为什么? 364 | \begin{multicols}{2} 365 | \begin{enumerate} 366 | \item $2x+1=6$ 367 | \item $x+3=2x$ 368 | \item $xy=0$ 369 | \item $0.5x+1=3$ 370 | \end{enumerate} 371 | \end{multicols} 372 | \end{ex} 373 | 374 | 求方程的解的过程,叫做\textbf{解方程}. 375 | 376 | 解方程不能只靠观察、试验,而需要有系统地、 377 | 有根据地去探求普遍方法.这就要用“数系运算通 378 | 性”及“等式性质”作为我们的有力工具了. 379 | 380 | \textbf{解方程的原理}是十分简朴的,那就是:对于一个 381 | 方程式中的未知数与已知数进行统一考虑,因为它们 382 | 都是数,又同在一个等式当中,所以,它们在运算上 383 | 就应该满足“通性”及等式的性质.只要我们有效地 384 | 应用这些性质,就可以逐步变形,求出方程式的解 385 | (根)来. 386 | 387 | 下面以方程$4x+2(12-x) =32$为例,分析一下 388 | 是如何有效地应用这些性质(特别是分配律)去求出 389 | 方程的解的,从中我们可以总结出代数解法的一般方 390 | 法和规律. 391 | 392 | 首先,由于在方程$4x+2(12-x)= 32$中,含有未 393 | 知数$x$,因而无法按运算顺序(先括号里,后括号外) 394 | 进行变形.这时候,分配律就要大显身手发挥作用 395 | 了,运用它可以“小表及里”地把括号打开,将方程 396 | 变形为:$4x+2\cdot 12-2 \cdot x = 32$; 397 | 398 | 其次,运用加法交换、结合律及分配律,可以逐 399 | 步将方程变形如下: 400 | \begin{equation} 401 | \begin{split} 402 | \underline{(4x-2x)}+24&=32\\ 403 | \underline{(4-2)x}+24&=32\\ 404 | \underline{2x}+24&=32 405 | \end{split} 406 | \end{equation} 407 | 408 | 由此,可总结出方程变形的第一个规律: 409 | 410 | \textbf{运用“通性”}(特别是运用\textbf{分配律})\textbf{可以“由 411 | 表及里”的将括号打开,并将“含有相同未知数且所 412 | 含未知数的次数也相同”的各项结合起来,合并在一 413 | 起}——这叫做\textbf{合并同类项}.从而使方程式简化. 414 | 415 | 再次,对于方程(2.3),利用等式性质可以继续 416 | 变形,将方程$2x+24 = 32$两边同加$(-24)$得: 417 | \[2x = 32 -24\] 418 | 419 | 可以发现,利用等式性质3进行的这一变形,就 420 | 相当于把方程$2x+24 = 32$中的左边一项$+24$,\textbf{改变符 421 | 号后,移到右边去},就得到方程式: 422 | \begin{equation} 423 | \begin{split} 424 | 2x &= 32 -24\\ 425 | 2x&=8 426 | \end{split} 427 | \end{equation} 428 | 429 | 由此,又可以总结出方程变形的第二个规律: 430 | 431 | \textbf{运用等式性质3把方程一边的任意一项改变符号 432 | 以后,移到方程的另一边}——这叫做\textbf{移项}.简单说, 433 | 就是“移项变号”. 434 | 435 | 最后,由方程(2.4),只要再用等式性质40就可以变形为最简单的形式,未知数应取的值也就一目 436 | 了然了.这就是: 437 | 438 | 将方程$2x = 8$两边同乘(或同除以)一个非零数$\frac{1}{2}$(或2),即可得出 439 | \begin{equation} 440 | x=4 441 | \end{equation} 442 | 443 | 由此,还可以总结出方程变形的第三个规律: 444 | 445 | \textbf{运用等式性质4,把方程两边各项同除以未知数 446 | 的系数(或乘以系数的倒数)},使方程化为最简形 447 | 式.得到未知数应取的值. 448 | 449 | 综合以上三条规律,得到\textbf{解方程的具体方法是: 450 | 展开括号、移项变号、合并同类项、除以未知数的系 451 | 数,使方程化为最简形式}.当然,在求出未知数的值 452 | 以后,还要代入原方程加以检验,最后确定出原方程 453 | 的解. 454 | 455 | \begin{example} 456 | 解方程$9x-5=5-x$ 457 | \end{example} 458 | 459 | 460 | \begin{solution} 461 | 先“移项变号”得 462 | \[9x+x=5+5\] 463 | 再“合并同类项”得: 464 | \[10x=10\] 465 | 最后“除以$x$的系数10”得: 466 | \[x=1\] 467 | 把$x=1$代入原方程的两边,进行检验: 468 | \[\text{左边}=9\x 1-5=4,\qquad \text{右边}=5-1=4\] 469 | 470 | $\therefore\quad $ 左边$=$右边 471 | 472 | $\therefore\quad x=1$ 是原方程$9x-5=5-x$的解. 473 | \end{solution} 474 | 475 | 476 | \begin{example} 477 | 解方程$-5=5x-7(1-x)$ 478 | \end{example} 479 | 480 | \begin{solution} 481 | \begin{align*} 482 | -5&=5x-7+7x \tag{用分配律展开括号}\\ 483 | -7x-5x &= 5-7 \tag{移项变号}\\ 484 | -12x&=-2 \tag{合并同类项}\\ 485 | x&=\frac{1}{6} \tag{两边除以$-12$} 486 | \end{align*} 487 | 把$x=\frac{1}{6}$代入原方程两边,经检验知: 488 | $\frac{1}{6}$是方程 $-5=5x-7(1-x)$ 的解. 489 | \end{solution} 490 | 491 | 492 | \begin{ex} 493 | \begin{enumerate} 494 | \item “解方程”和“方程的解”一样吗?区别是什么? 495 | \item 解方程的原理是什么?根据原理又得出些什么样具体方法 496 | 和规律? 497 | \item 试用解方程的具体规律,将下列各方程化为最简单形式: 498 | \begin{multicols}{2} 499 | \begin{enumerate} 500 | \item $x-7=9$ 501 | \item $6-y=y-5$ 502 | \item $2x+\frac{1}{2}=0$ 503 | \item $12=6x-9(2-x)$ 504 | \end{enumerate} 505 | \end{multicols} 506 | \end{enumerate} 507 | \end{ex} 508 | 509 | \section*{习题2.1} 510 | \addcontentsline{toc}{subsection}{习题2.1} 511 | 512 | \begin{enumerate} 513 | \item 试用算术、代数两种解法解下列各题,并比较优劣: 514 | \begin{enumerate} 515 | \item 某生产队今年植树18000棵,正是去年植树的2 516 | 倍还多400棵.问:去年植树多少棵? 517 | \item 一本864页的书,每页55行,每行40个字;再版 518 | 时计划每页增印5行,每行又多印8个字.问:再版这本书 519 | 时能比原版减少几页? 520 | \item 兄、弟二人,今年分别为15岁和9岁.问:几年 521 | 后,兄是弟的年龄的2倍? 522 | \end{enumerate} 523 | 524 | \item 引入未知数$x$,把下列问题中所求的量,用含有未知数 525 | 的算式列出来: 526 | \begin{enumerate} 527 | \item $x$的$1\frac{1}{2}$倍与$-7$的代数和. 528 | \item $x$的相反数与27的差. 529 | \item $x$与已知数$a$的平方和再减去10所得的差. 530 | \item $x$的20\%与51的差的一半. 531 | \item 浓度为51\%的盐水$x$克中,含有的纯盐量与含水 532 | 量. 533 | \item $x$克盐溶化到100克水中,盐水的浓度. 534 | \item 一件工作,小李$x$天作完,求:每天的工作量和 535 | 3天的工作量. 536 | \item 以每小时20公里的速度,要走出$x$公里的路程, 537 | 需要多少时间? 538 | \item 操场上有400米一圈的跑道,速度分别是$x$米/秒 539 | 和2$x$米/秒的两个人,同时,同地相背而跑时,在什么时间 540 | 相遇? 541 | \item 上题中,如果两人同向跑时,快的在什么时间追 542 | 上慢的. 543 | \end{enumerate} 544 | 545 | \item 适当引入未知数,列出下边问题的方程: 546 | \begin{enumerate} 547 | \item 某数与五的和,正好是这个数的3倍,试求某 548 | 数. 549 | \item 矩形的周长是40,长比宽多10,试求这个矩形的 550 | 面积. 551 | \item 浓度为85\%的酒精100斤与浓度为15\%的酒精100 552 | 斤混合在一起以后,它的浓度是多少? 553 | \item 甲、乙二人由同地出发,甲以每小时走5里的速 554 | 度先出发1.5小时,乙骑自行车经过50分钟才赶上甲.试 555 | 求:乙每小时骑车能走多少里? 556 | \end{enumerate} 557 | 558 | \item 指出下列各方程中:有那些项,各项的系数、次数分别 559 | 是多少?方程是几次的? 560 | \begin{multicols}{2} 561 | \begin{enumerate} 562 | \item $7x+1=0$ 563 | \item $-\frac{1}{5}x-1=x+10$ 564 | \item $0=0.1x$ 565 | \item $1-xy=3x$ 566 | \end{enumerate} 567 | \end{multicols} 568 | 569 | \item 利用解方程原理得出的具体规律,把下列方程式化为 570 | 最简形式: 571 | \begin{multicols}{2} 572 | \begin{enumerate} 573 | \item $\frac{1}{2}x=-1$ 574 | \item $\frac{1}{3}x+1=\frac{1}{3}$ 575 | \item $3-x=6$ 576 | \item $-7+2(1-x)=0$ 577 | \end{enumerate} 578 | \end{multicols} 579 | 580 | \item 解下列方程,并注明每步变形的根据: 581 | \begin{multicols}{2} 582 | \begin{enumerate} 583 | \item $x-1=2x$ 584 | \item $0=7x-\frac{1}{2}$ 585 | \item $8-10y=-32-5y$ 586 | \item $\frac{1}{4}=2x-\frac{1}{2}(1-4x)$ 587 | \end{enumerate} 588 | \end{multicols} 589 | 590 | \item 如果$a, b$都是已知的数,并且$a\ne -1$,试利用解方程的 591 | 原理,解方程: 592 | \[ax+1=b-x,\qquad b-ax=\frac{1}{2}+x \] 593 | 594 | \end{enumerate} 595 | 596 | \section{一元一次方程} 597 | \subsection{一元一次方程} 598 | 在第一节所列举的方程式中,可以找出具有这样 599 | 特点的方程: 600 | \begin{enumerate} 601 | \item 只含有一个未知数; 602 | \item 分母不含有未知数; 603 | \item 方程的次数是1次. 604 | \end{enumerate} 605 | 比如:$4x+x = 138$;$\frac{x}{5}=9$;$0.08y+0.04(12-y)=0.64$;$x+\frac{x}{2}=24$等 606 | 都属于这一类方程式. 607 | 608 | 我们把\textbf{只含一个未知数、分母不含未知数,且次 609 | 数又是1的方程,称为一元一次方程}. 610 | 611 | 一元一次方程的一般形式是: 612 | \[ax+b=0\quad \text{($a,b$是已知数,且$a\ne 0$)}\] 613 | 614 | \subsection{一元一次方程的解法} 615 | 我们利用由“通性”及等式性质而归纳出来的代 616 | 数解法的具体规律,来讨论一元一次方程的解法. 617 | 618 | \begin{example} 619 | 解方程$8x=5x-3$ 620 | \end{example} 621 | 622 | 623 | \begin{solution} 624 | 原方程移项得\quad $8x-5x=-3$ 625 | 626 | 合并同类项得\quad $3x=-3$ 627 | 628 | 两边除以3得\quad $x=-1$ 629 | \end{solution} 630 | 631 | \textbf{验算:} 把$x=-1$代入原方程中的两边, 632 | \[\begin{split} 633 | \text{左}&=8\x(-1)=-8,\\ 634 | \text{右}&=5\x(-1)-3=-8, 635 | \end{split}\] 636 | 左边$=$右边,$\therefore\quad $原方程的解是$x=-1$. 637 | 638 | \begin{example} 639 | 解方程$7(1-x)=2(x+3)-4 (5+4x)$. 640 | \end{example} 641 | 642 | 643 | \begin{solution} 644 | 先运用分配律去括号: 645 | \[7-7x=2x+6-20-16x\] 646 | 移项:把含$x$的各项移至方程左边,而把所有的常数项移至右边. 647 | \[ -7x-2x+16x=6-20-7\] 648 | 合并同类项:分别求含x项的系数的代数和与常数项的代数和: 649 | \[7x=-21\] 650 | 两边除以7:$x=-3$. 651 | \end{solution} 652 | 653 | \textbf{验算:} 654 | $x=-3$代入原方程两边, 655 | \[\begin{split} 656 | \text{左}&=7[1-(-3)]=28,\\ 657 | \text{右}&=2(-3+3)-4 [5+4(-3)]=28, 658 | \end{split}\] 659 | 左边$=$右边,$\therefore\quad $原方程的解是$x=-3$. 660 | 661 | 662 | 663 | \begin{example} 664 | 解方程$3(3y+1)=2(1+y)+3(y+3)$ 665 | \end{example} 666 | 667 | \begin{solution} 668 | 把原方程去括号: $9y+3=2+2y+3y+9$ 669 | 670 | 移项:$9y-2y-3y=2+9-3$ 671 | 672 | 合并同类项:$4y=8$ 673 | 674 | 除以$y$的系数4:$y=2$ 675 | \end{solution} 676 | 677 | 同学可以自己验算,从而得到: 原方程的解是$y=2$. 678 | 679 | 680 | \begin{ex} 681 | 解下列方程: 682 | \begin{multicols}{2} 683 | \begin{enumerate} 684 | \item $0=5x-1$ 685 | \item $x+5=-2-6x$ 686 | \item $3(1-x)+1=-(x-1)$ 687 | \item $5(x-1)-3(x-1)=0$ 688 | \end{enumerate} 689 | \end{multicols} 690 | \end{ex} 691 | 692 | \begin{example} 693 | 解方程$0.3(8x-1)=2x+2.9$ 694 | \end{example} 695 | 696 | \begin{solution} 697 | \begin{align*} 698 | 3 (8x-1)&=20x+29 \tag{原方程两边乘以10}\\ 699 | 24x-3&=20x+29 \tag{去括号}\\ 700 | 24x-20x&=29+3\tag{移项变号}\\ 701 | 4x &= 32\tag{合并同类项}\\ 702 | x&=8\tag{两边除以4}\\ 703 | \end{align*} 704 | 经验算以后,可知: 705 | 原方程的解是$x=8$. 706 | \end{solution} 707 | 708 | \begin{ex} 709 | 解下列方程: 710 | \begin{multicols}{2} 711 | \begin{enumerate} 712 | \item $0.1x+1=0.2$ 713 | \item $0.16(x+5)=1-0.2x$ 714 | \item $0.625=0.5(y-2)$ 715 | \item $-0.1(x-10)=9-0.2(1-x)$ 716 | \end{enumerate} 717 | \end{multicols} 718 | \end{ex} 719 | 720 | \begin{example} 721 | 解方程 722 | $\frac{2y-1}{3}-1=\frac{5y+1}{8}-\frac{3y+1}{6}$ 723 | \end{example} 724 | 725 | \begin{analyze} 726 | 遇到分数系数的方程时,可以先由等式 727 | 性质,两边乘以“各分母的最小公倍数”,把系数的 728 | 分母去掉,转化为整数系数方程以后,再求解.本例 729 | 中3,8,6的最小公倍数为24. 730 | \end{analyze} 731 | 732 | \begin{solution} 733 | 方程两边各项乘以24 734 | \begin{align*} 735 | 8 (2y-1)-24&=3 (5y+1)-4 (3y+1) \tag{去分母}\\ 736 | 16y-8-24&=15y + 3-12y-4 \tag{去括号}\\ 737 | 16y-15y+12y&=3-4+8+24 \tag{移项}\\ 738 | 13y&=31 \tag{合并同类项}\\ 739 | y&=\frac{31}{13}=2\frac{5}{13} \tag{两边除以13} 740 | \end{align*} 741 | 经验算(可以口算或在草稿纸上笔算,不写入解 742 | 题过程中)后知道,原方程的解是$y=2\frac{5}{13}$. 743 | \end{solution} 744 | 745 | \begin{ex} 746 | 解下列方程: 747 | \begin{multicols}{2} 748 | \begin{enumerate} 749 | \item $\frac{1}{5}+x=-1$ 750 | \item $\frac{1}{4}(1-5x)=0.5$ 751 | \item $\frac{3}{7}(1-x)=1-\frac{x}{7}$ 752 | \item $-\frac{1}{8}(24-16x)=-\frac{1}{9}(18x+9)$ 753 | \end{enumerate} 754 | \end{multicols} 755 | \end{ex} 756 | 757 | 归纳以上例题,可以看出: 758 | \begin{blk}{} 759 | 解一元一次方程的一般步骤为 760 | \begin{enumerate}[I.] 761 | \item 去分母(或化为整系数). 762 | \item 去括号. 763 | \item 移项变号. 764 | \item 合并同类项、化为$ax=-b$的形式$(a\ne 0)$. 765 | \item 除以未知数的系数得:$x=-\frac{b}{a}$. 766 | \end{enumerate} 767 | \end{blk} 768 | 769 | 这里还要指出: 770 | \begin{enumerate} 771 | \item 由于方程的形式多样,在解的 772 | 过程中不必死套以上五步,要根据具体情况,灵活应 773 | 用. 774 | \item 验算可以不必写出,但一定要自己口算或笔算 775 | 进行检验,以确实保证计算正确. 776 | \end{enumerate} 777 | 778 | \begin{example} 779 | 解下列方程: 780 | \begin{enumerate} 781 | \item $25(1-2x)=12.5+12.5(1-4x)$ 782 | \item $\frac{1}{2}(4x+6)=-1+2x$ 783 | \end{enumerate} 784 | \end{example} 785 | 786 | \begin{solution} 787 | \begin{enumerate} 788 | \item 观察所给方程,显然用不着化小数系 789 | 数为整数,去括号以后,一合并即可“凑整”.因 790 | 此 791 | \begin{align*} 792 | 25-50x&=12.5+12.5-50x \tag{去括号}\\ 793 | -50x+50x&=25-25 \tag{移项}\\ 794 | 0\cdot x&=0 \tag{合并同类项} 795 | \end{align*} 796 | 797 | 显然,$x$可以是任意数. 798 | 799 | 事实上,当$x$取任意数时,原方程两边都能得 800 | 到相等的结果. 801 | 802 | 所以,原方程的解是:$x$取任意数(无限多个). 803 | 804 | \item \begin{align*} 805 | 2x+3&=-1+2x \tag{去括号}\\ 806 | 2x-2x&=-1-3 \tag{移项}\\ 807 | 0\cdot x&=-4 \tag{合并同类项} 808 | \end{align*} 809 | 显然,无论$x$取什么值,方程的左边总是0, 810 | 边总是$-4$,不可能使左、右相等;这也就说明, 811 | 论x取什么值,总是不能使原方程式两边的值相等. 812 | 813 | 这时,我们就说:原方程式无解. 814 | \end{enumerate} 815 | \end{solution} 816 | 817 | \begin{ex} 818 | 解下列方程: 819 | \begin{enumerate} 820 | \item $4\left[\frac{1}{4}-\frac{1}{2}(3x-1)\right]=2[0.5(-3-x)+1]$ 821 | \item $7-9x=3\left(2\frac{1}{3}-x\right)-6x$ 822 | \item $\frac{1}{5}(10-20x)=7-4x$ 823 | \item $\frac{x}{2}+\frac{x}{3}+\frac{x}{6}-7=2x+5$ 824 | \end{enumerate} 825 | \end{ex} 826 | 827 | 828 | \begin{example} 829 | 解方程$\frac{1}{2}\left\{\frac{1}{3}\left[\frac{1}{4}\left(\frac{0.1x-0.5}{0.5}\right)-2\right]-3\right\}-4=0$ 830 | \end{example} 831 | 832 | \begin{analyze} 833 | 方程左边的算式很繁,但很有规律,特 834 | 别是小括号内的算式$\frac{0.1x-0.5}{0.5}$,可以利用分数的 835 | 基本性质,分子、分母同乘以10,就变成为 836 | $\frac{x-5}{5}=\frac{1}{5}x-1$, 837 | 因此原方程就可以写成: 838 | \[\frac{1}{2}\left\{\frac{1}{3}\left[\frac{1}{4}\left(\frac{1}{5}x-1\right)-2\right]-3\right\}-4=0 \] 839 | 这时,也不一定非要先去括号,或先去分母,那样计 840 | 算稍繁,为计算方便,可逐步进行去分母与合并. 841 | \end{analyze} 842 | 843 | \begin{solution} 844 | 原方程式可写成:\[\frac{1}{2}\left\{\frac{1}{3}\left[\frac{1}{4}\left(\frac{1}{5}x-1\right)-2\right]-3\right\}-4=0 \] 845 | \begin{align*} 846 | \frac{1}{3}\left[\frac{1}{4}\left(\frac{1}{5}x-1\right)-2\right]-3 &=8 \tag{两边乘以2}\\ 847 | \frac{1}{3}\left[\frac{1}{4}\left(\frac{1}{5}x-1\right)-2\right]&=11 \tag{移项、合并}\\ 848 | \frac{1}{4}\left(\frac{1}{5}x-1\right)-2&=33 \tag{两边乘以3}\\ 849 | \frac{1}{4}\left(\frac{1}{5}x-1\right)&=35 \tag{移项、合并}\\ 850 | \frac{1}{5}x-1&=140 \tag{两边乘以4}\\ 851 | \frac{1}{5}x&=141 \tag{移项、合并}\\ 852 | x&=705 \tag{两边乘以5}\\ 853 | \end{align*} 854 | 855 | 经检验,原方程的解是$x=705$. 856 | \end{solution} 857 | 858 | \begin{ex} 859 | 解下列方程: 860 | \begin{enumerate} 861 | \item $\frac{0.1-0.3x}{0.2}=5$ 862 | \item $\frac{x-3}{0.5}-\frac{x+4}{0.2}=16$ 863 | \item $\frac{3}{2}\left[\frac{2}{3}(x-1)-2\right]=3$ 864 | \item $\frac{1}{3}\left\{\frac{1}{3}\left[\frac{1}{3}\left(\frac{1}{3}-1\right)-1\right]-1\right\}-1=0$ 865 | \end{enumerate} 866 | \end{ex} 867 | 868 | \section*{习题2.2} 869 | \addcontentsline{toc}{subsection}{习题2.2} 870 | 871 | \begin{enumerate} 872 | \item 检验以下各题方括号中所给出的数是不是所给方程的解 873 | (或根)? 874 | \begin{enumerate} 875 | \item $2 (3x-4)=5(x-2),\qquad [3,\; -2]$ 876 | \item $ x(x+ 1)=12,\qquad [3,\; -3,\; 4,\; -4]$ 877 | \item $\frac{x}{2}+\frac{x}{3}+\frac{x}{4}=x,\qquad \left[-0.1,\; -2,\; \frac{1}{4},\; 7,\; -101,\; 0,\; 1981\right]$ 878 | \item $9-10x=\frac{1}{4}(2-36x)-x,\qquad [0,\; 1,\; -1,\; a]$ 879 | \end{enumerate} 880 | 881 | \item 解方程: 882 | \begin{multicols}{2} 883 | \begin{enumerate} 884 | \item $0.8x=1$ 885 | \item $7x-1=5x$ 886 | \item $0=6x-\frac{1}{2}$ 887 | \item $4-7x=7x-4$ 888 | \item $2|x|=2$ 889 | \item $1=3-|x|$ 890 | \end{enumerate} 891 | \end{multicols} 892 | 893 | \item 解方程: 894 | \begin{enumerate} 895 | \item $-5(x-1)=0$ 896 | \item $1-2(x-3)=x$ 897 | \item $10y+7=3(4y-1)+4(1-y)$ 898 | \item $9=2(3x-1)+x-4$ 899 | \item $4(2t+3)=8(1-t)-5(t-2)$ 900 | \item $5(z-1)-9(1-z)+3(2z-2)-(z-1)=2$ 901 | \end{enumerate} 902 | 903 | \item 解下列方程: 904 | \begin{enumerate} 905 | \item $-0.3(1-x)+0.1=2$ 906 | \item $0.7=x-0.2(5-x)$ 907 | \item $0.2(2x-1)-0.5(2-4x)+1=0$ 908 | \item $7(2x+1)-3(4x+2)+5(x+0.5)-1=0$ 909 | \item $1.8-8x-0.6(1.3-3x)=4(5x-0.4)$ 910 | \end{enumerate} 911 | 912 | \item 解方程: 913 | \begin{multicols}{2} 914 | \begin{enumerate} 915 | \item $\frac{7x-5}{4}=\frac{3}{8}$ 916 | \item $\frac{3-x}{2}=\frac{x-4}{3}$ 917 | \item $\frac{2x-1}{6}-\frac{5x-1}{8}=1$ 918 | \item $\frac{3x-1}{2}+x=\frac{2x+1}{5}-1$ 919 | \item $y-\frac{y-1}{2}=2-\frac{y-2}{5}$ 920 | \item $\frac{2}{5}y+\frac{1}{9}=\frac{1}{9}y-\frac{2}{5}$ 921 | \item $\frac{x-2}{5}-\frac{x+3}{10}-\frac{2x-5}{3}+3=0$ 922 | \item $\frac{9t+2}{7}-\frac{3+2t}{3}-\frac{3t-14}{2}=1$ 923 | \item $1\frac{1}{2}x-\frac{14-x}{3}=3x$ 924 | \item $x-\frac{3x+6}{2}+\frac{3x+6}{5}=0.5x$ 925 | \end{enumerate} 926 | \end{multicols} 927 | \item 用解方程的方法,求以下问题中的未知量: 928 | \begin{enumerate} 929 | \item 已知公式$\ell=\ell_0(1+\alpha t)$中,$\ell= 80.096$,$\ell_0 = 80$,$\alpha=0.000012$,试求$t$. 930 | \item 已知一梯形的面积是120$cm^2$,上底是12$cm$,高 931 | 是8$cm$,试求下底长是多少? 932 | \item 在公式$S=\frac{1}{2}at^2$中,$S=168$, $t=4$,试求$a$. 933 | \item 在公式$F=32+1\frac{4}{5}C$中,已知$F=77$,试求$C$. 934 | \end{enumerate} 935 | 936 | \item 下列各方程的解法是否正确?如果有错,请把它改正过 937 | 来: 938 | \begin{enumerate} 939 | \item 解: 940 | \[\begin{split} 941 | 3(x-1)&=7-x\\ 942 | 3x-3&=7-x=3x+x=7+3\\ 943 | &=4x=10\\ 944 | &=x=2\frac{1}{2} 945 | \end{split}\] 946 | $\therefore\quad x=2\frac{1}{2}$ 947 | \item 解: 948 | \[\begin{split} 949 | \frac{y}{2}+\frac{y}{4}&=1\\ 950 | 2y+y&=1\\ 951 | 3y&=1 952 | \end{split}\] 953 | $\therefore\quad y=\frac{1}{3}$ 954 | \item 解: 955 | \[\begin{split} 956 | \frac{x+1}{3}-\frac{x-2}{6}&=\frac{4+x}{2}\\ 957 | 2x+2-x-2&=12+3x\\ 958 | 2x-x-3x&=12\\ 959 | -2x&=12 960 | \end{split}\] 961 | $\therefore\quad x=-6$ 962 | \item 解: 963 | \[\begin{split} 964 | \frac{0.1x-0.5}{0.2}+1&=0\\ 965 | \frac{x-5}{2}+10&=0\\ 966 | x-5+20&=0\\ 967 | x&=-20+5 968 | \end{split}\] 969 | $\therefore\quad x=-15$ 970 | \end{enumerate} 971 | 972 | \item 解下列方程: 973 | \begin{enumerate} 974 | \item $\frac{0.4 y+0.9}{0.5}-\frac{0.03-0.02 y}{0.03}-\frac{y-5}{2}=0$ 975 | \item $x-\frac{1}{2}\left[x-\frac{1}{2}(x-1)\right]=\frac{2}{3}(x-1)$ 976 | \item $\frac{1}{2}\left\{\frac{1}{2}\left[\frac{1}{2}\left(\frac{1}{2} y-3\right)-3\right]-3\right\}-3=0$ 977 | \item $\frac{1}{8}\left\{4\left[\frac{5}{8}(t-1)+\frac{3}{8}(1-t)\right]-7(1-t)\right\}=100$ 978 | \item $\frac{9}{8}\left[\frac{2}{3}\left(\frac{4}{5} x-\frac{2}{3}\right)+1\right]=\frac{2}{3} x\left(\frac{1}{2}-\frac{1}{3}\right)-\frac{5}{8}$ 979 | \item $\frac{1}{2}\left\{x-\frac{1}{3}\left[x-\frac{1}{4}\left(x-\frac{2}{3}\right)-\frac{3}{2}\right]\right\}=x+\frac{3}{4}$ 980 | \end{enumerate} 981 | 982 | \item 解下列方程: 983 | \begin{multicols}{2} 984 | \begin{enumerate} 985 | \item $|x-4|=0$ 986 | \item $|x+1|=5$ 987 | \item $|x-1|+1=2$ 988 | \item $|2-x|=3|x-2|$ 989 | \end{enumerate} 990 | \end{multicols} 991 | 992 | \item 解下列文字系数的一元一次方程: 993 | \begin{multicols}{2} 994 | \begin{enumerate} 995 | \item $ax+1=0\quad (a\ne 0)$ 996 | \item $5x-a+1=0$ 997 | \item $ax+5=x-3\quad (a\ne 1)$ 998 | \item $ax+b=8-x\quad (a\ne -1)$ 999 | \item $\frac{x}{a}+\frac{x}{b}=1\quad (a+b\ne 0)$ 1000 | \end{enumerate} 1001 | (这里的字母$a,b$都是已知数) 1002 | \end{multicols} 1003 | 1004 | \item 如果已知方程$mx+2=2(m-x)$的根是$x=\frac{1}{2}$,试 1005 | 求这个方程中$m$的值. 1006 | \end{enumerate} 1007 | 1008 | 1009 | \section{一次方程组} 1010 | \subsection{二元一次方程} 1011 | 先看下边的实例: 1012 | 1013 | 今有货物10吨,要用大、小两种卡车一次运走, 1014 | 如果每辆大车能装2吨货,每辆小车只能装1吨货. 1015 | 试问:应派大、小车各几辆才能在每辆车都装满的情 1016 | 况下,正好一次运走全部货物? 1017 | 1018 | 显然,派车的方案不止一种,比如:派五辆大车 1019 | 而不派小车;或派四辆大车和两辆小车等等,一共有 1020 | 多少种可行方案呢?我们用代数解法去讨论. 1021 | 1022 | 设派出大车$x$辆,小车$y$辆,则由问题所给出的条件可以得出: 1023 | \begin{equation} 1024 | 2x+y=10 1025 | \end{equation} 1026 | 1027 | 显然这是一个方程式,它与一元一次方程相比, 1028 | 只是多了一个未知数(元).而其余的特点是一样 1029 | 的:分母不含未知数、方程的次数是1,因此,我们 1030 | 就把“\textbf{含有两个未知数且分母不含未知数的一次方 1031 | 程}”,叫做\textbf{二元一次方程式}. 1032 | 1033 | \textbf{能够使二元一次方程两边的值相等的未知数$x,y$ 1034 | 的一组值},叫做这个\textbf{二元一次方程的一个解},记作 1035 | $(x,y)$.比如:方程(2.6)的一个解,可以记作: 1036 | $ (x, y)=(5, 0)$,这就是说,把$x=5$, $y=0$这一 1037 | 组值代入(2.6)的两边,计算结果是相等的. 1038 | 1039 | 前边已经说过,我们所提问题的“派车方案”不 1040 | 止一种,因此,方程(2.6)的解就不止一个.但由于 1041 | 这里的$x, y$都表示所派的卡车辆数,因而它们的值 1042 | 就只能是\textbf{非负整数}了.否则,所得结果就毫无意义 1043 | 了.用列表的方法,很容易把所有的“派车方案” 1044 | (也就是方程(2.6)的解)找出来,这就是: 1045 | \begin{center} 1046 | \begin{tabular}{c|cccccc} 1047 | \hline 1048 | $x$ &5&4&3&2&1&0\\ 1049 | \hline 1050 | $y$ &0&2&4&6&8&10\\ 1051 | \hline 1052 | \end{tabular} 1053 | \end{center} 1054 | 其中每一个$(x,y)$的\textbf{数组值},就是方程(2.6)的一 1055 | 个解,也就是所提问题的一种“派车方案”. 1056 | 1057 | 把所有能使方程(2.6)成立的,且符合实际问题 1058 | 的各个\textbf{数组值}所组成的\textbf{集合},可以表示为: 1059 | \[A=\{(x,y)\}=\{(5,0),\; (4,2),\;(3,4),\;(2,6),\;(1,8),\;(0,10)\}\] 1060 | 1061 | 这个集合中的每一组值,都是(2.6)的一个解, 1062 | 我们把集合刀叫做方程(2.6)的符合题意的\textbf{解集}. 1063 | 1064 | 应该指出:如果不考虑具体问题,只是从数学上 1065 | 考虑方程$2x + y=10$的解时,就不能限制在\textbf{非负整数} 1066 | 的范围内了,而应该考虑到所有的有理数范围.这 1067 | 时,只要任意取$x$的一个有理数值,代入方程(2.6) 1068 | 就可以求出$y$的一个相应的值,从而得到方程(2.6) 1069 | 的一个解;当然,也可以先随便取$y$的一个值,代入 1070 | 方程(2.6),就能求出$x$的一个相应值,同样可得到 1071 | 方程(2.6)的一个解.这样可以无止境的作下去,因 1072 | 此,可以得到方程(2.6)的无限多个解.如下表: 1073 | \begin{center} 1074 | \begin{tabular}{c|ccccccccc} 1075 | \hline 1076 | $x$ &$\cdots$& $-3$ & $-\frac{1}{2}$ & $-1$ & $0$ & $\frac{1}{2}$ & $0.7$ &$10.6$ &$\cdots$\\ 1077 | \hline 1078 | $y$ &$\cdots$& $16$ & $11$ & $12$ & $10$ & $9$ & $8.6$ &$-11.2$ &$\cdots$\\ 1079 | \hline 1080 | \end{tabular} 1081 | \end{center} 1082 | 1083 | 所以,我们说: 1084 | \textbf{任一个二元一次方程有无数多个解}.正因为如 1085 | 此,二元一次方程也被称为\textbf{不定方程}. 1086 | 1087 | \begin{example} 1088 | 试求二元一次方程$3x +y=8$的正整数解. 1089 | \end{example} 1090 | 1091 | \begin{analyze} 1092 | 二元一次方程虽有无数多个解,但本题 1093 | 只要求\textbf{正整数解},因而\textbf{可能}是只有有限几个解. 1094 | \end{analyze} 1095 | 1096 | \begin{solution} 1097 | 可将原方程变形为: 1098 | \begin{align*} 1099 | y=8-3x \tag{移项变号} 1100 | \end{align*} 1101 | 1102 | 然后,可以设$x$分别取正整数1、2,相应地代 1103 | 入上式,求出$y$值分别为: 1104 | \[y=8-3\x1=5, \qquad y=8-3\x 2=2\] 1105 | 1106 | 如果继续取$x=3$,代入上式可知,$y=8-3\x3=-1$,这已经不符合所要求的“正整数”解.显然, 1107 | $x$取比3大的正整数时,相应的$y$值更不是“正整 1108 | 数”了,所以方程$3x +y=8$的正整数解集是: 1109 | \[\{(x,y)\}=\{(1,5),\; (2,2)\} \] 1110 | 1111 | \end{solution} 1112 | 1113 | \begin{ex} 1114 | \begin{enumerate} 1115 | \item 求$3y=9-6x$的非负整数解集. 1116 | \item 求$3x+2y=16$的正整数解集. 1117 | \end{enumerate} 1118 | \end{ex} 1119 | 1120 | \subsection{方程组与方程组的解} 1121 | 如果我们把前边所提到的实例,另加要求条件, 1122 | 改为“要用大、小卡车共六辆,一次运走10吨货物, 1123 | 大车每辆装满2吨,小车每辆装满1吨.试问:应如 1124 | 何派车? 1125 | 1126 | 不难发现,只是增加了一个条件“只能派六辆 1127 | 车”.也就是说,用代数法解决这个问题时,不仅可 1128 | 列出方程 1129 | \begin{equation} 1130 | 2x+y=10 1131 | \end{equation} 1132 | 而且还应该增加一个条件: 1133 | \begin{equation} 1134 | x+y=6 1135 | \end{equation} 1136 | 1137 | (2.8)式同样是一个二元一次方程,其中的$x,y$同样表示大、小卡车的辆数,也应限制在\textbf{非负整数} 1138 | 范围内取值.它的解也列表如下: 1139 | \begin{center} 1140 | \begin{tabular}{c|ccccccc} 1141 | \hline 1142 | $x$ &6&5&4&3&2&1&0\\ 1143 | \hline 1144 | $y$ &0&1&2&3&4&5&6 \\ 1145 | \hline 1146 | \end{tabular} 1147 | \end{center} 1148 | 1149 | 因此,方程(2.8)的非负整数解集为: 1150 | \[B=\{(x,y)\} =\{(6,0),\; (5, 1),\; 1151 | (4, 2),\; (3, 3),\; (2, 4),\; 1152 | (1,5),\; (0,6)\} \] 1153 | 1154 | 要解决所提的问题,只要求出同时适合方程(2.7) 1155 | 和方程(2.8)的数组值,也就是求出非负整数解集$A$ 1156 | 与解集$B$的\textbf{公共解解集}就可以了.这样的解集中的数 1157 | 组值,一定满足方程(2.7)与方程(2.8),因而也就 1158 | 是我们所提问题的解决方案. 1159 | 1160 | 显然,这个公共解就是: 1161 | \[(x,y)=(4, 2)\] 1162 | 也就是说,只要“派出大车4辆,小车2辆”就符合 1163 | 这个问题的要求. 1164 | 1165 | 在此,我们把\textbf{集合$A$与集合$B$的公共部分所组成 1166 | 的新集合},叫做集合$A$与$B$的\textbf{交集合},简称\textbf{交集},记作:$A\cap B$,读成“$A$与$B$的交”. 1167 | 1168 | 如上例: 1169 | \[\begin{split} 1170 | A&=\{(5, 0),\; (4, 2),\; (3, 4),\; (2, 6),\; (1, 8),\; (0, 10)\} \\ 1171 | B&=\{(6, 0),\; (5, 1),\; (4, 2),\; (3, 3),\; (2, 4),\; (1, 5),\; (0, 6)\}\\ 1172 | A\cap B&=\{(4, 2)\} 1173 | \end{split} \] 1174 | 1175 | \begin{figure}[htp] 1176 | \begin{center} 1177 | \begin{tikzpicture}[scale=1.5] 1178 | \draw (0,0) circle (1.2); 1179 | \draw (2,0) circle (1.8); 1180 | 1181 | \fill [pattern=north west lines] (0.55,1.07) to [bend left=28] (1.2,0) to [bend left=28] (0.55,-1.07)to [bend left=34] (0.55, 1.07); 1182 | 1183 | \node at (0,.8){$A$}; 1184 | \node at (.7,.25)[fill=white]{$A\cap B$}; 1185 | \node at (.7,-.25)[fill=white]{$(4, 2)$}; 1186 | \node at (2.2,1.5){$B$}; 1187 | 1188 | \node at (-.4,.5){$(5,0)$}; 1189 | \node at (-.4,0){$(3, 4)\; (2, 6)$}; 1190 | \node at (-.4,-.5){$(1, 8)\; (0, 10)$}; 1191 | 1192 | \node at (2.5,.6){$(6, 0)\quad (5, 1)$}; 1193 | \node at (2.5,0){$(3, 3)\quad (2,4)$}; 1194 | \node at (2.5,-.6){$(1, 5)\quad (0, 6)$}; 1195 | \end{tikzpicture} 1196 | \end{center} 1197 | \caption{} 1198 | \end{figure} 1199 | 1200 | 由此可见,在解决应用问题时,还可以引入两个 1201 | 或更多个未知数,列出几个方程式,把它们联合起来 1202 | 求得公共解. 1203 | 1204 | 我们把\textbf{几个方程式联合在一起,组成一个整体}, 1205 | 就叫做\textbf{联立方程式},也叫\textbf{方程组}.一个方程组中的几个方程,应用“\{”标出.如: 1206 | \[\begin{cases} 1207 | 2x+y=10\\ 1208 | x-y=6 1209 | \end{cases},\qquad \begin{cases} 1210 | 2x=y-5\\ 1211 | 3x=-4y 1212 | \end{cases}\] 1213 | 都表示方程组. 1214 | 1215 | \textbf{含有两个未知数的一次方程组,称为二元一次方 1216 | 程组}.比如: 1217 | \[\begin{cases} 1218 | x+y=2\\ 1219 | y=1 1220 | \end{cases},\qquad \begin{cases} 1221 | x-2y=3\\ 1222 | x+y=7 1223 | \end{cases},\qquad \begin{cases} 1224 | x=2+y\\ 1225 | y=-x 1226 | \end{cases}\] 1227 | 都是二元一次方程组. 1228 | 1229 | \textbf{能够同时满足方程组中每一个方程的未知数的一 1230 | 组值},叫做\textbf{方程组的解}. 1231 | 1232 | \textbf{也就是说:一个方程组中,每个方程的解集的交 1233 | 集},就是这一\textbf{方程组的解集}.例如: 1234 | $x=4$, $y=2$能够同时满足$2x+y=10$与$x+y=6$, 1235 | 因此,$(x,y)=(4,2)$就是方程组 1236 | $\begin{cases} 1237 | 2x+y=10\\ 1238 | x+y=6 1239 | \end{cases}$的一个解. 1240 | 1241 | \begin{ex} 1242 | \begin{enumerate} 1243 | \item 检验下列方括号中的数值组$(x,y)$, 1244 | 有没有方程组$\begin{cases} 1245 | 2x+y=-5\\ 1246 | 3x=-4y 1247 | \end{cases}$的解? 1248 | \[[(4,3),\quad (4,-3),\quad (-4,3), \quad (0,-5) ] \] 1249 | 1250 | \item 如果给你一个数值组$(x, y) = (1,1)$.你能造出一个二元 1251 | 一次方程组,使它的解是这个数值组吗? 1252 | \end{enumerate} 1253 | \end{ex} 1254 | 1255 | \subsection{二元一次方程组的解法} 1256 | 求方程组的解的过程,叫做\textbf{解方程组}. 1257 | 1258 | 如何解二元一次方程呢? 1259 | 1260 | 假如按照上边所说方法:“先求出方程组中每一 1261 | 个方程的解集,再求这些解集的\textbf{交集}”去作的话,就 1262 | 会发现:这种方法按理说没有问题,但实际作起来既 1263 | 繁,又没有准儿.因为每一个方程都有无限多个解, 1264 | 全部找出来不易;要求每个方程的公共解,就更不容 1265 | 易了.为此,我们需要进一步探求二元一次方程组求 1266 | 解的普遍可行的有效方法. 1267 | 1268 | 一般来说,解二元一次方程组的关键是:\textbf{设法把 1269 | 二元转化为一元方程求解}.总称为\textbf{消元法}. 1270 | 1271 | \subsubsection{代入消元法} 1272 | 1273 | 1274 | 1275 | \begin{example} 1276 | 解方程组 1277 | \begin{numcases}{} 1278 | 2x+y=13\\ 1279 | 7x+9y=84 1280 | \end{numcases} 1281 | \end{example} 1282 | 1283 | \begin{analyze} 1284 | 方程(2.9), (2.10)既然组成一个方程 1285 | 组,因而两个方程式中的未知数$x$就是表示同一个数 1286 | 量,而未知数$y$也表示另一个相同的数量.这样,就 1287 | 可以从(2.9)中首先解出:$y= 13-2x$ (移项变号), 1288 | 再把这个关系式代入(2.10)式中,去代换(2.10)中的$y$. 1289 | 从而使(2.10)式中消去未知数$y$,变成: 1290 | \[7x+9(13-2x) = 84\] 1291 | 解这个一元一次方程,就能求出二值.再利用(2.9)式, 1292 | 就能求出$y$的相应值. 1293 | \end{analyze} 1294 | 1295 | \begin{solution} 1296 | 由(2.9)式可得 1297 | \begin{equation} 1298 | y=13-2x 1299 | \end{equation} 1300 | 把(2.11)式代入(2.10)式,可得 1301 | \[7x+9(1.3-2x)=84\] 1302 | 解这个一元一次方程,经过去括号、移项变号及 1303 | 合并同类项,可得 1304 | \[-11x=-33\] 1305 | $\therefore\quad x=3$ 1306 | 1307 | 把$x=3$再代入(2.11)式,可得 1308 | \[y=13-2\x3 =7\] 1309 | \end{solution} 1310 | 1311 | \textbf{验算:} 把$x=3$, $y=7$同时代入(2.9)与(2.10)的两 1312 | 边,代入(2.9): 1313 | \[ \text{左}=2\x 3+7=13,\qquad \text{右}=13\] 1314 | \[\text{左边}=\text{右边}\] 1315 | 1316 | 代入(2.10): 1317 | \[\text{左}=7 \x 3+9 \x7=84,\qquad \text{右}=84\] 1318 | \[\text{左边}=\text{右边}\] 1319 | 1320 | 因此,原方程组的解是$(x,y)=(3,7)$. 1321 | 即,原方程组的解集为:$\{(x,y)\}=\{(3,7)\}$. 1322 | 1323 | \begin{example} 1324 | 解方程组 1325 | \begin{numcases}{} 1326 | 3x+4y=6\\ 1327 | 2x+3y=5 1328 | \end{numcases} 1329 | \end{example} 1330 | 1331 | \begin{solution} 1332 | 由(2.13)式得: 1333 | \begin{equation} 1334 | x=\frac{5-3y}{2} 1335 | \end{equation} 1336 | (2.14)代入(2.12)得: 1337 | \[3\x \frac{5-3y}{2}+4y=6 \] 1338 | 解这个一元一次方程 1339 | \[\begin{split} 1340 | 15-9y+8y&=12\\ 1341 | -y&=-3\\ 1342 | y&=3 1343 | \end{split}\] 1344 | 把$y=3$代入(2.14)得 1345 | \[x=\frac{5-3\x 3}{2}=-2\] 1346 | 经检验可知,原方程组的解是$(x,y)=(-2,3)$,即原方程组的解集是 1347 | \[\{(x,y)\}=\{(-2,3)\} \] 1348 | \end{solution} 1349 | 1350 | 通过以上两例,可以总结这种解法的步骤和要点如下: 1351 | \begin{blk}{} 1352 | \begin{enumerate}[I. ] 1353 | \item 由方程组中的任一个方程出发,把一个 1354 | 未知数写成含有另一个未知数的算式. 1355 | \item 把这个算式代入另一个方程中去,使它 1356 | 转化为一元方程式,达到\textbf{消元}的目的. 1357 | \item 解所得的一元方程. 1358 | \item 得到一个未知数的值以后,把它再代回 1359 | I所得的算式(其实,代入原方程组中的任一 1360 | 个方程中都可以),进而求得相应的另一个未知 1361 | 数的值. 1362 | \end{enumerate} 1363 | \end{blk} 1364 | 1365 | 这种求解的方法,叫做\textbf{代入消元法},简称\textbf{代入法}. 1366 | \begin{ex} 1367 | 用代入法解下列方程组: 1368 | \begin{multicols}{2} 1369 | \begin{enumerate} 1370 | \item $\begin{cases} 1371 | 2x+3y=40\\ y=2x 1372 | \end{cases}$ 1373 | \item $\begin{cases} 1374 | 7x+2y=11\\ x-y=-1 1375 | \end{cases}$ 1376 | \item $\begin{cases} 1377 | 5x=7y-1\\ 5x=17-14y 1378 | \end{cases}$ 1379 | \item $\begin{cases} 1380 | x+y=1\\ 10x+y=5\frac{1}{2} 1381 | \end{cases}$ 1382 | \item $\begin{cases} 1383 | \frac{x}{2}+\frac{y}{2}=\frac{1}{2}\\ 2x-4y=-1 1384 | \end{cases}$ 1385 | \item $\begin{cases} 1386 | 3x+5y-8=0\\ 7x-4y-3=0 1387 | \end{cases}$ 1388 | \end{enumerate} 1389 | \end{multicols} 1390 | \end{ex} 1391 | 1392 | \subsubsection{加减消元法} 1393 | \begin{example} 1394 | 解方程组 1395 | \begin{numcases}{} 1396 | x+y=12\\ 1397 | 3x-y=4 1398 | \end{numcases} 1399 | \end{example} 1400 | 1401 | \begin{analyze} 1402 | 由于每一个方程都是一个含有未知数 1403 | 的等式,而我们所要求的正是使每个方程能同时成为 1404 | 真正等式时未知数的值.因而,等式所具有的性质, 1405 | 对这些方程当然应该有效,特别是:两个等式的两边 1406 | 分别相加(或相减),其结果仍是一个等式,即如果 1407 | $A=B$,$C=D$,那么就有$A\pm C=B\pm D$.对方程也 1408 | 应适用. 1409 | 1410 | 根据这个道理,再观察这个方程组中的两个方程 1411 | 式,发现它们当中“同一个未知数$y$的系数是互为相 1412 | 反数”.因而,可以把两个方程式的两边分别相加, 1413 | 消去未知数$y$,得到一个只含有$x$的一元方程,从而 1414 | 就能解出$x$值,再进一步利用原方程组中的任一个方 1415 | 程,求出相应的$y$值.使问题得到解决. 1416 | \end{analyze} 1417 | 1418 | \begin{solution} 1419 | 将方程(2.15),(2.16)相加,(2.15)+(2.16)得: 1420 | \[4x=16 \qquad \therefore\quad x=4 \] 1421 | 把$x=4$代入(2.15): 1422 | \[4+y=12 \qquad \therefore\quad y=8\] 1423 | 经检验,原方程组的解是$(x,y)=(4,8)$,即:原方程组的解集是 1424 | \[\{(x,y)\}=\{4,8\}\] 1425 | \end{solution} 1426 | 1427 | 通过这个例题的分析与解法,试想一想:在什么情况下,两个方程相加能消元?又在什么情况下,两个方程相减能消元? 1428 | 1429 | 不难知道,只有在\textbf{两个方程式中,同一个末知数 1430 | 的两个系数绝对值相等}的情况下,才能\textbf{相加}或\textbf{相减达 1431 | 到消元}.而且,当某一未知数的\textbf{系数相同}时,可以\textbf{相 1432 | 减};当某一未知数的\textbf{系数互为相反数时},可以\textbf{相加}. 1433 | 1434 | 1435 | \begin{example} 1436 | 解方程组 1437 | \begin{numcases}{} 1438 | 2x+3y=8\\ 1439 | 3x+4y=11 1440 | \end{numcases} 1441 | \end{example} 1442 | 1443 | \begin{analyze} 1444 | 这个方程组中,未知数的系数没有什 1445 | 么特点,不具备“相加或相减”进行消元的条件.但 1446 | 是,我们可以应用等式的性质,设法创造条件,也就 1447 | 是设法使其中某一个未知数的两个系数绝对值相等以 1448 | 后,再去消元. 1449 | 1450 | 根据这个想法,就要用等式性质: 1451 | 1452 | 如果$A=B$, $C=D$,且有非零数$m, n$.那么,就 1453 | 有等式:$mA+nC= mB + nD$ 1454 | \end{analyze} 1455 | 1456 | \begin{solution} 1457 | 方程(2.17)两边同乘以3.(简写为:(2.17)$\x 3$),得: 1458 | \begin{equation} 1459 | 6x+9y=24 1460 | \end{equation} 1461 | (2.18)$\x 2$得: 1462 | \begin{equation} 1463 | 6x+8y=22 1464 | \end{equation} 1465 | (2.19)$-$(2.20)得:$y=2$ 1466 | 1467 | 把$y=2$代入(2.17)(或代入(2.18))得 1468 | \[2x+3\x 2=8\qquad \therefore\quad x=1\] 1469 | 经检验,原方程组的解是$(x,y)=(1,2)$,即:原方程组的解集是 1470 | \[\{(x,y)\}=\{(1,2)\} \] 1471 | \end{solution} 1472 | 1473 | 通过例2.17,例2.18的分析与解法,可以总结出这种 1474 | 解法的步骤与要点如下: 1475 | \begin{blk}{} 1476 | \begin{enumerate}[I. ] 1477 | \item 两方程中,若有“\textbf{同一未知数的两个系 1478 | 数绝对值相同}”的特点,可以\textbf{相加或相减}进行\textbf{消 1479 | 元};若没有上述特点,可以运用等式性质,先使 1480 | 原方程组变形为具有这一特点的形式,然后再把 1481 | 变形后的两方程\textbf{相加或相减},达到\textbf{消元}的目的. 1482 | \item 解所得的一元方程. 1483 | \item 把得到的一个未知数的值,代入原方程 1484 | 组中的任一个方程,求出另一个未知数相应的 1485 | 值. 1486 | \end{enumerate} 1487 | \end{blk} 1488 | 1489 | 这种求解的方法,叫做\textbf{加减消元法},简称\textbf{加减 1490 | 法}. 1491 | 1492 | \begin{ex} 1493 | 用加减消元法解下列方程组: 1494 | \begin{multicols}{2} 1495 | \begin{enumerate} 1496 | \item $\begin{cases} 1497 | x + y = 8 \\ x - y = 4 \\ 1498 | \end{cases}$ 1499 | \item $\begin{cases} 1500 | 3 x + 2y =21 \\ 2x + 2 y = 16 \\ 1501 | \end{cases}$ 1502 | \item $\begin{cases} 1503 | 2 x + \frac{y}{2} = 2 \\ \frac{y}{2} =x +\frac{1}{2} \\ 1504 | \end{cases}$ 1505 | \item $\begin{cases} 1506 | 7- 3 x =2 y \\ -9+ 7 y =5x \\ 1507 | \end{cases}$ 1508 | \item $\begin{cases} 1509 | 0.1=\frac{x}{4} -\frac{y}{2} \\ \frac{3x}{4}=\frac{1}{10}+\frac{5y}{2} \\ 1510 | \end{cases}$ 1511 | \item $\begin{cases} 1512 | 5m-4n = 33 \\3m+2n= 33 \\ 1513 | \end{cases}$ 1514 | \end{enumerate} 1515 | \end{multicols} 1516 | \end{ex} 1517 | 1518 | 归根结底,解二元一次方程组的关键就是消元, 1519 | 无论那种方法,在运用时都要注意:先由运算通性及 1520 | 等式性质把方程组的每个方程进行变形,整理成为一 1521 | 般形式: 1522 | \[\begin{cases} 1523 | a_1x+b_1y=c_1\\ 1524 | a_2x+b_2y=c_2 1525 | \end{cases} \text{ ($a_1,b_1,c_1,a_2,b_2,c_2$都是已知数)}\] 1526 | 1527 | 然后再根据特点,具体分析,灵活选用代入法或 1528 | 加减法进行\textbf{消元}.方法力求简捷、方便. 1529 | 1530 | \begin{example} 1531 | 解方程组: 1532 | \begin{multicols}{2} 1533 | \begin{enumerate} 1534 | \item $\begin{cases} 1535 | 4x+3y=7 \\ 1536 | 12x+9y=21 1537 | \end{cases}$ 1538 | \item $\begin{cases} 1539 | 4x+3y=7\\ 1540 | 8x+6y=11 1541 | \end{cases}$ 1542 | \end{enumerate} 1543 | \end{multicols} 1544 | \end{example} 1545 | 1546 | \begin{solution} 1547 | \begin{enumerate} 1548 | \item 1549 | \begin{numcases}{} 1550 | 4x+3y=7 \\ 1551 | 12x+9y=21 1552 | \end{numcases} 1553 | 如果把(2.21)式乘以3,得 1554 | \begin{equation} 1555 | 12x+9y=21 1556 | \end{equation} 1557 | (2.23)$-$(2.22)可得:$0=0$. 1558 | 1559 | 这个结果并不奇怪,仔细观察原方程组,就会发 1560 | 现,(2.22)式正是(2.21) $\x 3$的结果.这说明所给方程 1561 | 组,实质上是一个二元一次方程,因此,可以断定: 1562 | \textbf{原方程组有无数多个解}. 1563 | 1564 | \item \begin{numcases}{} 1565 | 4x+3y=7\\ 1566 | 8x+6y=11 1567 | \end{numcases} 1568 | (2.24)$\x 2$可得: 1569 | \begin{equation} 1570 | 8x+6y=14 1571 | \end{equation} 1572 | (2.25)$-$(2.26)可得:$0=-3$,这是不可能的. 1573 | 1574 | 因此,原方程组就是\textbf{矛盾方程组,无解}. 1575 | \end{enumerate} 1576 | \end{solution} 1577 | 1578 | 通过这一例题,你能不能发现:方程组如果有无 1579 | 数多解或无解,它们两个方程的各项系数之间应该具 1580 | 有什么规律和特点? 1581 | 1582 | \begin{ex} 1583 | 解下列方程组: 1584 | \begin{multicols}{2} 1585 | \begin{enumerate} 1586 | \item $\begin{cases} 1587 | \frac{x}{3}-\frac{y}{7}=1\\ 1588 | 1\frac{2}{3}x-\frac{5}{7}y=5 1589 | \end{cases}$ 1590 | \item $\begin{cases} 1591 | x=9-7y\\ 1592 | 3x+y=19-20y 1593 | \end{cases}$ 1594 | \end{enumerate} 1595 | \end{multicols} 1596 | \end{ex} 1597 | 1598 | 学习了二元一次方程组的解法以后,在解决实际 1599 | 问题时,就可以引入两个未知数,根据题目中的条 1600 | 件,列出两个方程式,组成方程组.进而解出方程组, 1601 | 使问题得到解决. 1602 | 1603 | \begin{example} 1604 | 兄弟二人的语文考试成绩共190分,如果 1605 | 知道兄是弟的成绩的90,那么,兄、弟各得多少 1606 | 分? 1607 | \end{example} 1608 | 1609 | \begin{solution} 1610 | 设弟弟得$x$分,兄得$y$分,则由题目所给条件可以得出: 1611 | \begin{numcases}{} 1612 | x+y=190\\ 1613 | y=\frac{90}{100}x 1614 | \end{numcases} 1615 | 将(2.28)代入(2.27):$x+\frac{90}{100}x=190$ 1616 | 解得 \qquad $x=100$ 1617 | 1618 | 把$x=100$代入(2.28):$y=90$ 1619 | 1620 | 答:兄得90分,弟得100分. 1621 | \end{solution} 1622 | 1623 | \begin{ex} 1624 | 引入两个未知数,列方程组解: 1625 | \begin{enumerate} 1626 | \item 某两数之和为100,之差为50.求两数. 1627 | \item 班上男、女同学共52人,而女生人数的一半比男生总数少 1628 | 4人.求男、女生各几人? 1629 | \end{enumerate} 1630 | \end{ex} 1631 | 1632 | \subsection{三元一次方程组及其解法} 1633 | \textbf{含有三个未知数的一次方程组},叫做\textbf{三元一次方 1634 | 程组}.例如: 1635 | 1636 | 方程组 1637 | \[\begin{cases} 1638 | 2x+3y+z=38\\3x+4y+2z=56\\4x+5y+z=66 1639 | \end{cases}\quad \begin{cases} 1640 | x+y+z=2\\x-y+z=0\\x\qquad -z=4 1641 | \end{cases}\quad \begin{cases} 1642 | 2m-R+2n=8\\ \qquad\;\; R+2n=-2\\ 3m+R-4n=1 1643 | \end{cases} \] 1644 | 等等,都是三元一次方程组. 1645 | 1646 | 解三元一次方程组的关键,仍然是消元,其具体 1647 | 方法仍是代入法和加减法.通过逐步消元,将三元转 1648 | 化为二元,再转化为一元.每一次转化,都是运用 1649 | “通性”和等式性质.以保证解出一元方程的解,再 1650 | 逐步求得三元一次方程组的解. 1651 | 1652 | \begin{example} 1653 | 解方程组 1654 | \begin{numcases}{} 1655 | 2x+3y+z=38\\ 1656 | 3x+4y+2z=56\\ 1657 | 4x+5y+z=66 1658 | \end{numcases} 1659 | \end{example} 1660 | 1661 | \begin{analyze} 1662 | 方程组中,未知数$z$的各项系数较简单, 1663 | 所以,可考虑先消去$z$,转化为二元一次方程组求解. 1664 | \end{analyze} 1665 | 1666 | \begin{solution} 1667 | (2.31)$-$(2.29):$2x+2y=28$,即: 1668 | \begin{equation} 1669 | x+y=14 1670 | \end{equation} 1671 | (2.29)$\x 2-$(2.30): 1672 | \begin{equation} 1673 | x+2y=20 1674 | \end{equation} 1675 | 将(2.32)、(2.33)联立,由(2.33)$-$(2.32):$y=6$ 1676 | 1677 | 将$y=6$代入(2.32):解出$x=8$. 1678 | 1679 | 把$x=8,\quad y=6$代入(2.29):解出$z=4$. 1680 | 1681 | 所以,原方程的解是:$(x,y,z)=(8,6,4)$ 1682 | \end{solution} 1683 | 1684 | \begin{rmk} 1685 | 三元一次方程组的一个解,也是一个 1686 | \textbf{数值组},它由三个有顺序的数组成,不能忽视$(x, y, z) 1687 | =( 8, 6, 4 )$的写法.如果写成解集的形式就是: 1688 | \[\{(x,y,z)\}=\{(8,6,4)\} \] 1689 | \end{rmk} 1690 | 1691 | \begin{example} 1692 | 解方程组: 1693 | \begin{numcases}{} 1694 | x+y+z=2\\ 1695 | x-y+z=0\\ 1696 | x\qquad -z=4 1697 | \end{numcases} 1698 | \end{example} 1699 | 1700 | \begin{analyze} 1701 | 这个方程组的特点是:各个未知数的 1702 | 系数绝对值在每个方程中都是1.因而,直接运用加 1703 | 减法消元比较方便. 1704 | \end{analyze} 1705 | 1706 | 1707 | \begin{solution} 1708 | (2.34)$-$(2.35):$2y=2$,$\therefore\quad y=1$ 1709 | 1710 | (2.34)$+$(2.36): 1711 | \begin{equation} 1712 | 2x+y=6 1713 | \end{equation} 1714 | 1715 | 把$y=1$代入(2.37):$2x+1=6$,$\therefore\quad x=\frac{5}{2}$ 1716 | 1717 | 把$x=\frac{5}{2}$代入(2.36):$\frac{5}{2}-z=4$,$\therefore\quad z=-\frac{3}{2}$ 1718 | 1719 | 所以,原方程组的解为:$(x,y,z)=\left(\frac{5}{2},1,-\frac{3}{2}\right)$,即解集为: 1720 | $$\{(x,y,z)\}=\left\{\left(\frac{5}{2},1,-\frac{3}{2}\right)\right\}$$ 1721 | \end{solution} 1722 | 1723 | \begin{example} 1724 | 解方程组: 1725 | \begin{numcases}{} 1726 | x+y\qquad =3\\ 1727 | \qquad y+z=5\\ 1728 | x\qquad +z=4 1729 | \end{numcases} 1730 | \end{example} 1731 | 1732 | 1733 | \begin{analyze} 1734 | 这个方程组的特点是:每一个方程中 1735 | 都缺少一个未知数,且系数都是1,分布很均匀.用 1736 | 消元法可以灵活处理. 1737 | \end{analyze} 1738 | 1739 | \begin{solution} 1740 | (2.38)$+$(2.39)$+$(2.40):$2x+2y+2z=12$,即: 1741 | \begin{equation} 1742 | x+y+z=6 1743 | \end{equation} 1744 | 应用(2.41)$-$(2.39),(2.41)$-$(2.40),(2.41)$-$(2.38)就可立即得出: 1745 | \[x=1,\quad y=2,\quad z=3 \] 1746 | 1747 | 所以,原方程组的解为:$(x,y,z)=(1,2,3)$,即解集为: 1748 | \[\{(x,y,z)\}=\{(1,2,3)\} \] 1749 | \end{solution} 1750 | 1751 | 引入三个未知数,列出三个一次方程,也可以解 1752 | 一些实际问题. 1753 | \begin{example} 1754 | 某中学应届毕业生总共有$a$人,其中升入 1755 | 高校和升入中专的占总人数的60\%;参加工作的正好 1756 | 与升入高校的人数一样多.试问:升入高校、升入中 1757 | 专和参加工作的各占多少人? 1758 | \end{example} 1759 | 1760 | \begin{solution} 1761 | 设升入高校$x$人,升入中专$y$人,参加工作 1762 | 的$z$人,则由题目所给条件,可以列出: 1763 | \begin{numcases}{} 1764 | x+y+z=a\\ 1765 | x+y=\frac{60}{100}a\\ 1766 | x=z 1767 | \end{numcases} 1768 | 1769 | 解这个方程组,(2.42)$-$(2.43):$z=a-\frac{60}{100}a$,$\therefore\quad z=\frac{2}{5}a$ 1770 | 1771 | 把$z=\frac{2}{5}a$代入(2.44):$\therefore\quad x=\frac{2}{5}a$ 1772 | 1773 | 把$x=z=\frac{2}{5}a$代入(2.42):$y=a-\frac{2}{5}a-\frac{2}{5}a$,$\therefore\quad y=\frac{1}{5}a$. 1774 | 1775 | 因此:这个班有$\frac{2}{5}a$人升入高校,有$\frac{1}{5}a$人升入中专,有$\frac{2}{5}a$人参加了工作. 1776 | \end{solution} 1777 | 1778 | \begin{ex} 1779 | 解下列三元一次方程组 1780 | \begin{multicols}{2} 1781 | \begin{enumerate} 1782 | \item $\begin{cases} 1783 | x+y-z=3\\-x+y+z=6\\x-y+z=4 1784 | \end{cases}$ 1785 | \item $\begin{cases} 1786 | x+2y+3z=6\\ 2x+3y+z=6\\3x+y+2z=6 1787 | \end{cases}$ 1788 | \item $\begin{cases} 1789 | 2x+y=2\\2y+z=7\\x+2z=3 1790 | \end{cases}$ 1791 | \item $\begin{cases} 1792 | x+y+z=3\\5x-y+z=9\\x-2y+7z=24 1793 | \end{cases}$ 1794 | \end{enumerate} 1795 | \end{multicols} 1796 | \end{ex} 1797 | 1798 | \section*{习题2.3} 1799 | \addcontentsline{toc}{subsection}{习题2.3} 1800 | \begin{enumerate} 1801 | \item 已知方程$3x-y=1$,试先用$x$的算式表示出$y$来,再按下 1802 | 表给出的$x$值求出$y$的相应的每一个值: 1803 | \begin{center} 1804 | \begin{tabular}{c|ccccccccc} 1805 | $x$ & $\cdots$ &$-3$&$-1$&0&$\frac{1}{2}$&$\frac{1}{3}$&1&2& $\cdots$\\ 1806 | \hline 1807 | $y$ & $\cdots$ &&&&&&&& $\cdots$\\ 1808 | \end{tabular} 1809 | \end{center} 1810 | \item 试求不定方程$3x+y=10$的正整数解. 1811 | \item 如果有10吨货物,要用马车和汽车一次运走,马车三辆 1812 | 可装一吨,汽车一辆能装3吨.试问:如何派车?有几种方 1813 | 案? 1814 | \item 判断下边方括号中给出的数值组$(x, y)$,是不是以下方程 1815 | 组的解? 1816 | \[\begin{cases} 1817 | 2x-y=7\\x+2y=-4 1818 | \end{cases}\] 1819 | \[[(-1,9),\quad (1,-5),\quad (0,-2),\quad (2,-3),\quad (3,-1),\quad (-2,-1)] \] 1820 | 1821 | \item 已知不定方程$my-2x=7$的一个解是:$(x,y)=(1,3)$, 1822 | 试求$m$的值,并求出有一个未知数取0时,这个不定方程的 1823 | 两个解. 1824 | \item 用代入法解下列二元一次方程组: 1825 | \begin{multicols}{2} 1826 | \begin{enumerate} 1827 | \item $\begin{cases} 1828 | 7x+5y=6\\ y=x+3 1829 | \end{cases}$ 1830 | \item $\begin{cases} 1831 | a+2b=0\\ 3a+4b=6 1832 | \end{cases}$ 1833 | \item $\begin{cases} 1834 | 7y-3=-11x\\ y=2x 1835 | \end{cases}$ 1836 | \item $\begin{cases} 1837 | x=\frac{5}{2}y\\ 3x-4y=-14 1838 | \end{cases}$ 1839 | \item $\begin{cases} 1840 | x+2y-1=0\\ x-4y+22=0 1841 | \end{cases}$ 1842 | \item $\begin{cases} 1843 | 3x=\frac{y+1}{2}\\ 3x=\frac{y}{4} 1844 | \end{cases}$ 1845 | \item $\begin{cases} 1846 | \frac{1}{2}y-\frac{x}{4}=-\frac{5}{4}\\y=3(x-2) 1847 | \end{cases}$ 1848 | \item $\begin{cases} 1849 | 3x-2y=5\\ x+4y=18 1850 | \end{cases}$ 1851 | \item $\begin{cases} 1852 | m+2n=4\\5m-1=4 1853 | \end{cases}$ 1854 | \item $2(a+b)=3b+\frac{1}{2}=3a+b$ 1855 | \end{enumerate} 1856 | \end{multicols} 1857 | 1858 | \item 用加减法解下列方程组: 1859 | \begin{multicols}{2} 1860 | \begin{enumerate} 1861 | \item $\begin{cases} 1862 | 3x-2y=5\\ x+2y=11 1863 | \end{cases}$ 1864 | \item $\begin{cases} 1865 | 2x+3y=11\\ y-2x=1 1866 | \end{cases}$ 1867 | \item $\begin{cases} 1868 | 5x+3y=7\\-5x+7y=3 1869 | \end{cases}$ 1870 | \item $\begin{cases} 1871 | 2x+5y=25\\4x+3y=15 1872 | \end{cases}$ 1873 | \item $\begin{cases} 1874 | 25x+17y=40\\17x+25y=40 1875 | \end{cases}$ 1876 | \item $\begin{cases} 1877 | 5x=1+3y\\3x+7y=27 1878 | \end{cases}$ 1879 | \item $\begin{cases} 1880 | 3x+5y=19\\ 8x-3y=67 1881 | \end{cases}$ 1882 | \item $\begin{cases} 1883 | 7x-3y=10\\ 3\frac{1}{2}x-50=6y 1884 | \end{cases}$ 1885 | \item $\begin{cases} 1886 | 6.2v-4s=0.2\\ 8v-5s=1 1887 | \end{cases}$ 1888 | \item $\begin{cases} 1889 | 3(x-1)=4(y-4)\\5(y-1)=3(x+5) 1890 | \end{cases}$ 1891 | \item $\begin{cases} 1892 | 2(x+1)-3(y-1)=10\\ 2(x+1)+7(y-1)=20 1893 | \end{cases}$ 1894 | \item $\begin{cases} 1895 | \frac{y}{2}+\frac{z}{3}=13\\ \frac{y}{3}-\frac{z}{4}=3 1896 | \end{cases}$ 1897 | \item $\begin{cases} 1898 | \frac{m+n}{3}-\frac{m-n}{4}=+0.35\\ \frac{m+n}{3}+\frac{m-n}{2}=0.5 1899 | \end{cases}$ 1900 | \item $0.1x-2=y+7=0.7x+y$ 1901 | \item $2(t-1)-s=t-1=5s$ 1902 | \end{enumerate} 1903 | \end{multicols} 1904 | 1905 | \item 解下列方程组:(任选方法) 1906 | \begin{multicols}{2} 1907 | \begin{enumerate} 1908 | \item $\begin{cases} 1909 | x=16-4y\\ y=34-4x 1910 | \end{cases}$ 1911 | \item $\begin{cases} 1912 | x+y=7\\x:5=y:2 1913 | \end{cases}$ 1914 | \item $\begin{cases} 1915 | \frac{2(x-0.5)}{5}-\frac{3(y+2)}{4}=1\\ 1916 | 3(2x-1)+15(y+2)=5 1917 | \end{cases}$ 1918 | \item $\begin{cases} 1919 | \frac{v}{3}-\frac{t}{5}=\frac{2}{15}\\ 1920 | \frac{2v}{3}+\frac{t}{5}=\frac{13}{15} 1921 | \end{cases}$ 1922 | \item $\begin{cases} 1923 | 3x-2y=7\\ 6x-4y=14 1924 | \end{cases}$ 1925 | \item $\begin{cases} 1926 | x+2y=-1\\ x+y=\frac{1}{2}x+3 1927 | \end{cases}$ 1928 | \end{enumerate} 1929 | \end{multicols} 1930 | 1931 | \item 如果已知方程组 1932 | \[\begin{cases} 1933 | (m+1)x-(n-3)y =-11\\ 1934 | mx+(n+2)y=7 1935 | \end{cases}\] 1936 | 的一个解是$(x,y)=(1,-2)$,试求$m,n$的值. 1937 | 1938 | \item 如果遇到三个方程式组成的二元一次方程组时,可以 1939 | 先就其中任两个方程求出解答,再将解答代入第三个方程中 1940 | 去检验.假如所求解答也满足第三个方程,那么,这个解就 1941 | 是原方程组的解;假如不满足,那么,原方程组就无解.试 1942 | 用这种方法解下列方程组: 1943 | \begin{multicols}{2} 1944 | \begin{enumerate} 1945 | \item $\begin{cases} 1946 | 4x-3y=1\\4x+y=5\\x-y=0 1947 | \end{cases}$ 1948 | \item $\begin{cases} 1949 | 4(x+y)+3=y+2\\ 7x-10=1-y\\ x+y=-1 1950 | \end{cases}$ 1951 | \item $\begin{cases} 1952 | 3(2x+y)-1=2\\ x+1=2\\ y-3=2 1953 | \end{cases}$ 1954 | \end{enumerate} 1955 | \end{multicols} 1956 | 1957 | \item 解下列三元一次方程组: 1958 | \begin{enumerate} 1959 | \item $\begin{cases} 1960 | x+y+2z=2\\ 3x-y-4z=5\\ 2x+3y-2z=0 1961 | \end{cases}$ 1962 | \item $\begin{cases} 1963 | x-y=\frac{3}{2}\\x+z=\frac{7}{10}\\y-z=-\frac{6}{5} 1964 | \end{cases}$ 1965 | \item $\begin{cases} 1966 | x+y=x+2y-3=z\\ 2x-3y+2z=5 1967 | \end{cases}$ 1968 | \item $ 1969 | x+y+z=y-8x+107=x-2(x+y)+84=108$ 1970 | \end{enumerate} 1971 | 1972 | \item 把下列方程组中的字母$t$当作已知数,试解出下列方 1973 | 程组: 1974 | \begin{multicols}{2} 1975 | \begin{enumerate} 1976 | \item $\begin{cases} 1977 | x+y+t=3\\ x-y+3t=-1 1978 | \end{cases}$ 1979 | \item $\begin{cases} 1980 | 2x+3y=1+t\\ y=x-t 1981 | \end{cases}$ 1982 | \end{enumerate} 1983 | \end{multicols} 1984 | 1985 | \item 如果$a,b$都是已知数,并且$|a|\ne |b|$,试解下面的字 1986 | 母系数方程组: 1987 | \[\begin{cases} 1988 | ax+by=b\\ bx+ay=a 1989 | \end{cases} \] 1990 | \end{enumerate} 1991 | 1992 | 1993 | \section{解应用问题} 1994 | 前面已经知道,用代数方法解决实际应用问题的 1995 | 时候,先要做好充分准备,打好基础,就是先要引入 1996 | 适当的未知数,根据题目中的数量关系直截了当地列 1997 | 出方程或方程组;然后再去运用由数系运算通性及等 1998 | 式性质而得出来的系统解法,求出符合题意的解答. 1999 | 2000 | 本节将集中讨论如何灵活地解决各种一次方程 2001 | (或方程组)的应用问题. 2002 | 2003 | 2004 | \begin{example} 2005 | 小明买练习本、生字本共15本,总共用去 2006 | 一元四角钱.如果练习本的单价是一角六分,生字本 2007 | 的单价是六分.试问:小明买练习本、生字本各多少 2008 | 本? 2009 | \end{example} 2010 | 2011 | \begin{analyze} 2012 | 方程就是一种等式.要列出符合题意 2013 | 的方程,就要先明确问题中的\textbf{等量关系}.这个问题 2014 | 中,有以下两个基本的等量关系: 2015 | \begin{enumerate} 2016 | \item 总共15本,即 \[(\text{练习本数})+(\text{生字本数})=15 \] 2017 | \item 总共用钱1.40元,即 2018 | \[(\text{练习本数})\x(\text{单价})+(\text{生字本数})\x(\text{单价})=1.40\] 2019 | \end{enumerate} 2020 | 因此,可以有两种考虑方法: 2021 | 2022 | \textbf{解法1:} 引入一个未知数 2023 | 2024 | 设小明买练习本$x$本,则利用关系1可知,买生字本$(15-x)$本, 2025 | 再利用关系2就可列出方程: 2026 | \[x\cdot (0.16)+(15-x)\cdot(0.06)=1.4\] 2027 | 即: 2028 | \[\begin{split} 2029 | 0.16x+0.9-0.06x &= 1.4\\ 2030 | 0.1x&=0.5\\ 2031 | x&=5\text{(本)} 2032 | \end{split}\] 2033 | \[15-x=15-5=10\text{(本)}\] 2034 | 2035 | 答: 小明买了练习本5本,生字本10本. 2036 | 2037 | 2038 | \textbf{解法2:} 引入两个未知数 2039 | 2040 | 设小明买练习本$x$本,生字本$y$本. 2041 | 则由关系式1与2,可以分别得出: 2042 | \begin{numcases}{} 2043 | x+y=15\\ 2044 | x\cdot 0.16+y\cdot 0.06=1.4 2045 | \end{numcases} 2046 | 解这个二元一次方程组,就可得出: 2047 | $(x,y)=(5,10)$ 2048 | 2049 | 答: 小明买了练习本5本,生字本10本. 2050 | \end{analyze} 2051 | 2052 | 由此可见,同一个问题,有时可以引入一个未知 2053 | 数,列出方程求解;也可以引入两个未知数,列出方 2054 | 程组求解.两种解法各有利弊:一元方程求解简便, 2055 | 但列方程较难;二元方程组列方程较易,但求解稍 2056 | 繁.因而,在应用中可以灵活选择,不必要求一律. 2057 | 一般来说,要选择“便于求解、引入未知数较少”的 2058 | 方法. 2059 | 2060 | \begin{ex} 2061 | 引入恰当的未知数,解下列各题: 2062 | \begin{enumerate} 2063 | \item 鸡兔同笼,共有头12个,有脚36只.问:笼中有鸡、兔各 2064 | 几只? 2065 | \item 若干学生平分若干支铅笔:如果每人5支,最后还多余3 2066 | 支,如果每人7支,最后又缺5支.试问:有多少学生? 2067 | 有多少支铅笔? 2068 | \end{enumerate} 2069 | \end{ex} 2070 | 2071 | \begin{example} 2072 | 兄弟二人,从他们的家出发走同一条路线, 2073 | 前往天安门广场.哥哥平均每小时走5公里,弟弟平 2074 | 均每小时走3公里,假如哥哥比弟弟晚出发一小时, 2075 | 却早到12分钟.试问:他们家到天安门广场有多远? 2076 | \end{example} 2077 | 2078 | \begin{analyze} 2079 | 这是一个“行程问题”,涉及到的等 2080 | 量关系是: 2081 | \[\text{路程}=\text{速度}\x\text{时间}\] 2082 | 也可以写成:另外两种形式的关系: 2083 | \[\begin{split} 2084 | \text{路程}\div \text{时间}&=\text{速度}\\ 2085 | \text{路程}\div \text{速度}&=\text{时间} 2086 | \end{split}\] 2087 | 题目中,不仅已知兄弟二人的速度,还已知兄弟二人走 2088 | 完这段路程的时间差.因此,只要选择适当的未知数, 2089 | 不难由基本等量关系列出方程式来,把问题解决的. 2090 | \end{analyze} 2091 | 2092 | \textbf{解法1:} 引入直接未知数 2093 | 2094 | 设他们的家到天安门广场的路程为$x$公里,由 2095 | 关系式$\text{时间}=\text{路程}\div \text{速度}$,就可以得出: 2096 | 哥哥走到天安门广场所用时间为$\frac{x}{5}$小时; 2097 | 弟弟走到天安门广场所用时间为$\frac{x}{3}$小时. 2098 | 2099 | 又由题意可知:哥比弟晚出发12分钟,这就是说:哥比弟少用$1\frac{12}{60}$小时,因此应有等量关系: 2100 | \[\text{哥所用时间}+1\frac{12}{60}=\text{弟所用时间}\] 2101 | 即: 2102 | \[\begin{split} 2103 | \frac{x}{5}+1\frac{12}{60}&=\frac{x}{3}\\ 2104 | \frac{x}{5}+\frac{6}{5}&=\frac{x}{3}\\ 2105 | 2x&=18\\ 2106 | x&=9\text{公里} 2107 | \end{split}\] 2108 | 答:他们家到天安门广场的距离为9公里. 2109 | 2110 | \textbf{解法2:} 引入间接未知数 2111 | 2112 | 设弟弟到天安门广场总共用了$t$小时,则由题意可知: 2113 | 哥哥用了$\left(t-1\frac{1}{5}\right)$小时.因此,由基本关系式 2114 | $\text{路程}=\text{速度}\x\text{时间}$ 可以知道: 2115 | \[\begin{split} 2116 | \text{哥哥所走路程} &=5\x \left(t-1\frac{1}{5}\right) \text{(公里)}\\ 2117 | \text{弟弟所走路程}&=3\x t \text{(公里)} 2118 | \end{split}\] 2119 | 2120 | 又知,哥、弟二人走的是同一路线,所以: 2121 | \[\begin{split} 2122 | 5\left(t-1\frac{1}{5}\right)&=3t\\ 2123 | 2t&=6\\ 2124 | t&=3\text{(小时)} 2125 | \end{split}\] 2126 | 2127 | 这就是说:弟弟用了三小时走到天安门广场. 2128 | 2129 | 由此,再利用基本关系式可以得到:他们家到天 2130 | 安门广场的路程为:$3\x3= 9$(公里) 2131 | 2132 | 答: 所求路程是9公里. 2133 | 2134 | 从这里可以看出,在解决应用问题时,对于同一 2135 | 个间题,有时可以从不同的角度引进未知数(可以 2136 | 直接设所求的量为未知数,也可以间接设一个与所求 2137 | 量有关的未知数).由于引用未知数的不同,同一个 2138 | 问题完全可以列出不同形式的方程.但最终所求的量 2139 | 的答数是一样的.这真是“异途同归”了. 2140 | 2141 | 2142 | \begin{ex} 2143 | 解下列应用题: 2144 | \begin{enumerate} 2145 | \item 甲、乙两地相距165里;小张以每小时30里的速度骑自行 2146 | 车由甲地去乙地,小李以每小时25里的速度骑自行车由乙 2147 | 地去甲地. 2148 | 2149 | 问:他们两人如果同时出发,几小时后相遇? 2150 | 在距甲地多少里的地方相遇? 2151 | \item 甲、乙两个车站相距284公里,甲站有一列快车以每小时70 2152 | 公里的速度要开往乙站;而乙站有一列慢车以每小时48公 2153 | 里的速度要开往甲站.如果慢车先开车一小时. 2154 | 2155 | 试问:快车开 2156 | 几小时后两车才能相遇?在距离甲站多少公里的地方相遇? 2157 | \end{enumerate} 2158 | \end{ex} 2159 | 2160 | 通过以上例题,可以归纳出解应用问题的一般步 2161 | 骤是: 2162 | 2163 | \begin{blk}{} 2164 | \begin{enumerate}[I. ] 2165 | \item 审清题意,明确数量关系. 2166 | \item 引入适当的未知数$x$.并注意写明未知 2167 | 数所代表的量的单位.如:$x$公里,$x$小时等. 2168 | \item 用含有未知数的算式,表示其它有关的 2169 | 量数. 2170 | \item 由基本关系式,列出方程或方程组(一 2171 | 般说:列出的方程个数与所设未知数的个数是相 2172 | 等的). 2173 | \item 解方程(或方程组). 2174 | \item 检验所求解是否符合题意后,写出答 2175 | 案. 2176 | \end{enumerate} 2177 | \end{blk} 2178 | 2179 | 必须指出:应用题的检验是必要的,一方面可以 2180 | 检查计算是否正确,另一方面还可以检验所求方程的 2181 | 解是否符合题意,不符合题意的要舍去. 2182 | 2183 | \begin{example} 2184 | 两地相距28公里,小明以每小时15公里的 2185 | 速度,小亮以每小时30公里的速度,分别骑自行车和 2186 | 开汽车从同一地前往另一地.如果小明先出发一小 2187 | 时,试问:小亮几小时以后,才能乘汽车追上小明? 2188 | \end{example} 2189 | 2190 | \begin{solution} 2191 | 设小亮开车$x$小时能追上小明, 2192 | 则小亮所行路程是:$30x$(公里). 2193 | 2194 | 这时,先出发一小时的小明,已经走过的路程 2195 | 为:$15(x+1)=15x + 15$(公里) 2196 | 2197 | 小亮要追上小明,必须有关系式: 2198 | $ 30x=15x+15$ 2199 | 2200 | $\therefore\quad x= 1$(小时) 2201 | 2202 | 这就是说:小亮开车一小时,就能追上小明. 2203 | 2204 | \textbf{检验:} $x=1$,虽然能使方程$30x =15x + 15$ 2205 | 成立,但这时,小亮开车走出30公里的路程,而两地 2206 | 实有距离只有28公里,由此可见,在题目所给出的两 2207 | 地之间,小亮是没有追上小明的.$x=1$不符合题意, 2208 | 应舍去. 2209 | 2210 | 答:在此两地之间,小亮追不上小明. 2211 | \end{solution} 2212 | 2213 | 2214 | \begin{ex} 2215 | 解下列“追及问题” 2216 | \begin{enumerate} 2217 | \item 甲、乙两人从同地出发练习短距离赛跑,甲每秒跑7米, 2218 | 乙每秒跑6.5米.如果甲让乙先跑一秒钟,试问:甲经过 2219 | 几秒钟才能追上乙? 2220 | \item 甲、乙两人同时从同地出发,绕400米一圈的跑道赛跑, 2221 | 如果甲跑一圈要一分钟,乙跑一圈要一分零20秒,试问: 2222 | 甲、乙出发几分钟以后,才能再相遇? 2223 | \end{enumerate} 2224 | \end{ex} 2225 | 2226 | 2227 | 2228 | \begin{example} 2229 | 一只船在两个码头之间航行.顺水时需要 2230 | 4.5小时,逆水返回需要$5\frac{1}{3}$小时.如果水流速度是每 2231 | 小时1公里.试问:这两个码头相距多少公里? 2232 | \end{example} 2233 | 2234 | \begin{analyze} 2235 | 这种应用问题,实际上仍是“行程问 2236 | 题”.如果能求出这只船在顺水时的速度或在逆水中 2237 | 的速度,那么,两码头的距离就立即可以求出来,题 2238 | 目中只有水流速度.但只要注意:船在静水中的速度 2239 | 与水流速度、逆水速度、顺水速度有以下基本关系, 2240 | 问题就容易解决了. 2241 | \[\begin{split} 2242 | \text{顺水速度}&=\text{船在静水中的速度}+\text{水速}\\ 2243 | \text{逆水速度}&=\text{船在静水中的速度}-\text{水速}\\ 2244 | \end{split}\] 2245 | \end{analyze} 2246 | 2247 | \begin{solution} 2248 | 设船在静水中的速度是$x$公里/小时. 2249 | 则顺水速度为$(x+1)$公里/小时, 2250 | 2251 | 因此,船在顺水中,行驶的距离为: 2252 | $(x+1) \x 4.5$ (公里). 2253 | 2254 | 又逆水速度为$(x-1)$公里/小时,因此船在逆水返回时,所行驶的路程为: 2255 | $(x-1)\x5\frac{1}{3} $(公里). 2256 | 2257 | 由于顺水、逆水都是航行于两码头之间,因此: 2258 | \[(x+1)\x 4.5=(x-1)\x 5\frac{1}{3}\] 2259 | 解这个方程: 2260 | \[\begin{split} 2261 | 27 (x+1)&=32 (x-1)\\ 2262 | (32-27)x&=27+32\\ 2263 | 5x&=59\\ 2264 | x&=11\frac{4}{5}\text{(公里/小时)} 2265 | \end{split}\] 2266 | $\therefore\quad 4.5 (x+1) =4.5(11.8+1)=57.6$(公里) 2267 | 2268 | 答:两码头之间相距57.6公里. 2269 | \end{solution} 2270 | 2271 | \begin{ex} 2272 | \begin{enumerate} 2273 | \item 汽船从甲地顺流开往乙地,所用的时间比逆流由乙地开回 2274 | 甲地所用时间少1.5小时,扣果这只汽船在静水中速度是 2275 | 20公里/小时,水流速度是3公里/小时.试问:甲、乙两 2276 | 地相距多少? 2277 | \item $A, B$两码头相距若千公里,某船由$A$码头顺水行至$B$码头 2278 | 要3小时,回来时多用半小时,如果知道船在静水中的速 2279 | 度是26公里/小时,试求水流的速度? 2280 | \end{enumerate} 2281 | \end{ex} 2282 | 2283 | \begin{example} 2284 | 有一水池,用两台水泵抽水.如果单开甲 2285 | 泵,5小时抽完这一池水;如果单开乙泵,2.5小时 2286 | 才能抽完.试问: 2287 | \begin{enumerate} 2288 | \item 两泵同时开,几小时能把水抽完? 2289 | \item 如果甲泵先抽2小时,剩下的再单用乙泵来 2290 | 抽,还需要多少时间才能抽完? 2291 | \end{enumerate} 2292 | \end{example} 2293 | 2294 | \begin{analyze} 2295 | 这是应用问题中常见的“工程问题”. 2296 | 常把整个“工程的量”看作“1”进行分析. 2297 | 2298 | 在这个问题中,如果把“一池水”的总量看作整 2299 | 体“1”,那么,由题意可知:甲泵一小时的抽水量 2300 | 为$\frac{1}{5}$;乙泵每小时的抽水量是$\frac{1}{2.5}$;因此甲、乙两 2301 | 泵一齐开,每小时的抽水量就是$\left(\frac{1}{5}+\frac{1}{2.5}\right)$.这样, 2302 | 再利用基本关系: 2303 | \[\text{每小时的工作量} \x \text{工作时间}=\text{总工作量}1\] 2304 | 就可以解决问题了. 2305 | 2306 | \end{analyze} 2307 | 2308 | \begin{solution} 2309 | \begin{enumerate} 2310 | \item 设两泵同时开,$x$小时可以抽完这池水,因此就有: 2311 | \[\begin{split} 2312 | \left(\frac{1}{5}+\frac{1}{2.5}\right)x&=1\\ 2313 | \frac{3}{5}x&=1\\ 2314 | x&=\frac{5}{3} \text{(小时)} 2315 | \end{split}\] 2316 | 这就是说,两泵合开,$1\frac{2}{3}$时(1小时40分钟) 2317 | 可以抽完这一池水. 2318 | 2319 | 经检验,符合题意. 2320 | 2321 | 答:两泵合并,1小时40分钟可以抽完. 2322 | 2323 | \item 设乙泵再抽$x$小时将水抽完, 2324 | 因此,依题意就有: 2325 | \[\begin{split} 2326 | \frac{1}{5}\x 2+\frac{1}{2.5}\x x&=1\\ 2327 | \frac{1}{2.5}x&=\frac{3}{5}\\ 2328 | x&=1.5 \text{(小时)} 2329 | \end{split}\] 2330 | 2331 | 答:甲泵抽2小时后,乙泵再抽1.5小时,就可以抽完这一池水. 2332 | \end{enumerate} 2333 | \end{solution} 2334 | 2335 | 2336 | 2337 | 2338 | \begin{ex} 2339 | \begin{enumerate} 2340 | \item 检修一台机器,甲班需要7.5小时完成,乙班需要5小时 2341 | 完成.试问: 2342 | \begin{enumerate} 2343 | \item 两班同时参加检修,要几小时才能完成? 2344 | \item 如果甲班先做4小时,其余的工作由乙班完成, 2345 | 还要多少时间才能完成? 2346 | \end{enumerate} 2347 | 2348 | \item 某工程,甲队需要20天完成,乙队需要30天完成,丙队需 2349 | 要40天完成;如果甲、乙、丙三队联合工作8天以后,其 2350 | 余工程由甲队单独完成.试问:甲队还需要几天完成? 2351 | \end{enumerate} 2352 | \end{ex} 2353 | 2354 | \begin{example} 2355 | 浓度为95\%的盐水有600克,问: 2356 | \begin{enumerate} 2357 | \item 只要再加多少水,就能稀释成浓度为75\%的盐水? 2358 | \item 只要再加多少盐,就能得到浓度是98\%的盐水? 2359 | \end{enumerate} 2360 | \end{example} 2361 | 2362 | 2363 | \begin{analyze} 2364 | 这是应用题中的“浓度问题”(即百 2365 | 分数问题).只要明确基本关系: 2366 | \begin{enumerate} 2367 | \item $\text{浓度(百分比)}=\frac{\text{溶质}}{\text{溶质}+\text{溶剂}}$ 2368 | \item 增加溶剂,可使浓度减小,但其中的溶质总量不变. 2369 | \item 增加溶质,可使浓度加大,但其中的溶剂总量不变. 2370 | \end{enumerate} 2371 | 这一类应用问题是不难解决的. 2372 | \end{analyze} 2373 | 2374 | \begin{solution} 2375 | \begin{enumerate} 2376 | \item 设加水(溶剂)$x$克, 2377 | 则加水前,95\%的盐水600克中,含有纯盐(溶 2378 | 质)的总量是: $600 \x 95\%$克 2379 | 2380 | 加水后,75\%的盐水$(600+x)$克中,含有纯盐的 2381 | 总量是: $(600+x) \x 75\%$克. 2382 | 2383 | 因此,由分析中的关系2可知: 2384 | \[ 600 x 95\%=(600+x) \x 75\%\] 2385 | 即:$8\x95=600+x$ 2386 | $\therefore\quad x=160$(克) 2387 | 2388 | 答:需要加水160克,就得到浓度是75\%的盐水. 2389 | \item 设加盐(溶质)$y$克, 2390 | 则加盐前,95\%的盐水600克中,含有水量: 2391 | $600\x (1-95\%)$克 2392 | 2393 | 加盐后,98\%的盐水$(600+y)$克中,含有水量: 2394 | $(600+y) \x (1-98\%)$克. 2395 | 2396 | 因此由分析中的关系3可知: 2397 | \[600 \x (1-95\%)=(600+y) \x (1-98\%)\] 2398 | 即:\[\begin{split} 2399 | 600\x5\%&=(600+y) \x 2\%\\ 2400 | 300\x 5&=600+y\\ 2401 | y&=900 \text{(克)} 2402 | \end{split}\] 2403 | 2404 | 答:只要加盐900克,就可得到浓度为98\%的盐 2405 | 水. 2406 | \end{enumerate} 2407 | \end{solution} 2408 | 2409 | \begin{example} 2410 | 实验室里只有浓度为8\%的盐水和浓度为 2411 | 5\%的盐水;现在需要把两种浓度的盐水混合起来, 2412 | 制成浓度为6\%的盐水300克,试问:这两种浓度的 2413 | 盐水各用多少克才能合适? 2414 | \end{example} 2415 | 2416 | \begin{analyze} 2417 | 这同样是浓度问题.解决这类稍复杂 2418 | 的混合溶液问题的关键是要抓住:\textbf{混合前、后的总重量不变;混合前、后纯溶质(盐等)的重量不变;混 2419 | 合前、后纯溶剂(水等)的重量当然也不变}等关系, 2420 | 引进未知数,列出方程式. 2421 | 2422 | 由此,对本题可作如下列表分析: 2423 | \begin{center} 2424 | \begin{tabular}{|c|c|c|c|} 2425 | \hline 2426 | 溶液 & \multicolumn{2}{c|}{混合前} &混合后\\ 2427 | \hline 2428 | 浓度 & $\quad8\%\quad$ & 5\% & 6\%\\ 2429 | \hline 2430 | 总重量& \multicolumn{2}{c|}{$x\text{克}\quad +\quad y\text{克}$} &300克\\ 2431 | \hline 2432 | 纯盐重 & \multicolumn{2}{c|}{$(x\cdot 8\%+y\cdot 5\%)\text{克}$} &$300\x 6\%$克\\ 2433 | \hline 2434 | \end{tabular} 2435 | \end{center} 2436 | \end{analyze} 2437 | 2438 | 2439 | \begin{solution} 2440 | 设需用浓度8\%的盐水$x$克,需用浓度5\%的盐水$y$克. 2441 | 则由题意可得: 2442 | \[\begin{cases} 2443 | x+y=300\\ 2444 | x\x 8\% +y\x 5\%=300\x 6\% 2445 | \end{cases}\] 2446 | 解这个二元一次方程组,可得: 2447 | \[(x,y)=(100,200) \] 2448 | 2449 | 答:用100克浓度为8\%的盐水和200克浓度为500 2450 | 的盐水,混合后就可得到浓度为6\%的盐水300克. 2451 | 2452 | \textbf{注:} 引入一个未知数也可以解这一题,试试 2453 | 看. 2454 | 2455 | \end{solution} 2456 | 2457 | \begin{ex} 2458 | \begin{enumerate} 2459 | \item 含糖15\%的糖水20斤. 2460 | \begin{enumerate} 2461 | \item 要想得到含糖10\%的糖水,需要加多少斤的水? 2462 | \item 要想得到含糖24\%的糖水,需要蒸 2463 | 发掉多少斤水? 2464 | \end{enumerate} 2465 | \item 两种铜块,分别含铜64\%和80\%.试问:这两种铜块各取 2466 | 多少克熔化以后,才能得到含铜74\%的铜块500克? 2467 | \end{enumerate} 2468 | \end{ex} 2469 | 2470 | 2471 | 2472 | \begin{example} 2473 | (我国古代问题):上禾三束、中禾二束、 2474 | 下禾一束,共有禾三十九斗;上禾二束、中禾三束、 2475 | 下禾一束,共有禾三十四斗;上禾一束、中禾二束、 2476 | 下禾三束,共有禾二十六斗. 2477 | 2478 | 问:上、中、下禾每束 2479 | 各有禾几斗? 2480 | \end{example} 2481 | 2482 | \begin{note} 2483 | 上、中、下禾,就是“上等谷物、中 2484 | 等谷物、下等谷物”;一束,就是“一捆”;斗,是 2485 | 古代一种衡器,可以用来量谷物的容积. 2486 | \end{note} 2487 | 2488 | \begin{solution} 2489 | 设上禾每束有禾$x$斗, 2490 | 中禾每束有禾$y$斗, 2491 | 下禾每束有禾$z$斗. 2492 | 则依题意可得出下列各方程: 2493 | \[\begin{cases} 2494 | 3x+2y+z=39\\ 2495 | 2x+3y+z=34\\ 2496 | x+2y+3z=26 2497 | \end{cases}\] 2498 | 解这个三元一次方程组,可以得出: 2499 | \[ (x,y,z)=\left(9\frac{1}{4},\; 4\frac{1}{4},\; 2\frac{3}{4}\right)\] 2500 | 2501 | 答:上禾每束有禾$9\frac{1}{4}$斗; 2502 | 中禾每束有禾$4\frac{1}{4}$斗; 2503 | 下禾每束有禾$2\frac{3}{4}$斗. 2504 | \end{solution} 2505 | 2506 | 2507 | 2508 | \section*{习题2.4} 2509 | \addcontentsline{toc}{subsection}{习题2.4} 2510 | 2511 | 适当选择未知数,列方程解下列问题: 2512 | \begin{enumerate} 2513 | \item 甲煤场有煤2000吨,每天运出15吨;乙煤场有煤800吨, 2514 | 每天运进25吨.几天后两煤场的煤存量相同? 2515 | \item 有一个仓库存货200吨,每天运出25吨;第二仓库存货 2516 | 80吨,每天运进5吨.几天后第一仓库存货是第二仓库存货 2517 | 的三倍? 2518 | \item $A, B$两油槽中,分别有油480升、180升,要使$A$中的油 2519 | 是$B$中的二倍,须从$A$中抽出多少油给$B$才行? 2520 | \item 生产队现有水田108亩,旱地54亩.要使全队的旱地只 2521 | 占水田的20\%,还应把多少亩旱地改为水田? 2522 | \item 一面靠墙,其余三面要用350米的篱笆围成一个长方形 2523 | 养猪场,要使它的长是宽的2倍,应如何围法? 2524 | \item 长方形的长与宽之比为5:2,周长28尺,试求这个长方 2525 | 形的面积. 2526 | \item 一桶煤油毛重8斤,从桶中取出煤油一半以后,毛重4.5 2527 | 斤,问:这一桶煤油有多少?桶重多少? 2528 | \item 小华现已读数学书41页,英语书28页.她的计划是每天 2529 | 读数学书5页、英语书4页.问:几天后,她读的数学书页 2530 | 数是英语书的两倍? 2531 | \item 甲、乙两人共有图书78本.如果甲赠给乙5本以后,两 2532 | 人的图书就一样多.问:甲、乙原来各有几本? 2533 | \item 一堆水果,如果用每筐能装45斤的小筐来装,可以装 2534 | 40筐,如果改用大筐装,每筐可比小筐多装$\frac{1}{10}$.问:这堆水 2535 | 果能装几大筐? 2536 | \item 一个班共有学生52人,其中共青团员、少先队员、同 2537 | 学的人数比为2:10:1.问:这个班应有团、队员和同学各几 2538 | 人? 2539 | \item 甲、乙两地相距162公里,甲地有一列慢车,每小时可 2540 | 以开48公里,乙地有一列快车,每小时可以开60公里,试 2541 | 问: 2542 | \begin{enumerate} 2543 | \item 两火车同时相向而行,多少时间可以相遇? 2544 | \item 两车同时相背而行,几小时以后,两车相距270公 2545 | 里. 2546 | \item 如果两车相向而行,慢车先开出一小时,再用多少 2547 | 时间两车才能相遇? 2548 | \item 两车同时同向而行(快车在后边).几小时后,快 2549 | 车可以追上慢车? 2550 | \item 两车同时同向而行(慢车在后边).几小时后,两 2551 | 车相距200公里? 2552 | \end{enumerate} 2553 | 2554 | \item 卡车以每小时30公里的速度由甲地开往乙地,半小时 2555 | 以后,小轿车以每小时40公里的速度也从甲地开往乙地,结 2556 | 果两车同时到达.问:甲、乙两地相距多少公里? 2557 | \item 甲、乙两运动员在四百米环形跑道上竞走,甲每分钟 2558 | 走150米,乙每分钟走100米.如果两人同时由同一起点出 2559 | 发,问:几分钟以后,甲才能再一次与乙相遇? 2560 | \item 买进水果若干筐,每筐原价3元.如果按照每筐四元 2561 | 的价钱卖出,那么,卖出全部水果的一半又10筐时,就已经 2562 | 收回了全部水果的成本.问:一共买进水果多少筐? 2563 | \item 轮船在静水里的速度是16公里/小时,由甲地顺水航行 2564 | 到乙地要7小时,逆水返回要9小时,试求甲、乙两地的距 2565 | 离. 2566 | \item 如果风速是3公里/小时,汽车速度是20公里/小时. 2567 | 要求这一汽车由$A$地顺风开往$B$地比逆风开回来少用1.5小 2568 | 时,试求:$A,B$两地的距离. 2569 | \item 一件工作,甲单独做5小时完成,乙单独做8小时完 2570 | 成.问:两人合做几小时完成? 2571 | \item 某项任务,师傅单独做4小时可以完成,徒弟单独做6 2572 | 小时可以完成.如果先由徒弟单独做2小时以后,师、徒二 2573 | 人再合做,试问:他们再做几小时,才能完成这一任务? 2574 | \item 某游泳池装有$A, B, C$三个进水管,若单独开放一管, 2575 | 分别要45分钟、1小时、1.5小时才能注满.问:三管一齐 2576 | 开,需要几小时才能注满? 2577 | \item 甲、乙、丙三个生产队合修一条水渠,计划用52人, 2578 | 按照各队收益面积的比3:4:6摊派劳动力.问:各队应派几 2579 | 人? 2580 | \item 100个工人,100一台机器,老工人一人看管3台,徒工 2581 | 3个人看管一台.问:有多少老工人?有多少徒工? 2582 | \item 现有含氮16\%的氨水30斤.要配置成0.15\%的氨水对 2583 | 菜地追肥.问:需要加多少水? 2584 | \item 现有含盐8\%和20\%的两种盐水,要配置成含盐12\% 2585 | 的盐水100公斤,两种盐水应各取多少? 2586 | 2587 | \item 在含酒精20\%的液体中,加入10公斤的水,就变成含 2588 | 酒精16\%的液体,试求:原来的液体有多少公斤? 2589 | \item 两块合金,一块含铜90\%,一块含铜80\%.现在要把 2590 | 两块熔合在一起,得到含铜82.5\%的合金240克,问:应各 2591 | 取多少克? 2592 | 2593 | \item 锡的比重是7.3${\rm g}/{\rm cm}^3$,铅的比重是11.3${\rm g}/{\rm cm}^3$,现在要想得到铅锡合金166g,它的比重是8.3${\rm g}/{\rm cm}^3$.试问:铅、锡应各取多少克? 2594 | 2595 | \item 一个容器装49升水,另一个容器装56升水,如果把第 2596 | 二个容器的水倒满第一个容器,那么第二个容器还剩下的水 2597 | 相当于这个容器容量的一半,如果把第一个容器的水倒满第 2598 | 二个容器,那么第一个容器还剩下的水相当于这个容器的容 2599 | 量的三分之一.求这两个容器的容量各是多少? 2600 | \item $M, N$两地相距24公里,公共汽车和直达快车在8点 2601 | 45分准时从$M$和$N$迎面开出.这两辆车平常都在8点52分相 2602 | 遇.有一次,直达快车晚开8分钟,就在8点57分与公共汽 2603 | 车相遇.试求这两种车的速度各是多少? 2604 | 2605 | \end{enumerate} 2606 | 2607 | 将下列应用题,引入两个(或三个)未知数求解. 2608 | 2609 | \begin{enumerate} 2610 | \item 甲数的7\%与乙数的8\%一共是12;而甲数的8\%与乙 2611 | 数的7\%的差是4.5,求这两个数. 2612 | \item 如果把长方形的长增加6米,宽减少3米,它的面积不 2613 | 变;如果长减少3米,宽增加2.4米,它的面积仍不变.试 2614 | 求长方形原来的长和宽各是多少? 2615 | \item 甲、乙两班学生,开学初团员共有8人,各占全班人 2616 | 数的$\frac{1}{10}$、$\frac{1}{12}$,学期末团员共有18人,各占全班人数的$\frac{1}{4}$、$\frac{1}{6}$. 2617 | 问:两班各有学生多少人? 2618 | \item 某人乘车以每小时40公里的速度可以按时由甲地到达 2619 | 乙地,但因出发时间晚了15分钟,汽车便以每小时50公里速 2620 | 度行驶,结果却比预定时间还提前3分钟到达乙地.试求: 2621 | 甲、乙两地之间的距离. 2622 | \item 小王与小张从相距36公里的$A, B$两地相向而行.如果 2623 | 小王先动身2小时,则他们在小张动身两小时三十分钟后相 2624 | 遇;如果小张先动身2小时,则他们在小王动身3小时后相 2625 | 遇.试问:小张与小王每小时各走多少公里? 2626 | \item 甲、乙二人同时绕400米的环形跑道行走.如果他们同 2627 | 时从同一起点背向而行,2,5分钟可以相遇;如果他们同时 2628 | 由同一起点同向而行,12.5分钟甲能追上乙,试求甲、乙两 2629 | 人每分钟各走几米? 2630 | \item 一批零件共420个,如果甲先做2天,再与乙合做2天 2631 | 可以做完,如果乙先做2天,再与甲合做3天可以做完.试 2632 | 求:甲、乙两人每人每天各能做多少个零件? 2633 | \item 有52人要担任运土、挖土两项任务,如果每人平均挖土 2634 | 6方或运土480.公斤(已知每方土重1.8吨),问:如何分 2635 | 配人力,才能使挖土、运土主作配套,不至于窝工? 2636 | \item 一艘轮船载重量为520吨,容积为3000立方米.现有 2637 | $A, B$两种货物,$A$货每吨有2立方米,$B$货每吨有8立方 2638 | 米.问:这艘船应装$A, B$两种货物各多少吨,才能充分利 2639 | 用它的船仓? 2640 | \item 有三个数总和是15.第一个数与第二个数的差恰是第 2641 | 二个数与第三个数的差,而且第二、三两数的和比第一个数 2642 | 大2.求这三个数各是多少? 2643 | \item 有人民币三种:二角、伍角、一元共37张,价值13元9 2644 | 角.如果已知二角票的张数是伍角票张数的二倍,问:三种 2645 | 票各有几张? 2646 | \item (我国古代问题)今有上禾二束、中禾兰束、下禾四 2647 | 束都有谷不满一斗,若上禾取中禾、中禾取下禾,下禾取上 2648 | 禾各一束,则有谷都正好一斗,问:上、中、下禾一束各有 2649 | 谷几斗? 2650 | 2651 | \end{enumerate} 2652 | 2653 | \section*{本章内容要点} 2654 | 2655 | 一、这一章主要是应用数系运算通性及等式性 2656 | 质,解一次方程和方程组,并且能解决相应的应用问 2657 | 题. 2658 | 2659 | \vskip 2ex 2660 | 2661 | 二、含有未知数的等式,叫做方程.能使方程式 2662 | 两边相等的未知数的值,就是方程的解. 2663 | 2664 | \vskip 2ex 2665 | 2666 | 三、含有一个未知数的方程,叫做一元方程,如 2667 | 果只含有一个未知数,分母不含未知数,且最高次项 2668 | 的指数是1的方程,叫做一元一次方程.解一元一次 2669 | 方程的原理和方法是:由数的运算通性和等式性质, 2670 | 归纳出去分母、去括号、移项变号、合并同类项、除 2671 | 以未知数的系数等具体规则,应用这些具体规则,就 2672 | 可以把方程的解求出来. 2673 | 2674 | \vskip 2ex 2675 | 2676 | 四、含有两个未知数的一次方程,叫做二元一次 2677 | 方程.二元一次方程的解是一个数值组,记作$(x, y)$; 2678 | 任一个二元一次方程的解都有无限多个. 2679 | 2680 | \vskip 2ex 2681 | 2682 | 五、两个以上的方程联合在一起,组成一个方程 2683 | 组.因而,由几个含有两个未知数的一次方程所组成 2684 | 的方程组二网做三元一次方程组.能够同时满足方程 2685 | 组中所有各个方程的未知数所取的数值组,叫做这个 2686 | 方程组的解. 2687 | 2688 | \vskip 2ex 2689 | 2690 | 六、解多元一次方程组的关键是消元.具体方法 2691 | 有:加、减消元法和代入消元法. 2692 | 2693 | \vskip 2ex 2694 | 2695 | 这里应特别指出:加减法消元是较普遍且重要的 2696 | 方法,其要点就是,应用等式的性质,将两个方程中 2697 | 相同的某一个未知数的系数变形成为绝对值相等.然 2698 | 后把方程的两边分别相加或相减,就可以消去这个未 2699 | 知数. 2700 | 2701 | 因此,可以说:解多元一次方程的过程,就是逐 2702 | 步消元求解的过程.即 2703 | \begin{center} 2704 | \begin{tikzpicture}[>=latex, scale=1.3] 2705 | \node (A) at (0,0) {\Large 多元} ; 2706 | \node (B) at (2,0) {\Large ……} ; 2707 | \node (C) at (4,0) {\Large 三元} ; 2708 | \node (D) at (6,0) {\Large 二元} ; 2709 | \node (E) at (8,0) {\Large 一元} ; 2710 | 2711 | \draw[->] (A)--node[above]{消元}(B); 2712 | \draw[->] (B)--node[above]{消元}(C); 2713 | \draw[->] (C)--node[above]{消元}(D); 2714 | \draw[->] (D)--node[above]{消元}(E); 2715 | 2716 | \end{tikzpicture} 2717 | \end{center} 2718 | 2719 | \vskip 2ex 2720 | 2721 | 七、解应用问题就是运用数学工具解决实际问 2722 | 题,这就要: 2723 | \begin{enumerate} 2724 | \item 审题:弄清题意,分析问题中涉及到的量与量 2725 | 之间的关系. 2726 | \item 引入未知数,并用未知数表示出有关的量. 2727 | \item 正确列出方程(或方程组).一般来说,引入 2728 | 的未知数个数与所列方程的个数是相等的. 2729 | \item 准确地求出来知数的值,即所列方程(方程 2730 | 组)的解. 2731 | \item 检验:所列方程(方程组)的解是否符合题 2732 | 意.将不合理的值舍去,从而写出正确答案. 2733 | \end{enumerate} 2734 | 2735 | 其中,分析量之间的关系是列方程的关键;列方 2736 | 程是解决问题的基础;解方程又是解决问题的主要手 2737 | 段.这都是解应用问题时,必须注意的几个重要环节. 2738 | 2739 | \section*{复习题二} 2740 | \addcontentsline{toc}{section}{复习题二} 2741 | 2742 | \begin{enumerate} 2743 | \item 如果已知汽车的速度是$a$公里/小时,摩托车的速度是$b$ 2744 | 公里/小时 $(a>b)$.试求: 2745 | \begin{enumerate} 2746 | \item 两车同时、同地、同向开出,5小时后,两车相距 2747 | 多少公里? 2748 | \item 一个人坐汽车走了$m$小时,接着又坐摩托车走了,小 2749 | 时,一共走了多少公里? 2750 | \item 两车都开出了10公里,汽车比摩托车少用几小时? 2751 | \item 两车都开了$d$小时,摩托车比汽车少走多少公里? 2752 | \end{enumerate} 2753 | 2754 | \item 用图中已知的字母,表示出阴影部分的面积来: 2755 | \begin{center} 2756 | \begin{tikzpicture}[>=latex,scale=.7] 2757 | \begin{scope} 2758 | \draw[pattern=north east lines] (2,1.5) circle (2.5); 2759 | \draw[fill=white] (0,0) rectangle (4,3); 2760 | \draw[<->](0,0)--node[above]{$2R$}(4,3); 2761 | \node at (2,0)[above]{$a$}; 2762 | \node at (0,1.5)[right]{$b$}; 2763 | 2764 | \end{scope} 2765 | \begin{scope}[xshift=6cm] 2766 | \draw[pattern=north east lines] (0,0) rectangle (5,3); 2767 | \draw[fill=white] (.5,0) arc (180:0:2); 2768 | \draw [->] (2.5,0)--node[above]{$R$}+(40:2); 2769 | \node at (2.5,3)[above]{$a$}; 2770 | \node at (5,1.5)[right]{$b$}; 2771 | \end{scope} 2772 | \begin{scope}[yshift=-6cm] 2773 | \draw[pattern=north east lines] (0,0) arc (180:0:2.6); 2774 | \draw[fill=white](2.6,1.3) circle(1.3); 2775 | \draw (0,0)--(5.2,0); 2776 | \draw[|<->|] (0,-.2)--node[below]{$D$}(5.2,-.2); 2777 | \end{scope} 2778 | \begin{scope}[xshift=6cm, yshift=-6cm] 2779 | 2780 | \fill [pattern=north east lines] (0,0) rectangle (6,4); 2781 | \fill [white](0,0) rectangle (1,1); 2782 | \fill [white](0,3) rectangle (1,4); 2783 | \fill [white](5,0) rectangle (6,1); 2784 | \fill [white](5,3) rectangle (6,4); 2785 | 2786 | \foreach \x in {0,4} 2787 | { 2788 | \draw (1,\x)--(5,\x); 2789 | \draw (\x*1.5, 1)--(\x*1.5, 3); 2790 | } 2791 | 2792 | \draw (0,1)--(1,1)--node[left]{$x$}(1,0); 2793 | \draw (0,3)--(1,3)--node[left]{$x$}(1,4); 2794 | \draw (5,0)--(5,1)--(6,1); 2795 | \draw (5,4)--(5,3)--node[above]{$x$}(6,3); 2796 | \node at (3,4)[above]{$4x$}; 2797 | \node at (6,2)[right]{$2x$}; 2798 | 2799 | \end{scope} 2800 | \end{tikzpicture} 2801 | \end{center} 2802 | 2803 | \item 解下列方程式: 2804 | \begin{enumerate} 2805 | \item $\frac{x}{2}-7-\frac{x}{5}=\frac{1}{5}-\frac{x}{4}+\frac{x}{20} $ 2806 | \item $\frac{1}{3}\left(3x+\frac{10-7x}{2}\right)-\frac{1}{6}\left(2x+\frac{2x+2}{3}\right)=\frac{x}{2}-1$ 2807 | \item $3\left\{x-\frac{3x+1}{4}-\left[1-2\left(x-\frac{3-x}{5}\right)\right]\right\}=8x-6$ 2808 | \item $(x-4)(x+3)=(x+1)(x+2)$ 2809 | \item $(2x+3)(2x-3)=(2x-3)^2$ 2810 | \end{enumerate} 2811 | 提示:最后两题先用分配律展开. 2812 | 2813 | \item \begin{enumerate} 2814 | \item 在公式$V=\frac{\pi\cdot N\cdot D}{1000}$中,已知$V=120$,$D=80$,$\pi=3.14$,求$N$(保留两位小数). 2815 | \item 如果球的体积公式是$V=\frac{4}{3}\pi R^3$,已知球的半径$R=20{\rm cm}$,体积$V=3.2\x 10^4{\rm cm}^3$.试计算$\pi$的值应取多少? 2816 | \end{enumerate} 2817 | 2818 | \item 解下列字母系数的方程式: 2819 | \begin{enumerate} 2820 | \item $mx+n=m(2x+1)\qquad (m\ne 0)$ 2821 | \item $mx+\frac{x}{m}=1\qquad (m\ne 0)$ 2822 | \item $\frac{x}{a}+\frac{1}{b}=2\qquad (a\ne 0,\quad b\ne 0)$ 2823 | \end{enumerate} 2824 | 2825 | \item 试求下列各题中的$k$值. 2826 | \begin{enumerate} 2827 | \item $kx^2-12x-3=0$ 有一个根为1 2828 | \item $x^2+kx+15=0$ 有一个根为$-\frac{1}{2}$ 2829 | \item $x^2+kx-2k=0$ 有一个根为$0$ 2830 | \item $x^2-2ax+k=0$ 有一个根为$a$ 2831 | \end{enumerate} 2832 | 2833 | \item 解下列方程: 2834 | \[|x-2|=1,\qquad |2x-4|+|x-2|=0\] 2835 | 2836 | \item 求不定方程$3x+5y=15$的非负整数解. 2837 | 2838 | \item 已知方程组$\begin{cases} 2839 | mx+2y=n\\ 4x-ny=2m-1 2840 | \end{cases}$的解是$(x,y)=(1,-1)$,试求$m,n$的值. 2841 | 2842 | \item 解下列方程组: 2843 | \begin{enumerate} 2844 | \begin{multicols}{2} 2845 | \item $\begin{cases} 2846 | \frac{1}{2}x-y+1=0\\ 2\frac{1}{2}x+6y=28 2847 | \end{cases}$ 2848 | \item $\begin{cases} 2849 | \frac{x}{2}=\frac{y}{5}\\ 75\%x+40\%y=1.4 2850 | \end{cases}$ 2851 | \end{multicols} 2852 | 2853 | \item $\begin{cases} 2854 | \frac{2x-3y+1}{2}+\frac{3x-2y-3}{3}=1\\ 2855 | \frac{x+2y+6}{4}-\frac{4x+2y-2}{5}=0 2856 | \end{cases}$ 2857 | \item $\frac{2x+y+6}{4}=\frac{4x-3y-7}{8}=\frac{-6x-7y+10}{8}$ 2858 | \item $2x-y+11=-7y+3z=7x+z+7=7$ 2859 | \begin{multicols}{2} 2860 | \item $\begin{cases} 2861 | \frac{y}{2}+\frac{z}{3}=x+2\\x-y=\frac{1}{2}-\frac{z}{4}\\\frac{x}{4}+\frac{y}{2}=\frac{z}{2}-\frac{7}{4} 2862 | \end{cases}$ 2863 | \item $\begin{cases} 2864 | x+y=3a\\y+z=4a\\z+x=5a 2865 | \end{cases}$($a$是已知数) 2866 | \item $\begin{cases} 2867 | x+2y=2\\y+3z=1\\z-t=4\\x+y+z+t=-1 2868 | \end{cases}$ 2869 | \item $\begin{cases} 2870 | x+y+z+u=a\\ x+y-z-u=b\\ x-y+z-u=c\\x-y-z-u=d 2871 | \end{cases}$ 2872 | \end{multicols} 2873 | \end{enumerate} 2874 | 2875 | \item 如果已知$3x-z=x+y+z=4x+2y-z$,试求:$x:y:z$ 2876 | \begin{multicols}{2} 2877 | \begin{enumerate} 2878 | \item $\begin{cases} 2879 | x=t\\ y=2t^2 2880 | \end{cases}$ 2881 | \item $\begin{cases} 2882 | x=3-t\\ y-5=t+1 2883 | \end{cases}$ 2884 | \end{enumerate} 2885 | \end{multicols} 2886 | 2887 | \end{enumerate} 2888 | 2889 | \textbf{适当引入未知数,列出方程或方程组解下列各应用题: 2890 | } 2891 | 2892 | \begin{enumerate} 2893 | \item 如果有三个紧相连的整数之和为126,试求这三个整 2894 | 数. 2895 | \item 如果三个相邻的偶数之和为114,求出这三个偶数来. 2896 | \item 一个分数的分子是分母的$\frac{5}{8}$,且分子、分母之和为 2897 | 143,试求这个分数. 2898 | \item 一个二位的自然数恰好等于它的两个数字之和的五倍, 2899 | 试求这个二位数是多少? 2900 | \item 一个四位数$7abc$,如果把7调到最后一位上,就变成 2901 | 另一个四位数$abc7$,那么,这个四位数就比原来减少了864, 2902 | 试求这个四位数. 2903 | \item 一个六位数$1abcde$的三倍,正好是六位数$abcde1$.试 2904 | 求这个六位数. 2905 | 2906 | \textbf{提示:} 这类数字问题,均可设未知部分为$x$,再由已知数字所在的位数,把这个数表示成含二的算式.如: 2907 | 2908 | 第5题中,设$abc=x$,则$7abc=7000+abc = 7000+x$ 2909 | 2910 | 第6题中,设$abcde = x$,则$1abcde =100000 + x$等 2911 | 2912 | \item 一个长方形的长是宽的三倍,如果把宽增加3米,这个 2913 | 长方形就变成一个正方形了.试求这个长方形的面积是多少? 2914 | 2915 | \item 一个正方形环(如图)面积 2916 | 为56${\rm cm}^2$,这个环形各边的 2917 | 宽都是2cm,试求这个环形的 2918 | 外周长、里周长各是多少? 2919 | 2920 | \begin{center} 2921 | \begin{tikzpicture}[>=latex, scale=.7] 2922 | \fill[pattern=north east lines, draw] (0,0) rectangle (4.5,4.5); 2923 | \draw [fill=white](1.2,1.2) rectangle (3.3,3.3); 2924 | \draw [<->](3.3,1.5)--node[fill=white, above]{2}(4.5,1.5); 2925 | \end{tikzpicture} 2926 | \end{center} 2927 | 2928 | \item 老陈用13.10元钱,买笔记本和笔尖两种用品,笔记本 2929 | 与笔尖的单价分别是1.50元与0.80元,你能知道老陈买了几 2930 | 本笔记本?几个笔尖? 2931 | \item 一批任务,原计划每天要完成120件,实际上每天比原 2932 | 计划多完成40件,结果提前6天完成任务.问:原计划几天 2933 | 完成?这批任务一共多少件? 2934 | \item 加工一批零件,第一天完成$\frac{1}{2}$,第二天完成剩下的$\frac{1}{3}$, 2935 | 这时只剩下18个零件没有加工.试问:第一、二两天各加工 2936 | 了多少个零件? 2937 | \item 抽水机若干台,在规定时间内可以完成任务,如果减 2938 | 少3台机器,就要延长6小时,如果增加2台机器,就能提 2939 | 前2小时完成任务,问:原有几台抽水机?原计划几小时完 2940 | 成所给任务? 2941 | \item 甲从$A$到$B$用8小时,乙从$B$到$A$用12小时,且甲比乙 2942 | 每小时多走10里,如果甲、乙二人同时由$A, B$相向而行,经 2943 | 过几小时,他们两人在距A多远的地方相遇? 2944 | \item 甲、乙、丙三人,甲每分钟走20丈,乙每分钟走22.5 2945 | 丈,丙每分钟走25丈.如果甲、乙二人在东村,丙在西村, 2946 | 他们三人同时相向而行.丙遇到乙以后三分钟才遇到甲.求 2947 | 东、西两村之间的距离是多少丈? 2948 | \item 甲、乙二人同绕400米环形道赛跑,如果他们同时由 2949 | 同一起点出发背向而跑,25秒相遇,如果他们同时由同一起 2950 | 点同向而跑,2分零5秒甲能追上乙.试求甲、乙二人每秒 2951 | 钟各跑几米? 2952 | \item 某市举行环城自行车赛.35分钟以后最快的运动员遇 2953 | 到最慢的.已知环城一周为6公里,最慢的运动员的速度是 2954 | 最快的速度的$\frac{5}{7}$.试求这两名运动员的速度各是多少? 2955 | \item 两个伐木队在一月份共伐木900立方米.二月份比一月 2956 | 份第一队多伐15\%,第二队多伐12\%,因而二月份共伐木 2957 | 1020立方米.试求二月份两队各伐木多少? 2958 | \item $A, B$两块合金,$A$重12公斤,含纯银70\%; $B$含纯银 2959 | 56\%,现用两块合金熔化后得到含纯银60\%的新合金,求$B$ 2960 | 合金的重量. 2961 | \item 水池的进水管被堵,流量减少40,试间:注满一池 2962 | 水所需要的时间增加了百分之几? 2963 | \item 由金、银、铜制成三种合金$A, B,C$;已知金、银、铜 2964 | 的重量比在三种合金中分别是:在$A$中$1:3:2$,在$B$中 2965 | $2:1:1$;在$C$中$1:2:5$.而且知道三种合金中总共含金 2966 | 5.5克,含银8克,含铜9.5克.试求:三种合金$A, B,C$各 2967 | 自的重量. 2968 | \item 某车间每天可以生产甲种零件300个或乙种零件500个 2969 | 或丙种零件600个.这三种零件各一个可以配成一套.现在 2970 | 要在63天的生产中,便所生产的三种零件全部配套.试问: 2971 | 这个车间应该对这三种零件的生产各用几天才成? 2972 | \item 小光用0.93元买了单价各为0.10元、0.12元、0.15元的 2973 | 铅笔一共8支,如果把0.10元的铅笔数与0.12元的对调一 2974 | 下,就可以节约0. 02元钱.问:小光原先买三种铅笔各多少 2975 | 支? 2976 | 2977 | \item (中国古代数学问题) 2978 | \begin{enumerate} 2979 | \item 好马日行240里,劣马日行150里.今劣马先行12日, 2980 | 好马几日可追上劣马? 2981 | \item 以绳测井.若将绳三折测之,绳多4尺,若将绳四 2982 | 折测之,绳多1尺.求绳长,井深各几何? 2983 | \item 今卖牛二、羊五,可买猪十三,尚余钱一千;若卖 2984 | 牛三、猪三,恰可买羊九;若卖羊六,猪八,只能买牛五, 2985 | 还欠钱六百.试问:牛、羊、猪的单价各是多少? 2986 | \end{enumerate} 2987 | 2988 | 2989 | \item 由海里提出40升海水,经化验知道其中含清水36.6 2990 | 升;在这40升海水中又增添了若干清水后,再从中提出40 2991 | 升,经化验后知道,这时含清水38升.试求:增添了多少清 2992 | 水? 2993 | 2994 | \end{enumerate} 2995 | 2996 | -------------------------------------------------------------------------------- /6.tex: -------------------------------------------------------------------------------- 1 | \chapter{分式与根式} 2 | \section{分式与分式方程} 3 | \subsection{分式与分式的基本性质} 4 | 我们已经知道,两个整数$m,n$的比,是一个有 5 | 理数$\frac{m}{n},\quad (n\ne 0)$. 同样,两个多项式$f(x)$和$g(x)$的比$\frac{f(x)}{g(x)},\quad (g(x)\ne 0)$就叫做\textbf{有理式}. 6 | 7 | 在有理式$\frac{f(x)}{g(x)},\quad (g(x)\ne 0)$中,如果$g(x)$的次数为0,那么,有理式 8 | $\frac{f(x)}{g(x)}$就是\textbf{整式},也就是我们已 9 | 经学过的多项式;如果$g(x)$的次数高于0次,那么, 10 | 有理式$\frac{f(x)}{g(x)}$就叫做\textbf{分式}. 11 | 12 | 例如:$\frac{1}{x}$, $\frac{x+5}{x^2-9}$, $\frac{2x^2-3x+1}{x-3}$, $\frac{x^2+y^2}{2x+y}$等都是分式.$\frac{(x^2+1)^2}{x^2+1}$虽然恒等于整式$x^2+1$,而从形式上仍然叫做分式. 13 | 14 | 但是,像$\frac{x^2-1}{1}$、$\frac{3x+1}{2}$等都是整式,而这些整式也就是多项式:$x^2-1$,$\frac{3}{2}x+\frac{1}{2}$等. 15 | 16 | 对于整式来说,由于其中的未知数决不会出现在分母当中,因而未知数可以取一切实数值;但对于分式来说,由于它的分母中必定含有未知数,因而未知数的取值,就要求限制在“使分母不等于零的实数值”的范围内.例如: 17 | 18 | 在分式$\frac{2x+3}{x^2-3}$中,未知数$x$只允许取“$x^2-3\ne 0$”的值,即 19 | $x\ne \pm\sqrt{3}$ 20 | 的一切实数值.也就是说,在这个分式中的未知数可以取除“$\pm\sqrt{3}$”以外的其它任何实数值. 21 | 22 | 在分式中,分子的次数如果低于分母的次数,就叫做\textbf{真分式};分子的次数如果不低于分母的次数,就 23 | 叫做\textbf{假分式}.例如,$\frac{1}{x-3}$,$\frac{x+y}{2x^2+y}$等都是真分式;$\frac{2x+4}{x-3}$,$\frac{x^2+4x+6}{x+3}$,$\frac{x^2+y^2}{2x+y}$等都是假分式. 24 | 25 | \subsubsection{分式的基本性质} 26 | 27 | \begin{enumerate} 28 | \item 分式的分子、分母同乘以一个非零多项式, 29 | 分式的值不变.用式子表示就是: 30 | 31 | 如果$h(x)\ne 0$,那么$\frac{f(x)}{g(x)}=\frac{f(x)\cdot h(x)}{g(x)\cdot h(x)}$. 32 | 33 | 例如:$\frac{x}{x-3}=\frac{x(x-2)}{(x-3)(x-2)}\quad (x\ne 3,\; x\ne 2)$ 34 | 35 | \item 分式的分子、分母同除以一个非零多项式. 36 | 分式的值不变.用式子表示就是: 37 | 38 | 如果$h(x)\ne 0$,那么$\frac{f(x)}{g(x)}=\frac{f(x)\div h(x)}{g(x)\div h(x)}$. 39 | 40 | 例如:$\frac{x^2(x^2+1)}{x^2+1}=x^2$ 41 | \end{enumerate} 42 | 43 | 利用以上两个基本性质,可以进行分式的约分和通分. 44 | 45 | \subsubsection{约分} 46 | 47 | 如果一个分式的分子、分母有公因式时,可以类似于分数的约分,把最高公因式约去,使分式化简.分式的分子与分母没有非零次的公因式时,叫做\textbf{不可约分式(既约分式)}.不可约分式是最简分式,约分就是化分式为最简分式. 48 | 49 | \begin{example} 50 | 约简 51 | \begin{enumerate} 52 | \item $\frac{18 a^{12} x^{3} y^{2}}{12 a^{3} b^{2} x^{5} z}$ 53 | \item $\frac{8 a^{2}-6 a^{5} b}{16 a^{20}}$ 54 | \item $\frac{32(x-2 y)^{2}(-x+y)}{24(2 y-x)^{2}(x-y)}$ 55 | \item $\frac{m^{2}-4 n^{2}}{-m^{2}-4 m n-4 n^{2}}$ 56 | \end{enumerate} 57 | \end{example} 58 | 59 | \begin{solution} 60 | \begin{enumerate} 61 | \item $\frac{18 a^{12} x^{3} y^{2}}{12 a^{3} b^{2} x^{5} z}=\frac{3 a^{8} y^{2}}{2 b^{3} x^{2} z}$ 62 | \item $\frac{8 a^{2}-6 a^{5} b}{16 a^{20}} =\frac{2 a^{2}\left(4-3 a^{3} b\right)}{16 a^{20}} = \frac{4-3 a^{3} b}{8 a^{18}} $ 63 | \item $ \frac{32(x-2 y)^{2}(-x+y)}{24(2 y-x)^{2}(x-y)} =\frac{-4(x-2 y)^{2}(x-y)}{3(x-2 y)^{2}(x-y)}=-\frac{4}{3}$ 64 | \item \[\begin{split} 65 | \frac{m^{2}-4 n^{2}}{-m^{2}-4 m n-4 n^{2}} &= 66 | \frac{m^{2}-4 n^{2}}{-\left(m^{2}+4 m n+4 n^{2}\right)}\\&=-\frac{(m-2n)(m+2n)}{(m+2n)^2}\\&=-\frac{m-2n}{m+2n} 67 | \end{split} \] 68 | \end{enumerate} 69 | \end{solution} 70 | 71 | \begin{example} 72 | 约简 $\frac{\poly{1,-2,0,1,-2}}{\poly{1,0,1,0,1}}$ 73 | \end{example} 74 | 75 | \begin{solution} 76 | 用辗转相除法求得: 77 | \[(\poly{1,-2,0,1,-2}, \poly{1,0,1,0,1})=\poly{1,-1,1}\] 78 | 用除法求得 79 | \[\begin{split} 80 | \poly{1,-2,0,1,-2}&=(\poly{1,-1,1})(\poly{1,-1,-2})\\ 81 | \poly{1,0,1,0,1}&=(\poly{1,-1,1})(\poly{1,1,1}) 82 | \end{split}\] 83 | $\therefore\quad \frac{\poly{1,-2,0,1,-2}}{\poly{1,0,1,0,1}}=\frac{\poly{1,-1,-2}}{\poly{1,1,1}}$ 84 | \end{solution} 85 | 86 | \begin{example} 87 | 约简 $\frac{\poly{1,-6,11,-6}}{\poly{1,-8,11,-12}}$ 88 | \end{example} 89 | 90 | \begin{solution} 91 | 用余式定理和综合除法求得: 92 | \[\begin{split} 93 | \poly{1,-6,11,-6}&=(x-1)(x-3)(x-2)\\ 94 | \poly{1,-8,11,-12}&=(x-1)(x-3)(x-4) 95 | \end{split}\] 96 | $\therefore\quad \frac{\poly{1,-6,11,-6}}{\poly{1,-8,11,-12}}=\frac{x-2}{x-4}$ 97 | \end{solution} 98 | 99 | 下面我们和多项式一样,引入一个分式的符号:$F(x)$,它表示关于$x$的一个分式;同样,$G(x)$、 100 | $T(x)$可以分别表示$x$的另外的分式,例如:$F(x)=\frac{1}{x}$, $G(x)=\frac{2x}{x^2-3}$ 101 | $T (x) =\frac{7x}{9-x}$等.很自然,在一个 102 | 分式$F(x)$中,当$x=a$时,分式的值可以表示为$F (a)$. 103 | 104 | \begin{example} 105 | 先把下面的分式化简,再求它的值. 106 | \[F(x)=\frac{-1-x^3}{2x^2-2x+2},\qquad \text{其中 } x=5 \] 107 | \end{example} 108 | \begin{solution} 109 | \[\begin{split} 110 | F(x)&=\frac{-(x^3+1)}{2(x^2-x+1)}\\ 111 | &=-\frac{(x+1)(x^2-x+1)}{2(x^2-x+1)}=-\frac{x+1}{2}\\ 112 | F(5)&=-\frac{5+1}{2}=-3 113 | \end{split}\] 114 | \end{solution} 115 | 116 | \begin{example} 117 | 约简$F(x)=\frac{2(x^2-1)(x-1)^2}{(1-x)^3(x+1)^2}$,问$x$取什么整数值时,能使$F(x)$的值是正整数. 118 | \end{example} 119 | 120 | \begin{solution} 121 | \[\begin{split} 122 | F(x)&=\frac{2(x+1)(x-1)(x-1)^2}{-(x-1)^3 (x+1)^2}\\ 123 | &=-\frac{2(x+1)(x-1)^3}{(x-1)^3(x+1)^2}\\ 124 | &=-\frac{2}{x+1} 125 | \end{split}\] 126 | 127 | 要使$F(x)$的值是正整数,必须使$x+1=-1$或$x+1=-2$. 128 | 129 | 解得$x=-2$或$x=-3$. 130 | 131 | $\therefore\quad $当$x=-2$或$x=-3$时,$F(x)$的值是正整数. 132 | \end{solution} 133 | 134 | 由以上各例,可以得出: 135 | \begin{enumerate} 136 | \item 约分应当约去分子与分母的最高公因式及分子与分母的系数的最大公约数; 137 | \item 如果分式的分子、分母是多项式,可以把 138 | 它们分别分解因式以后,再进行约分;对于高次多项式,应用余式定理或辗转相除法可以求得最高公因式. 139 | \end{enumerate} 140 | 141 | \subsubsection{通分} 142 | 143 | 对于分母不相同的几个分式,可将每个分式的分子、分母乘以适当的非零多项式,而使各分式的分母都相同,这种运算叫做\textbf{通分}.通分时应取原来每个分式的分母的最低公倍式与它们各系数的最小公倍数之积作公分母. 144 | 145 | \begin{example} 146 | 把$\frac{a}{2b},\quad \frac{b}{3a^2},\quad \frac{c}{4ab}$通分. 147 | \end{example} 148 | 149 | \begin{solution} 150 | $[2b,\; 3a^2,\; 4ab]=12a^2b$ 151 | 152 | 因此: 153 | \[\begin{split} 154 | \frac{a}{2b}&=\frac{a\cdot 6a^2}{2b\cdot 6a^2}=\frac{6a^3}{12a^2b}\\ 155 | \frac{b}{3a^2}&=\frac{b\cdot 4b}{3a^2\cdot 4b}=\frac{4b^2}{12a^2b}\\ 156 | \frac{c}{4ab}&=\frac{c\cdot 3a}{4ab\cdot 3a}=\frac{3ac}{12a^2b}\\ 157 | \end{split}\] 158 | \end{solution} 159 | 160 | \begin{example} 161 | 把$\frac{2x}{x^2-y^2}$,$\frac{3y}{x^3+y^3}$通分. 162 | \end{example} 163 | 164 | \begin{solution} 165 | 由于: 166 | \[\begin{split} 167 | x^2-y^2&=(x+y)(x-y)\\ 168 | x^3+y^3&=(x+y)(x^2-xy+y^2) 169 | \end{split}\] 170 | 171 | $\therefore\quad [x^2-y^2, x^3+y^3]=(x-y)(x+y)(x^2-xy+y^2)$ 172 | 173 | 因此: 174 | \[\begin{split} 175 | \frac{2x}{x^2-y^2}&=\frac{2x(x^2-xy+y^2)}{(x-y)(x+y)(x^2-xy+y^2)}\\ 176 | \frac{3y}{x^3+y^3}&=\frac{3y(x-y)}{(x-y)(x+y)(x^2-xy+y^2)} 177 | \end{split}\] 178 | \end{solution} 179 | 180 | \begin{ex} 181 | \begin{enumerate} 182 | \item 不改变分式的值,使分子、分母的最高次幂的系数变为正数. 183 | \begin{multicols}{2} 184 | \begin{enumerate} 185 | \item $\frac{-2 a}{-3 b}$ 186 | \item $\frac{7 x^{2}}{-9 y^{2}}$ 187 | \item $ \frac{1-3 x^{2}+2 x}{7+7 x-x^{2}}$ 188 | \item $-\frac{-x^{2}-2 x^{4}+5}{x^{2}+2 x^{4}-5}$ 189 | \end{enumerate} 190 | \end{multicols} 191 | 192 | \item 193 | \begin{enumerate} 194 | \item 在什么条件下 $\quad \frac{3 a-6 b}{a+b}=0$ 195 | \item 在什么条件下 $\quad \frac{2 a-b}{b-a}=1$ 196 | \end{enumerate} 197 | 198 | 199 | \item 约简下列各分式: 200 | \begin{enumerate} 201 | \begin{multicols}{2} 202 | \item $\frac{8 x^{5} y^{7}}{-12 x^{3} y^{2}}$ 203 | \item $\frac{32 p^{4} q^{3}}{16 p^{3} q^{4}}$ 204 | \item $\frac{-6 a^{2} b^{4}}{-3 a^{4} b^{5}}$ 205 | \item $\frac{15 m^{3} n^{4}(a+b)^{3}}{18 m^{2} n^{3}(a+b)^{4}}$ 206 | \item $\frac{2(3 x-2 y)}{3(2 y-3 x)}$ 207 | \item $\frac{-a b(x+y)^{3}(x-y)}{b(x+y)^{2}(y-x)^{2}}$ 208 | \item $\frac{3 a(x-y)^{3}}{15(y-x)^{3}}$ 209 | \item $\frac{(a-b)(b-c)(c-a)}{(b-a)(a-c)(c-b)}$ 210 | \end{multicols} 211 | \item $\frac{-(x+y-z)(x-y+z)(x-y-z)}{(y+z-x)(y-z+x)(y-z-x)}$ 212 | \begin{multicols}{2} 213 | \item $\frac{x^{2}-y^{2}}{x^{2}-2 x y+y^{2}}$ 214 | \item $\frac{1-x^{3}}{x^{2}-1}$ 215 | \item $\frac{x^{2}+9 x+14}{x^{2}+8 x+7}$, 216 | \item $\frac{9 a^{4}-1}{6 a^{2} b^{2}+2 b^{2}}$ 217 | \item $\frac{a^{2}+b^{2}-c^{2}+2 a b}{a^{2}-b^{2}+c^{2}-2 a c}$ 218 | \item $\frac{1+3 a^{2}-3 a-a^{3}}{\left(a^{2}-2 a+1\right)\left(2 a^{2}-3 a+1\right)}$ 219 | \item $\frac{6 x^{3}+11 x^{2}-x-6}{12 x^{3}-8 x^{2}-27 x+18}$ 220 | \item $\frac{x^{4}-2 x^{2}-3 x-2}{2 x^{4}-4 x^{3}+2 x-4}$ 221 | \item $\frac{x^{3}+x^{2}-5 x-2}{x^{3}+2 x^{2}-2 x-1}$ 222 | \end{multicols} 223 | \end{enumerate} 224 | 225 | 226 | \item 通分 227 | \begin{enumerate} 228 | \begin{multicols}{2} 229 | \item $\frac{z}{10 x^{2} y^{3}},\quad \frac{y}{10 x^{3} z^{2}}$ 230 | \item $\frac{c}{-2 a b}, \quad \frac{b}{3 a c}, \quad \frac{a}{5 b c}$ 231 | \item $\frac{4}{3 y},\quad \frac{y-1}{-2 y^{2}},\quad \frac{y^{2}-1}{4 y^{3}}$ 232 | \item $\frac{3 b}{5 a},\quad \frac{-2 c}{3 b},\quad \frac{5 a}{-2 c}$ 233 | \end{multicols} 234 | \item $\frac{1}{(a-b)(a-c)},\quad \frac{1}{(b-c)(b-a)}, \quad \frac{1}{(c-a)(c-b)}$ 235 | \item $\frac{x}{x-y},\quad \frac{y}{x+y}, \quad \frac{2}{y^{2}-x^{2}}$ 236 | \item $\frac{a-1}{a+1},\quad -\frac{1+a}{1-a}, \quad \frac{a^{2}+1}{a^{2}-1}$ 237 | \item $\frac{2 y-3}{2 y^{2}-3 y-2}, \quad \frac{y-2}{4 y^{2}+8 y+3}$ 238 | \item $\frac{1}{x^{3}-3 x^{2}+2 x}, \quad \frac{2}{x^{4}-x^{2}}, \quad \frac{-1}{x^{2}+x-2}$ 239 | \item $\frac{1}{x^{3}-6 x^{2}+11 x-6}, \quad \frac{1}{2 x^{3}-7 x^{2}+7 x-2}$ 240 | \end{enumerate} 241 | 242 | \item 约简$F(x)=\frac{6x^2-12x+6}{x^3-3x^2+3x+1}$ 问$x$取何整数值,能使$F(x)$的值是正整数. 243 | \end{enumerate} 244 | \end{ex} 245 | 246 | \subsection{分式的运算} 247 | 分式的四则运算法则和分数的四则运算法则是一样的. 248 | 249 | \subsubsection{分式的加减法} 250 | 251 | 同分母的分式相加(减),只要把分子相加(减)作为分子,分母不变,并把结果化简; 252 | 异分母的分式相加(减),就要先进行通分,再转化为同分母分式的相加(减). 253 | 254 | 列 1. 255 | 测 2 256 | $m_{1}$ 原式 $\quad$ $\begin{aligned}=& \frac{2}{x}-\frac{x-3}{2(x+1)^{2}}+\frac{1}{2(x+1)}-\frac{2(2 x+1)}{x(x+1)^{2}} \\=& \frac{2 \cdot 2(x+1)^{2}}{2 x(x+1)^{2}}-\frac{(x-3) \cdot x}{2 x(x+1)^{2}}+\frac{x(x+1)}{2 x(x+1)^{2}} \\ &-\frac{2(2 x+1) \cdot 2}{2 x(x+1)^{2}} \end{aligned}$ 257 | 258 | 259 | \begin{example} 260 | 计算: $\frac{x+3 y}{x^{2}-y^{2}}-\frac{x+2 y}{x^{2}-y^{2}}+\frac{2x-3y}{x^2-y^2}$ 261 | \end{example} 262 | 263 | 264 | \begin{solution} 265 | \[\begin{split} 266 | \text{原式}&=\frac{x+3y-(x+2y)+(2x-3y)}{x^2-y^2}\\ 267 | &=\frac{2x-2y}{x^2-y^2}=\frac{2(x-y)}{(x+y)(x-y)}\\ 268 | &=\frac{2}{x+y} 269 | \end{split}\] 270 | \end{solution} 271 | 272 | 273 | \begin{example} 274 | 计算: 275 | $\frac{2}{x}-\frac{x-3}{2 x^{2}+4 x+2}+\frac{1}{2 x+2}-\frac{4 x+2}{x(x+1)^{2}}$ 276 | \end{example} 277 | 278 | \begin{solution} 279 | \[\begin{split} 280 | \text{原式}&= \frac{2}{x}-\frac{x-3}{2(x+1)^{2}}+\frac{1}{2(x+1)}-\frac{2(2 x+1)}{x(x+1)^{2}} \\ 281 | &= \frac{2 \cdot 2(x+1)^{2}}{2 x(x+1)^{2}}-\frac{(x-3) \cdot x}{2 x(x+1)^{2}}+\frac{x(x+1)}{2 x(x+1)^{2}}-\frac{2(2 x+1) \cdot 2}{2 x(x+1)^{2}} \\ 282 | &=\frac{4(x+1)^{2}-(x-3) x+x(x+1)-4(2 x+1)}{2 x(x+1)^{2}}\\ 283 | &=\frac{4 x^{2}+4 x}{2 x(x+1)^{2}}=\frac{4 x(x+1)}{2 x(x+1)^{2}}=\frac{2}{x+1} 284 | \end{split}\] 285 | \end{solution} 286 | 287 | \begin{example} 288 | 化简:$\frac{1}{(a-b)(a-c)}+\frac{1}{(b-c)(b-a)}+\frac{1}{(c-a)(c-b)}$ 289 | \end{example} 290 | 291 | \begin{analyze} 292 | $\because \quad a-c=-(c-a),\quad b-a=-(a-b),\quad c-b=-(b-c)$, 293 | 294 | $\therefore\quad (a-c,b-a,c-b)=(a-b)(b-c)(c-a)$. 295 | \end{analyze} 296 | 297 | \begin{solution} 298 | \[\begin{split} 299 | \text{原式}&=-\frac{1}{(a-b)(c-a)}-\frac{1}{(b-c)(a-b)}-\frac{1}{(c-a)(b-c)}\\ 300 | &=\frac{-(b-c)-(c-a)-(a-b)}{(a-b)(b-c)(c-a)}\\ 301 | &=\frac{-b+c-c+a-a+b}{(a-b)(b-c)(c-a)}\\ 302 | &=0 303 | \end{split}\] 304 | \end{solution} 305 | 306 | \begin{example} 307 | 计算$\frac{a^3}{a-1}-a^2-a-1$ 308 | \end{example} 309 | 310 | \begin{analyze} 311 | $-a^2-a-1=-(a^2+a+1)$.一个分式和一个整式的代数和,可以把整式$a^2+a+1$当作 312 | $\frac{a^2+a+1}{1}$. 313 | \end{analyze} 314 | 315 | \begin{solution} 316 | \[\begin{split} 317 | \text{原式}&=\frac{a^3}{a-1}-\frac{a^2+a+1}{1} \\ 318 | &= \frac{a^2}{a-1}-\frac{(a^2+a+1)(a-1)}{a-1} \\ 319 | &=\frac{a^3-(a^3-1)}{a-1}\\ 320 | &=\frac{1}{a-1} 321 | \end{split}\] 322 | \end{solution} 323 | 324 | 325 | \begin{example} 326 | 计算:$\frac{x^2-1}{x^4+x^2-2x}+\frac{2x^2+3x-2}{2x^3+x^2+3x-2}$ 327 | \end{example} 328 | 329 | \begin{analyze} 330 | 由余式定理得$x-1$是第一个分式的分子、分母的公因式,将此分式约简: 331 | \[\frac{x^2-1}{x^4+x^2-2x}=\frac{(x+1)(x-1)}{(x-1)(x^3+x^2+2x)}=\frac{x+1}{x^3+x^2+2x}\] 332 | 由余式定理得$2x-1$是第二个分式的分子、分母的公因式,将此分式约简: 333 | \[\frac{2x^2+3x-2}{2x^3+x^2+3x-2}=\frac{(x+2)(2x-1)}{(2x-1)(x^2+x+2)}=\frac{x+2}{x^2+x+2}\] 334 | \end{analyze} 335 | 336 | \begin{solution} 337 | \[\begin{split} 338 | \text{原式}&=\frac{x+1}{x^3+x^2+2x}+ \frac{x+2}{x^2+x+2} \\ 339 | &= \frac{x+1}{x(x^2+x+2)}+ \frac{x(x+2)}{x(x^2+x+2)} \\ 340 | &=\frac{x^2+3x+1}{x(x^2+x+2)} 341 | \end{split}\] 342 | \end{solution} 343 | 344 | \begin{example} 345 | 已知$a+b+c=0$,求证:$\frac{1}{b^2+c^2-a^2}+\frac{1}{c^2+a^2-b^2}+\frac{1}{a^2+b^2-c^2}=0$ 346 | \end{example} 347 | 348 | \begin{analyze} 349 | 利用已知条件$a+b+c=0$,使各个分母化简. 350 | \end{analyze} 351 | 352 | \begin{proof} 353 | 由于: 354 | \[\begin{split} 355 | \frac{1}{b^2+c^2-a^2}&= \frac{1}{b^2+c^2-(b+c)^2}=-\frac{1}{2bc} \\ 356 | \frac{1}{c^2+a^2-b^2}&= \frac{1}{c^2+a^2-(c+a)^2}=-\frac{1}{2ac} \\ 357 | \frac{1}{a^2+b^2-c^2}&=\frac{1}{a^2+b^2-(a+b)^2}=-\frac{1}{2ab} 358 | \end{split}\] 359 | 因此: 360 | \[\begin{split} 361 | &\quad \frac{1}{b^2+c^2-a^2}+\frac{1}{c^2+a^2-b^2}+\frac{1}{a^2+b^2-c^2}\\ 362 | &=-\frac{1}{2bc}-\frac{1}{2ac}-\frac{1}{2ab}\\ 363 | &=-\frac{a+b+c}{2abc}=0 364 | \end{split}\] 365 | \end{proof} 366 | 367 | \begin{ex} 368 | \begin{enumerate} 369 | \item 计算下列各式的值: 370 | \begin{enumerate} 371 | \item $\frac{a^{2}-2 a+1}{a^{2}+a+1}-\frac{a^{2}+3 a-3}{a^{2}+a+1}+\frac{5 a-4}{a^{2}+a+1}$ 372 | \begin{multicols}{2} 373 | \item $\frac{1}{m^{4} n^{3}}+\frac{2}{m^{3} n^{4}}$ 374 | \item $\frac{5 a}{6 b^{2} c}-\frac{7 b}{12 a c^{2}}+\frac{11 c}{8 a^{2} b}$ 375 | \item $\frac{1}{2 a-b}+\frac{1}{2 a+b}$ 376 | \item $\frac{2 x}{a-b}-\frac{x}{b-a}$ 377 | \item $\frac{2}{x-y}-\frac{x+y}{(y-x)^{2}}$ 378 | \item $a-b+\frac{2 b^{2}}{a+b}$ 379 | \item $\frac{y^{3}}{x-y}+x^{2}+x y+y^{2}$ 380 | \end{multicols} 381 | 382 | \end{enumerate} 383 | 384 | \item 计算下列各式: 385 | \begin{enumerate} 386 | \item $\frac{1}{(x-3)(2-x)}+\frac{1}{(x-2)(3-x)}$ 387 | \item $\frac{1}{x^{2}-3 x+2}+\frac{1}{x^{2}-5 x+6}+\frac{1}{4 x-x^{2}-3}$ 388 | \item $\frac{(a+b)^{2}}{(a-b)(b-c)}+\frac{6 a b}{(b-a)(b-c)}-\frac{a^{2}+b^{2}}{(a-b)(c-b)}$ 389 | \item $\frac{x^{2}-4}{x^{3}-3 x^{2}-x+c}-\frac{3 x^{2}-14 x-5}{3 x^{3}-2 x^{2}-10 x-3}$ 390 | \end{enumerate} 391 | \end{enumerate} 392 | \end{ex} 393 | 394 | \subsubsection{分式的乘法} 395 | 396 | 两个分式相乘时,分子的乘积作为积的分子,分母的乘积作为积的分母,再把结果化简. 397 | 即: 398 | \[\frac{f(x)}{g(x)}\cdot \frac{h(x)}{q(x)}=\frac{f(x)\cdot h(x)}{g(x)\cdot q(x)} \] 399 | 400 | 401 | \begin{example} 402 | 计算:$\frac{a^{2}-b^{2}}{a^{2}+a b+b^{2}} \times \frac{a-b}{a^{3}+b^{3}}$ 403 | \end{example} 404 | 405 | 406 | \begin{solution} 407 | \[\begin{split} 408 | \text{原式}&=\frac{\left(a^{2}-b^{2}\right)(a-b)}{\left(a^{2}+a b+b^{2}\right)\left(a^{3}+b^{3}\right)} \\ 409 | &=\frac{(a+b)(a-b)^{2}}{\left(a^{2}+a b+b^{2}\right)(a+b)\left(a^{2}-a b+b^{2}\right)}\\ 410 | &=\frac{(a-b)^{2}}{a^{4}+a^{2} b^{2}+b^{4}}\\ 411 | \end{split}\] 412 | \end{solution} 413 | 414 | \begin{example} 415 | 计算:$\left(x y^{2}-2 x y+x\right) \cdot \frac{y^{3}+1}{y^{3}-y}$ 416 | \end{example} 417 | 418 | \begin{solution} 419 | \[\begin{split} 420 | \text{原式}&=\frac{x(y^2-2y+1)}{1}\cdot \frac{y^3+1}{y(y^2-1)} \\ 421 | &= \frac{x(y-1)^2 (y+1)(y^2-y+1)}{y(y+1)(y-1)} \\ 422 | &= \frac{x(y-1)(y^2-y+1)}{y} \\ 423 | \end{split}\] 424 | 425 | \end{solution} 426 | 427 | \begin{example} 428 | 计算:$\left(\frac{x^{2}+x+1}{x^{2}-2 x+1}-\frac{x^{3}+1}{(x-1)^{3}}\right)\cdot \left(x^{2}-2 x+1\right)$ 429 | \end{example} 430 | 431 | 432 | \begin{solution} 433 | \[\begin{split} 434 | \text{原式}&=\left(\frac{x^{2}+x+1}{(x-1)^2}-\frac{x^{3}+1}{(x-1)^{3}}\right)\cdot (x-1)^2 \\ 435 | &= \frac{(x^2+x+1)(x-1)-(x^3+1)}{(x-1)^3} \cdot (x-1)^2 \\ 436 | &=\frac{(x^3-1)-(x^3+1)}{x-1}=-\frac{2}{x-1} 437 | \end{split}\] 438 | \end{solution} 439 | 440 | \subsubsection{分式的除法} 441 | 两个分式相除时,把除式的分子、分母颠倒后与被除式相乘即可. 442 | 即: 443 | \[ \frac{f(x)}{g(x)}\div \frac{h(x)}{q(x)}=\frac{f(x)}{g(x)}\x \frac{q(x)}{h(x)}=\frac{f(x)\cdot q(x)}{g(x)\cdot h(x)} \] 444 | 445 | \begin{example} 446 | 计算:$\frac{x^{2}-1}{x^{2}+1} \div \frac{x^{2}-1}{x^{4}-1}$ 447 | \end{example} 448 | 449 | \begin{solution} 450 | \[\begin{split} 451 | \text{原式}&=\frac{x^{2}-1}{x^{2}+1} \times \frac{x^{4}-1}{x^{2}-1} \\ 452 | &= \frac{\left(x^{2}-1\right)\left(x^{2}+1\right)\left(x^{2}-1\right)}{\left(x^{2}+1\right)\left(x^{2}-1\right)} \\ &=x^{2}-1 453 | \end{split}\] 454 | \end{solution} 455 | 456 | \begin{example} 457 | 计算:$\frac{x^{3}-y^{3}}{x^{2}+y^{2}}\div (x-y)$ 458 | \end{example} 459 | 460 | \begin{solution} 461 | \[\begin{split} 462 | \text{原式}&=\frac{x^{3}-y^{3}}{x^{2}+y^{2}} \times \frac{1}{x-y}\\ 463 | &=\frac{(x-y)\left(x^{2}+x y+y^{2}\right)}{x^{2}+y^{2}} \times \frac{1}{x-y}\\ 464 | &=\frac{x^{2}+x y+y^{2}}{x^{2}+y^{2}} 465 | \end{split}\] 466 | \end{solution} 467 | 468 | \begin{example} 469 | 计算 $\left(\frac{1}{a}+\frac{1}{b}\right) \div\left(\frac{1}{a}-\frac{1}{b}\right)$ 470 | \end{example} 471 | 472 | \begin{solution} 473 | \[\begin{split} 474 | \text{原式}&=\frac{b+a}{ab}\div \frac{b-a}{ab} \\ 475 | &= \frac{b+a}{ab} \x \frac{ab}{b-a} \\ 476 | &=\frac{b+a}{b-a} 477 | \end{split}\] 478 | \end{solution} 479 | 480 | 481 | \begin{example} 482 | 化简: 483 | $\frac{ \frac{2(1-x)}{1+x}+\frac{(1-x)^2}{(1+x)^2} +1 }{\frac{2(1+x)}{1-x}+\left(\frac{1+x}{1-x}\right)^2+1}$ 484 | \end{example} 485 | 486 | \begin{note} 487 | 这是一个分子、分母都是分式的繁分式,实际上就是两个分式相除.可以先把它们分别化简后,再进行除法运算. 488 | \end{note} 489 | 490 | 491 | \begin{solution} 492 | \[\begin{split} 493 | \text{原式}&= \frac{ \frac{2(1-x)(1+x)+(1-x)^{2}+(1+x)^{2}}{(1+x)^{2}} }{\frac{2(1+x)(1-x)+(1+x)^{2}+(1-x)^{2}}{(1-x)^{2}}} \\ 494 | &= \frac{[(1+x)+(1-x)]^{2}}{(1+x)^{2}}\div \frac{[(1+x)+(1-x)]^{2}}{(1-x)^{2}} \\ 495 | &= \frac{[(1+x)+(1-x)]^{2}}{(1+x)^{2}}\x \frac{(1-x)^{2}}{[(1+x)+(1-x)]^{2}}\\ 496 | &=\frac{(1-x)^2}{(1+x)^2}=\left(\frac{1-x}{1+x}\right)^2 497 | \end{split}\] 498 | \end{solution} 499 | 500 | \begin{ex} 501 | \begin{enumerate} 502 | \item 计算下列各式 503 | \begin{enumerate} 504 | \begin{multicols}{2} 505 | \item $\frac{3 a b}{4 x y}\div \frac{10 x^{2} y}{21 a^{2} b}$ 506 | \item $8 a^{2} b^{4} \cdot\left(-\frac{3 a}{4 b^{3}}\right)$ 507 | \item $\frac{(a-b)^{2}}{a b} \div(a-b)$, 508 | \item $\left(\frac{3 a^{2} b}{-2 c^{3}}\right)^{3}$ 509 | \item $\frac{a^{2}-x^{2}}{4 a x} \div \frac{x-a}{8 x}$ 510 | \item $\frac{x^{2}-x}{x-3} \div \frac{x^{2}-x^{3}}{3-x}$ 511 | \end{multicols} 512 | \item $\left(x^{2}-6 x+9\right) \div \frac{x^{2}-9 x+18}{x+3}$ 513 | \item $\frac{a+x}{(m+n)^{2}} \cdot \frac{x^{2}-y^{2}}{12} \cdot \frac{m+n}{n-m} \cdot \frac{6\left(m^{2}-n^{2}\right)}{x+y}$ 514 | \end{enumerate} 515 | \item 化简: 516 | \begin{multicols}{2} 517 | \begin{enumerate} 518 | \item $\frac{\frac{c}{b}}{a}$ 519 | \item $\frac{c}{\frac{b}{a}}$ 520 | \item $\frac{a-\frac{1}{a}}{a-1}$ 521 | \item $\frac{P+2-\frac{1}{P+2}}{P+2+\frac{1}{P+2}}$ 522 | \end{enumerate} 523 | \end{multicols} 524 | \item 化简: $G(a)=\frac{1-\frac{4 a+1}{a+1}}{a}$, 问$a$取何整数值时, $G(a)$ 等于 正整数. 525 | \end{enumerate} 526 | \end{ex} 527 | 528 | \subsection{分式方程} 529 | 如果方程式中含有分式,那么这样的方程,叫做 530 | \textbf{分式方程},例如$\frac{2}{x}=1$, $y+1+\frac{2}{y}=\frac{y^2}{y-1}$, $\frac{5}{x-1}=\frac{1}{x+3}$ 531 | 等,都是分式方程.如果$x$是未知数,$a$ 532 | 表示一个非零常数,那么$\frac{x}{a}+x=1$,就不是分式方程. 533 | 534 | 解分式方程主要是设法把原方程变形为整式方程,也就是在方程两边乘以同一个含有未知数的整式.这个整式一般是分母的最低公倍式. 535 | 536 | 537 | 538 | \begin{example} 539 | 解方程$\frac{5}{x-1}=\frac{1}{x+3}$ 540 | \end{example} 541 | 542 | \begin{solution} 543 | 两边乘以分母的最低公倍式:$(x-1)(x+3)$, 并约简得: 544 | \[5(x+3)=x-1\] 545 | 546 | 解整式方程:$4x=-16\quad \Rightarrow\quad x=-4$ 547 | 548 | 验根:把$x=-4$分别代入原方程两边. 549 | \[\begin{split} 550 | \text{左式}&=\frac{5}{-4-1}=-1\\ 551 | \text{右式}&=\frac{1}{-4+3}=-1 552 | \end{split}\] 553 | 554 | $\because\quad \text{左}=\text{右}$ 555 | 556 | $\therefore\quad $原方程的解集是:$\{-4\}$. 557 | \end{solution} 558 | 559 | 560 | \begin{example} 561 | 解方程:$\frac{1}{x+2}+\frac{4x}{x^2-4}=1+\frac{2}{x-2}$ 562 | \end{example} 563 | 564 | \begin{solution} 565 | 原方程就是:$\frac{1}{x+2}+\frac{4x}{(x+2)(x-2)}=1+\frac{2}{x-2}$ 566 | 567 | 两边乘以分母的最低公倍式$(x+2)(x-2)$, 并约简得: 568 | \[(x-2)+4x=(x+2)(x-2)+2(x+2)\] 569 | 解整式方程 $x^2-3x+2=0,\qquad \therefore\quad x_1=1,\;x_2=2$ 570 | 571 | 验根:把$x=1$代入原方程两边: 572 | \[\begin{split} 573 | \text{右式}&=\frac{1}{1+2}+\frac{4}{1-4}=\frac{1}{3}-\frac{4}{3}=-1\\ 574 | \text{左式}&=1+\frac{2}{1-2}=1-2=-1 575 | \end{split}\] 576 | $\therefore\quad x=1$是原方程的根. 577 | 578 | 把$x=2$代入原方程时,由于分母$x-2=0$, $x^2-4=0$, 579 | 就是说:当$x=2$时原方程没有意义,所以 $x=2$不是原方程的根,应舍去它. 580 | 581 | 因此:原方程的解集是$\{1\}$. 582 | \end{solution} 583 | 584 | 从以上两例可以看出:分式方程的两边乘以同一个含有未知数的整式,并进行约简,就得到一个新的整式方程.这个整式方程的根,可能是原分式方程的根,也可能不是原分式方程的根.而这里不适合原方程的根,就叫做原方程的\textbf{增根},验根后应该舍去(例如,在例6.22中的$x=2$就是增根). 585 | 586 | 我们不禁要问:解分式方程的过程中,为什么可能增根呢? 587 | 588 | 先观察例6.22,原分式方程未知数$x$的可取值范围是$x\ne \pm2$的一切实数,整式方程$x^2-3x+2=0$的$x$可取值范围扩大为一切实数,这样解整式方程得到的根$x_1=1$, 恰好在原方程$x$的可取值范围内,所以适合原方程,是原方程的根.而另一根$x_2=2$, 恰好在原方程$x$可取值范围外,所以不适合原方程,是原方程的增根. 589 | 590 | 再观察例6.21,原分式方程未知数$x$的可取值范围是$x\ne 1$且$x\ne -3$的一切实数,整式方程$5(x+3)=x-1$的$x$可取值范围扩大为一切实数,但这个整式方程的根$x=-4$, 恰好在原分式方程的可取值范围内,所以是原方程的根. 591 | 592 | 解分式方程过程中,由于原方程两边乘以含有未知数的整式,约简而得到一个整式方程.这样就扩大了未知数的可取值范围,自然就有产生增根的可能.但是,增根并不可怕,只要通过检验,就可以鉴别出来把它舍去.所以,解分式方程是必须进行验根的. 593 | 594 | 仔细观察、分析,不难发现:分式方程的增根,都正好是“使原方程中的一些分母的值为零”的未知数值.因此,解分式方程时,比较简捷的验根的方法是:把整式方程的根,逐个代入分母的最低公倍式中,如果其值不等于零,则是原方程的根;如果其值等 595 | 于零,则它是原方程的增根,要舍去. 596 | 597 | 598 | \begin{example} 599 | 试求一个正实数$x$满足下述条件:$x=\frac{1}{x-1}$ 600 | \end{example} 601 | 602 | \begin{solution} 603 | 方程两边乘以$x-1$,并约简得$x(x-1)=1$. 604 | 605 | 解整式方程:$x^2-x-1=0$, 606 | 607 | $\therefore\quad x_1=\frac{1}{2}+\frac{\sqrt{5}}{2},\qquad x_2=\frac{1}{2}-\frac{\sqrt{5}}{2}$. 608 | 609 | 验根:把$x_1=\frac{1}{2}+\frac{\sqrt{5}}{2}$代入$x-1$,其值不等于零.把$x_2=\frac{1}{2}-\frac{\sqrt{5}}{2}$代入$x-1$,其值不等于零. 610 | 611 | $\therefore\quad $原方程的解集是:$\left\{\frac{1}{2}+\frac{\sqrt{5}}{2},\; \frac{1}{2}-\frac{\sqrt{5}}{2}\right\}$ 612 | 613 | 但$\because\quad \frac{1}{2}-\frac{\sqrt{5}}{2}<0$,不合题意应舍去. 614 | 615 | $\therefore\quad $所求正实数是:$\frac{1}{2}+\frac{\sqrt{5}}{2}$. 616 | \end{solution} 617 | 618 | 综合以上各例,可以概括出解分式方程的一般步骤是: 619 | \begin{enumerate} 620 | \item 方程两边乘以分母的最低公倍式,并约简变 621 | 形为整式方程. 622 | \item 解整式方程. 623 | \item 验根:把整式方程的根分别代入原方程分母的最低公倍式中去.如果其值不等于零,则是原方程的根;如果其值等于零,则是原方程的增根,要舍去. 624 | \end{enumerate} 625 | 626 | \begin{ex} 627 | 解下列方程: 628 | \begin{multicols}{2} 629 | \begin{enumerate} 630 | \item $\frac{5}{y}=\frac{3}{y-2}$ 631 | \item $1-\frac{1}{x-4}=\frac{5-x}{x-4}$ 632 | \item $1+\frac{1}{x-4}=\frac{5-x}{x-4}$ 633 | \item $\frac{2}{1-x^2}=\frac{1}{1+x}+1$ 634 | \end{enumerate} 635 | \end{multicols} 636 | 637 | \end{ex} 638 | 639 | \begin{example} 640 | 解方程:$\frac{2}{1+x}-\frac{3}{1-x}=\frac{6}{x^2-1}$ 641 | \end{example} 642 | 643 | \begin{solution} 644 | 原方程就是$\frac{2}{x+1}+\frac{3}{x-1}=\frac{6}{(x+1)(x-1)}$ 645 | 646 | 方程两边乘以$(x+1)\cdot (x-1)$,并约简得: 647 | \[2(x-1)+3(x+1)=6\] 648 | 解整式方程 649 | \[\begin{split} 650 | 2x-2+3x+3&=6\\ 651 | 5x&=5\\ 652 | x&=1 653 | \end{split}\] 654 | 655 | 验根:把$x=1$代入$(x+1)\cdot (x-1)$所得的值等于零. 656 | 657 | $\therefore\quad x=1$是增根(舍去), 658 | 659 | $\therefore\quad$原方程的解集是空集$\emptyset$. 660 | \end{solution} 661 | 662 | 663 | 664 | \begin{example} 665 | 解方程$\frac{3}{x-2}-\frac{4}{x-1}=\frac{1}{x-4}-\frac{2}{x-3}$ 666 | \end{example} 667 | 668 | \begin{analyze} 669 | 如果开始就乘以分母的最低公倍式,这样很复杂,所以先采取方程两边分别通分,这样比较简便. 670 | \end{analyze} 671 | 672 | \begin{solution} 673 | 方程两边分别通分得: 674 | \[\begin{split} 675 | \frac{3x-3-4x+8}{(x-1)(x-2)}&=\frac{x-3-2x+8}{(x-3)(x-4)}\\ 676 | \frac{-x+5}{(x-1)(x-2)}&=\frac{-x+5}{(x-3)(x-4)} 677 | \end{split}\] 678 | 679 | 方程两边乘以$(x-1)(x-2)(x-3)(x-4)$,得: 680 | \begin{equation} 681 | (-x+5)(x-3)(x-4)=(-x+5)(x-1)(x-2) 682 | \end{equation} 683 | 即: 684 | \[(-x+5)[(x^2-7x+12)-(x^2-3x+2)]=0\] 685 | 686 | 解整式方程 $(-x+5)(-4x+10)=0$ 687 | 688 | $\therefore\quad x_1=5,\quad x_2=\frac{5}{2}$ 689 | 690 | 验根:把$x_1=5,\quad x_2=\frac{5}{2}$分别代入分母的最低公倍式中,很明显其值都不等于零. 691 | 692 | $\therefore\quad $ 原方程的解集是$\left\{5,\frac{5}{2}\right\}$. 693 | \end{solution} 694 | 695 | \begin{rmk} 696 | 如果在方程(6.1)的两边除以$-x+5$, 那么就会丢失$x=5$这一个根,所以在解方程的过程中,如果方程两边除以含有未知数的整式,那末原方程就有丢根的可能,丢根是不易找回来的,因此在解方程的过程中,要尽量避免进行这种变形. 697 | \end{rmk} 698 | 699 | \begin{ex} 700 | 解下列方程: 701 | \begin{multicols}{2} 702 | \begin{enumerate} 703 | \item $x=\frac{1}{x-1}$ 704 | \item $\frac{10x}{2x-1}+\frac{5}{1-2x}=2$ 705 | \item $\frac{1}{1-y}+\frac{3y-y^2}{y^2-1}=0$ 706 | \item $\frac{1}{t+2}+\frac{1}{t+7}=\frac{1}{t+3}+\frac{1}{t+6}$ 707 | \end{enumerate} 708 | \end{multicols} 709 | \end{ex} 710 | 711 | 在解有些分式方程的过程中,如果利用换元法,引进一个辅助未知数,那末,就可以得到一个容易解的方程,使解法简化. 712 | 713 | 714 | \begin{example} 715 | 解方程 $\frac{(x-1)^2}{x}+\frac{x}{(x-1)^2}=2$ 716 | \end{example} 717 | 718 | \begin{solution} 719 | 设$\frac{(x-1)^2}{x}=y$,则$\frac{x}{(x-1)^2}=\frac{1}{y}$ 720 | 721 | 代入原方程就是 $y+\frac{1}{y}=2$,两边乘以$y$,并约简得 722 | \[y^2+1=2y\quad \Rightarrow\quad y^2-2y+1=0\] 723 | 解这个方程,得:$y=1$. 724 | 725 | 把$y=1$代入 $\frac{(x-1)^2}{x}=y$,得: 726 | \begin{equation} 727 | \frac{(x-1)^2}{x}=1 728 | \end{equation} 729 | 两边乘以$x$,并约简得: 730 | \begin{equation} 731 | (x-1)^2=x 732 | \end{equation} 733 | 即:$x^2-3x+1=0$ 734 | 735 | $\therefore\quad x_1=\frac{3}{2}+\frac{\sqrt{5}}{2},\qquad x_2=\frac{3}{2}-\frac{\sqrt{5}}{2}$. 736 | 737 | 把$x_1,x_2$分别代入方程(6.2)的分母中,其值不等于零. 738 | 739 | $\therefore\quad $原方程的解集是$\left\{\frac{3}{2}+\frac{\sqrt{5}}{2},\; \frac{3}{2}-\frac{\sqrt{5}}{2}\right\}$ 740 | 741 | \end{solution} 742 | 743 | \begin{note} 744 | 解上例的过程中,由方程(6.2)到方程(6.3)有产生增根的可能,所以只要把$x_1,x_2$代入方程(6.2)验根就可以. 745 | \end{note} 746 | 747 | 748 | 749 | \begin{example} 750 | 解方程 751 | $\frac{1}{a}+\frac{a}{x}=\frac{1}{b}+\frac{b}{x}\quad (a\ne b)$ 752 | \end{example} 753 | 754 | \begin{solution} 755 | 方程两边乘以$abx$得: 756 | \[bx+a^2b=ax+ab^2\] 757 | 解整式方程 $(b-a)x=ab(b-a)$ 758 | 759 | $\because\quad a\ne b,\qquad \therefore\quad b-a\ne 0$ 760 | 761 | $\therefore\quad x=ab$. 762 | 763 | 验根:把$x=ab$代入$abx$得$a^2b^2$. 764 | 765 | $\because\quad a\ne 0,\; b\ne 0$ (如果$a=0$, $b=0$, 那么原方程 766 | 无意义). 767 | 768 | $\therefore\quad a^2b^2\ne 0$ 769 | 770 | $\therefore\quad $原方程的解集是$\{ab\}$. 771 | \end{solution} 772 | 773 | \begin{ex} 774 | 解下列方程: 775 | \begin{enumerate} 776 | \item $\frac{a+b}{x}-\frac{a}{b}=1\quad (a+b\ne 0)$ 777 | \item $\frac{1}{x}-\frac{1}{a}-\frac{1}{b}=\frac{1}{x-a-b}\quad (a+b\ne 0)$ 778 | \item 在$\frac{1}{R}=\frac{1}{r}-\frac{1}{r-r_1}$中,已知$R,r_1$,求$r$.(其中各字母表示正数,$r_1>4R$) 779 | \end{enumerate} 780 | \end{ex} 781 | 782 | \begin{example} 783 | 某公社原计划要在一定的日期里开垦荒地 784 | 960亩,如果实际每天比原计划多开垦40亩,可提前 785 | 4天完成原计划.求原计划一天开垦荒地的亩数和完 786 | 成的天数. 787 | \end{example} 788 | 789 | \begin{analyze} 790 | 这个应用题中的数量关系,可列表如下: 791 | \begin{center} 792 | \begin{tabular}{cccc} 793 | \hline 794 | & 工作总量&一天的工作量&所需天数\\ 795 | \hline 796 | 原计划工作情况&960亩& $x$亩 & $\tfrac{960}{x}$\\ 797 | 实际工作情况&960亩&$(x+40)$亩 & $\tfrac{960}{x+40}$\\ 798 | \hline 799 | \end{tabular} 800 | \end{center} 801 | \[\text{原计划需要的天数}=\text{实际需要天数}+4\text{(天)}\] 802 | \end{analyze} 803 | 804 | 805 | \begin{solution} 806 | 设原计划每天开垦荒地$x$亩,则原计划需要 807 | $\frac{960}{x}$(天)完成,实际每天开垦荒地$(x+40)$亩,实际需要 808 | $\frac{960}{x+40}$ 809 | (天) 810 | 811 | 按题意得:$\frac{960}{x}=\frac{960}{x+40}+4$ 812 | 813 | 两边乘以 814 | $x(x+40)$, 得: 815 | \[960(x+40)=960x+4x(x+40)\] 816 | 整理得:$x^2+40x-9600=0$ 817 | 818 | $\therefore\quad x_1=80,\quad x_2=-120$ 819 | 820 | 检验:$x_1=80$是原方程的根. $x_2=-120$是原方程的根,但不合题意,应舍去. 821 | 822 | 又$\frac{960}{x}=\frac{960}{80}=12$(天) 823 | 824 | 答:原计划每天开垦荒地80亩,需要12天. 825 | \end{solution} 826 | 827 | 828 | \begin{example} 829 | $A$、$B$两地相距87公里,甲骑自行车从$A$ 830 | 出发向$B$驶去,经过30分钟后,乙骑自行车由$B$出发, 831 | 用每小时比甲快4公里的速度向$A$驶来,两人在距离 832 | $B$45公里的$C$处相遇,求各人的速度. 833 | \end{example} 834 | 835 | \begin{analyze} 836 | \begin{center} 837 | \begin{tikzpicture}[>=latex] 838 | \draw (0,0)node[below]{$A$}--(4,0)node[below]{$C$}--(8.2,0)node[below]{$B$}; 839 | \foreach \x in {0,4,8.2} 840 | { 841 | \draw (\x,0)--(\x,.75); 842 | } 843 | \draw[<->] (0,.35)--node[fill=white]{$(87-45)$公里}(4,.35); 844 | \draw[<->](4,.35)--node[fill=white]{45公里}(8.2,.35); 845 | \end{tikzpicture} 846 | \end{center} 847 | \begin{center} 848 | \begin{tabular}{cccc} 849 | \hline 850 | & 所行距离 & 速度 & 时间\\ 851 | \hline 852 | 甲& $(87-45)$公里& $x$公里/小时 & $\tfrac{87-45}{x}$ \\ 853 | 乙& 45 公里& $(x+4)$公里/小时& $\tfrac{45}{x+4}$ \\ 854 | \hline 855 | \end{tabular} 856 | \end{center} 857 | 858 | \[\text{甲自$A$到$C$所需要时间}=\text{乙由$B$到$C$所需 859 | 要时间}+\frac{30}{60}\text{小时}\] 860 | \end{analyze} 861 | 862 | \begin{solution} 863 | 设甲每小时行$x$公里,则乙每小时行$(x+4)$ 864 | 公里,按题意: 865 | \[\frac{87-45}{x}=\frac{45}{x+4}+\frac{30}{60} \] 866 | 两边乘以$2x(x+4)$得: 867 | \[\begin{split} 868 | 2\x 42(x+4) &=2\x 45x+x(x+4)\\ 869 | x^2+10x-336&=0 870 | \end{split}\] 871 | $\therefore\quad x_1=14,\qquad x_2=-24$ 872 | 873 | 检验:$x=14$是原方程根, 874 | $x=-24$是原方程根,但不合题意,舍去. 875 | \[x+4=14+4=18\] 876 | 877 | 答:甲每小时行14公里,乙每小时行18公里. 878 | \end{solution} 879 | 880 | \begin{example} 881 | 甲乙两个工程队合做一项工程,两队合做 882 | 两天后,由乙队单独做1天就完成了全部工程.已知 883 | 乙队单独做所需的天数是甲队单独做所需天数的$1\frac{1}{2}$ 884 | 倍.求甲、乙两队单独做各需多少天? 885 | \end{example} 886 | 887 | \begin{solution} 888 | 设甲队独做$x$天完成,乙队独做$\frac{3}{2}x$ 889 | 天完成,则甲每天工作量是$\frac{1}{x}$, 890 | 乙每天工作量是$\frac{1}{\tfrac{3}{2}x}$, 891 | 甲、乙两队合做一天的工作量是$\frac{1}{x}+\frac{1}{\tfrac{3}{2}x}$; 892 | 合做两天的工作量是$2\left(\frac{1}{x}+\frac{1}{\tfrac{3}{2}x}\right)$. 893 | 894 | 按题意得: 895 | \[2\left(\frac{1}{x}+\frac{1}{\tfrac{3}{2}x}\right)+\frac{1}{\tfrac{3}{2}x}=1\] 896 | 就是 $\frac{2}{x}+\frac{4}{3x}+\frac{2}{3x}=1$ 897 | 898 | 方程两边乘以$3x$得: 899 | \[\begin{split} 900 | 6+4+2&=3x\\ 901 | 3x&=12\\ 902 | x&=4 903 | \end{split}\] 904 | 905 | 经检验,$x=4$是原方程的根.又$\frac{3}{2}x=6$. 906 | 907 | 答:甲独做4天完成任务,乙独做6天完成任 908 | 务. 909 | \end{solution} 910 | 911 | \begin{ex} 912 | \begin{enumerate} 913 | \item 甲、乙两个车工,各车1500个螺丝.乙改进了操作方 914 | 法,生产效率提高到甲的3倍,因此比甲少用20个工时 915 | 完成任务.他们每小时各车多少个螺丝? 916 | \item 甲、乙两个车站相距96公里,快车和慢车同时从甲站开 917 | 出,1小时后,快车在慢车前12公里,快车比慢车早40 918 | 分钟到达乙站.快车和慢车的速度各是多少? 919 | \item 甲、乙、丙三人合做一件工作12天完成,已知甲1天完 920 | 成的工作,乙须要2天,两须要3天,问三人单独完成. 921 | 这件工作,各需要多少天? 922 | \end{enumerate} 923 | \end{ex} 924 | 925 | \section*{习题6.1} 926 | \addcontentsline{toc}{subsection}{习题6.1} 927 | \begin{enumerate} 928 | \item 下列各分式在什么条件下无意义: 929 | \begin{multicols}{2} 930 | \begin{enumerate} 931 | \item $\frac{1}{2x-3}$ 932 | \item $\frac{x-y}{x+y}$ 933 | \item $\frac{x}{x^4+1}$ 934 | \item $\frac{2x-1}{x^2-2}$ 935 | \end{enumerate} 936 | \end{multicols} 937 | \item 当$x$取何值时,分式$\frac{x-2}{(1-x)(x+3)}$ 938 | \begin{enumerate} 939 | \item 没有意义; 940 | \item 分式值等于0; 941 | \item 分式值等于1. 942 | \end{enumerate} 943 | \item 分式$\frac{a+3}{a-4}$和$\frac{(a+3)(a-3)}{(a-4)(a-3)}$ 944 | 的值是不是永远相等? 945 | \item 先化简下列各式,再求它的值. 946 | \begin{enumerate} 947 | \item $\frac{3 a^{2}-a b}{8 a^{2}-6 a b+b^{2}}$, 其中 $a=- \frac{2}{3}, \quad b=\frac{1}{2}$ 948 | \item $\frac{75(x-2 y)^{3}(2 x-y)^{2}}{15(y-2 x)^{2}(2 y-x)}$, 其中 $x=4.5,\quad -y=1.7$ 949 | \end{enumerate} 950 | 951 | \item 计算下列各式: 952 | \begin{enumerate} 953 | \item $\frac{a}{a^{2}-1}+\frac{a^{2}+ a-1}{a^{3}-a^{2}+a-1}+\frac{a^{2}-a-1}{a^{3}+a^{2}+a+1}-\frac{2 a^{3}}{a^{4}-1}$ 954 | \item $a-\frac{a^{2}-b^{2}}{a}+\frac{a^{2}+b^{2}}{a}-b$ 955 | \item $\frac{1}{x-2}+\frac{2}{x+1}-\frac{2}{x-1}-\frac{1}{x+2}$ 956 | \item $\frac{1}{1-x}+\frac{1}{1+x}+\frac{2}{1+x^{2}}+\frac{4}{1+x^{4}}$ 957 | \item $\frac{x^{2}+7 x+12}{x^{2}-8 x+15} \div \frac{x^{2}+3 x-4}{x^{2}-5 x+6} \div \frac{x^{2}+x-6}{x^{2}-4 x-5}$ 958 | \item $\frac{(a-b)^{2}}{a^{2}-a b+b^{2}} \cdot \frac{a^{3}+b^{3}}{(a-b) x^{2}} \div \frac{b^{2}-a^{2}}{x^{2}}$ 959 | \item $\frac{x}{x-1}+\frac{x}{x+1}-\frac{x+1}{x^{3}-1} \div \frac{1}{x^{2}+x+1}$ 960 | \item $\left(x-1+\frac{1}{x}\right)\div \frac{x^{2}-x+1}{x}$ 961 | \item $\left(1+\frac{a}{x}+\frac{a^2}{x^2}\right)\left(1-\frac{a}{x}\right)-\frac{2x^3-a^3}{x^3}$ 962 | \item $\left(\frac{x}{y}-\frac{y}{x}\right)\div \left(\frac{x}{y}+\frac{y}{x}-2\right)\div \frac{x}{x-y}$ 963 | \end{enumerate} 964 | 965 | \item 化简 $F(x)=-\frac{1}{1-\frac{1+x}{x-\frac{1}{x}}}$ 966 | ,又$x$取何值能使 $F(x)$的值等于2?$F(x)$的值能等于1吗?为什么? 967 | \item 解下列各方程: 968 | \begin{enumerate} 969 | \item $\frac{1-3 x}{1+3 x}+\frac{3 x+1}{3 x-1}=\frac{12}{1-9 x^{2}}$ 970 | \item $\frac{7}{x^{2}+x}-\frac{1}{x-x^{2}}=\frac{6}{x^{2}-1}$ 971 | \item $5+\frac{96}{x^{2}-16}=\frac{2 x-1}{x+4}-\frac{3 x-1}{4-x}$ 972 | \item $\frac{1}{x+2}-\frac{1}{x+4}=\frac{1}{x+3}-\frac{1}{x+1}$ 973 | \item $\left(x+\frac{1}{x}\right)^{2}-\frac{9}{2}\left(x+\frac{1}{x}\right)+5=0$ 974 | \item $\frac{x^{2}+3 x+1}{4 x^{2}+6 x-1}-\frac{3\left(4 x^{2}+6 x-1\right)}{x^{2}+3 x+1}-2=0$ 975 | \end{enumerate} 976 | 977 | \item 解下列关于 $x$ 的方程: 978 | \begin{enumerate} 979 | \item $x+\frac{1}{x}=a+\frac{1}{a}$ 980 | \item $\frac{1}{b+x}=\frac{3b}{2x^2}-\frac{1}{x}$ 981 | \item $\frac{x+m}{x-n}+\frac{x+n}{x-m}=2\quad (n+m\ne 0)$ 982 | \item $\frac{2x}{x+b}+\frac{x}{x-b}=\frac{b^2}{4x^2-4b^2}$ 983 | \end{enumerate} 984 | 985 | \item 解下列应用题: 986 | \begin{enumerate} 987 | \item 甲组人数比乙组人数多10人,甲、乙两组人数的 988 | 比是$\frac{5}{4}$, 989 | 求两组人数. 990 | \item 甲做90个机器零件所用的时间和乙做120个机器 991 | 零件所用的时间相同,已知两人每小时一共做35 992 | 个机器零件,两人每小时各做多少个? 993 | \item 马车后轮周长比前轮周长大20厘米,行了2500米 994 | 时,前轮比后轮多转了60转.求前后轮的周长各 995 | 等于多少米?(精确至0.01米) 996 | \item 一辆汽车原定在若干小时内以某一定的速度到达 997 | 相距300里的目的地,如果每小时加快10里,那 998 | 么可以早到$1\frac{1}{2}$ 999 | 小时.求原定的速度. 1000 | \item 甲组的工作效率比乙组高25\%, 因此甲组加工 1001 | 2000个零件所用的时间比乙组加工1800个零件所 1002 | 用的时间还少30分钟.甲,乙两组每小时各能加 1003 | 工多少个零件? 1004 | \item 某工厂有一个水池,上面装有甲、乙两个水管, 1005 | 如果把两个水管都打开,1小时20分就可以把水池 1006 | 注满,若打开甲管10分钟和打开乙管12分钟,就 1007 | 可以注满水池的$\frac{2}{15}$, 1008 | 求单独一个水管注满水池各 1009 | 需多少时间? 1010 | \item 汽船顺流、逆流各走48公里,共经5小时.如果 1011 | 水流速度每小时4公里,求汽船在静水中的速度. 1012 | \item 一架飞机顺风飞行1380公里和逆风飞行1020公里 1013 | 所需的时间相等,已知这架飞机的速度是每小时 1014 | 300公里.求风的速度. 1015 | \end{enumerate} 1016 | \end{enumerate} 1017 | 1018 | \section{二次根式与根式方程} 1019 | 我们已经学习过平方根,也就是如果$x^2=a$, 那 1020 | 么$x$叫做$a$的二次方根,简称平方根.正数的平方根 1021 | 是两个相反的数,记作$\pm\sqrt{a}$; 0的平方根是0; 负 1022 | 数在实数范围内没有平方根. 1023 | 1024 | 正数的正平方根叫做算术平方根;零的算术平方 1025 | 根是零,记作$\sqrt{0}=0$. 1026 | 1027 | \subsection{二次根式和二次根式的变形} 1028 | 表示平方根的式子,叫做\textbf{二次根式}. 1029 | 1030 | 例如:$\sqrt{3},\quad -2\sqrt{7},\quad -\sqrt{x}\; (x\ge 0),\quad \sqrt{b^2+1}$ 1031 | 等都是二次根式. 1032 | 1033 | 由第三章学过的平方根与算术平方根的意义,可 1034 | 以得到二次根式的基本性质: 1035 | \begin{blk}{} 1036 | \begin{enumerate} 1037 | \item $\left(\sqrt{a}\right)^2=a\quad (a\ge 0)$ 1038 | \item $\sqrt{a^2}=|a|=\begin{cases} 1039 | a, & a\ge 0\\ -a, & a<0 1040 | \end{cases}$ 1041 | \item $\sqrt{a\cdot b}=\sqrt{a}\cdot \sqrt{b}\quad (a\ge 0,\; b\ge 0)$ 1042 | \item $\sqrt{\frac{a}{b}}=\frac{\sqrt{a}}{\sqrt{b}}\quad (a\ge 0,\; b>0)$ 1043 | \end{enumerate} 1044 | \end{blk} 1045 | 1046 | 利用二次根式的基本性质,可以进行化简. 1047 | 1048 | 1049 | \begin{example} 1050 | 化简$\sqrt{(x-2)^2}$ 1051 | \end{example} 1052 | 1053 | \begin{solution} 1054 | \[\sqrt{(x-2)^2}=|x-2|=\begin{cases} 1055 | x-2, & x\ge2\\ 2-x, & x<2 1056 | \end{cases}\] 1057 | \end{solution} 1058 | 1059 | 1060 | 1061 | \begin{example} 1062 | 化简$\sqrt{x^2-2xy+y^2}$ 1063 | \end{example} 1064 | 1065 | \begin{solution} 1066 | \[\begin{split} 1067 | \sqrt{x^2-2xy+y^2}&=\sqrt{(x-y)^2}=|x-y|\\ 1068 | &=\begin{cases} 1069 | x-y, & x\ge y\\ 1070 | y-x, & x0,\; b>0)$ 1178 | \end{enumerate} 1179 | \end{multicols} 1180 | 1181 | \end{example} 1182 | 1183 | 1184 | \begin{solution} 1185 | \begin{enumerate} 1186 | \item $\sqrt{x^{5}}=\sqrt{x^{4} \cdot x}=\sqrt{x^{4}} \cdot \sqrt{x}=x^{2} \sqrt{x}$ 1187 | \item $\sqrt{8 a^{3} b^{2}}=\sqrt{4 a^{2} b^{2} \cdot 2 a}=\sqrt{4 a^{2} b^{2}} \cdot \sqrt{2 a}=2 {a} b \sqrt{2 a}$ 1188 | \item $\sqrt{a^{3}+3 a^{2} b+3 ab^{2}+b^{3}}=\sqrt{(a+b)^{3}}=(a+b) \sqrt{a+b}$ 1189 | \item $\sqrt{\frac{c}{a^{2} b^{2}}}=\frac{\sqrt{c}}{\sqrt{a^{2} b^{2}}}=\frac{\sqrt{c}}{a b}=\frac{1}{a b} \sqrt{c}$ 1190 | \end{enumerate} 1191 | \end{solution} 1192 | 1193 | \begin{example} 1194 | 把下列各根号外面的因式移到根号里面去. 1195 | \[2a\sqrt{a},\qquad (m-n)\sqrt{m}\;\; (m-n\ge 0),\qquad -x\sqrt{a}\;\;(x>0) \] 1196 | \end{example} 1197 | 1198 | \begin{solution} 1199 | \begin{enumerate} 1200 | \item $2a\sqrt{a}=\sqrt{(2a)^2\cdot a}=\sqrt{4a^3}$ 1201 | \item $(m-n)\sqrt{m}=\sqrt{(m-n)^2\cdot m}$ 1202 | \item $-x\sqrt{a}=-\sqrt{ax^2}$ 1203 | \end{enumerate} 1204 | \end{solution} 1205 | 1206 | 这里要注意,$-x\sqrt{a}=\sqrt{a(-x)^2}=\sqrt{ax^2}$ 1207 | 是 1208 | 不正确的,因为$-x<0$, 不能进行内移. 1209 | 1210 | \subsubsection{化去根号内的分母} 1211 | 在根号内的分母中,如果有开得尽的因式,就可 1212 | 以用它的算术根代替而移到根号外面;如果有开不尽 1213 | 的因式,就可以利用分式的基本性质,将分子、分母 1214 | 乘以同一个适当的非零代数式,使分母的各因式都能 1215 | 开得尽,从而用其算术根代替,并移到根号外面,使 1216 | 根号里面不含有分母. 1217 | \begin{example} 1218 | 化去根号里面的分母 1219 | \[\sqrt{\frac{3}{50}},\qquad \sqrt{\frac{c}{a^2b}}\quad (a>0,\; b>0),\qquad \sqrt{\frac{(a+b)^2}{a^2-b^2}}\quad (a>b)\] 1220 | \end{example} 1221 | 1222 | \begin{solution} 1223 | \begin{enumerate} 1224 | \item $\sqrt{\frac{3}{50}}=\sqrt{\frac{3\x2}{50\x2}}=\frac{\sqrt{6}}{\sqrt{100}}=\frac{1}{10}\sqrt{6}$ 1225 | \item $\sqrt{\frac{c}{a^2b}}=\frac{1}{a}\sqrt{\frac{bc}{b^2}}=\frac{1}{ab}\sqrt{bc}$ 1226 | \item \[\begin{split} 1227 | \sqrt{\frac{(a+b)^2}{a^2-b^2}} &= \sqrt{\frac{(a+b)^2}{(a+b)(a-b)}}\\ 1228 | &=\sqrt{\frac{a+b}{a-b}}\\ 1229 | &=\sqrt{\frac{(a+b)(a-b)}{(a-b)^2}}\\ 1230 | &=\frac{\sqrt{a^2-b^2}}{\sqrt{(a-b)^2}}\\ 1231 | &=\frac{1}{a-b}\sqrt{a^2-b^2} 1232 | \end{split}\] 1233 | \end{enumerate} 1234 | 1235 | \end{solution} 1236 | 1237 | \begin{ex} 1238 | \begin{enumerate} 1239 | \item 把下列根号内的因式移到根号外面来: 1240 | \begin{multicols}{2}\begin{enumerate} 1241 | \item $\sqrt{27}$ 1242 | \item $\sqrt{98}$ 1243 | \item $\sqrt{0.32}$ 1244 | \item $\sqrt{0.0003}$ 1245 | \item $\sqrt{x^3}$ 1246 | \item $\sqrt{16a}$ 1247 | \item $\sqrt{121a^4}$ 1248 | \item $\sqrt{\frac{x^2y}{16}}\quad (x\ge 0)$ 1249 | \item $\sqrt{x^2+6x+9}\quad (x\ge 3)$ 1250 | \item $\sqrt{\poly{1,6,12,8}}$ 1251 | \end{enumerate} \end{multicols} 1252 | 1253 | \item 把下列各根号外面的因式移到根号里面去: 1254 | \begin{multicols}{2} 1255 | \begin{enumerate} 1256 | \item $5\sqrt{2}$ 1257 | \item $2\sqrt{7}$ 1258 | \item $x\sqrt{y}\quad (x\ge 0)$ 1259 | \item $\frac{2}{5}\sqrt{a}$ 1260 | \item $2m\sqrt{mn}\quad (m\ge 0)$ 1261 | \item $3x\sqrt{\frac{y}{3x}}\quad (x>0)$ 1262 | \end{enumerate} 1263 | \end{multicols} 1264 | 1265 | \item 化去根号里面的分母. 1266 | \begin{enumerate} 1267 | \item $\sqrt{1\frac{1}{2}}$ 1268 | \item $a\sqrt{\frac{a}{b}}\quad (b>0)$ 1269 | \item $\sqrt{\frac{a^3y}{b^4z^3}}\quad (a\ge 0,\; b>0)$ 1270 | \item $(x+y)\sqrt{\frac{1}{x+y}}\quad (x+y>0)$ 1271 | \item $\frac{4a}{3m}\sqrt{\frac{3m}{2a}}\quad (a>0)$ 1272 | \item $(m-n)\sqrt{\frac{m+n}{m-n}}\quad (m-n>0)$ 1273 | \end{enumerate} 1274 | \end{enumerate} 1275 | \end{ex} 1276 | 1277 | \subsection{最简二次根式与同类根式} 1278 | 如果一个二次根式具备下列两个条件称为最简二 1279 | 次根式. 1280 | \begin{enumerate} 1281 | \item 被开方数每一个因数的指数都小于开方次数2; 1282 | \item 根号内不含分母. 1283 | \end{enumerate} 1284 | 1285 | 如根式$\sqrt{2a},\; 3\sqrt{a^2+b^2},\; \sqrt{4x^2+1}$等都是最简二 1286 | 次根式;如$\sqrt{a^3b},\; \sqrt{\frac{b}{3a}}$ 1287 | 等就不是最简二次根式. 1288 | 1289 | \begin{example} 1290 | 把下列根式化成最简根式. 1291 | \begin{enumerate} 1292 | \item $\sqrt{4 x^{3} y^{5}}\quad (x \ge 0)$ 1293 | \item $\sqrt{8 x^{3} y^{4}}\quad (x \ge 0)$ 1294 | \item $x y \sqrt{\left(\frac{1}{x}+\frac{1}{y}\right)(x+y)} \quad(x>0, \; y>0)$. 1295 | \end{enumerate} 1296 | \end{example} 1297 | 1298 | \begin{solution} 1299 | \begin{enumerate} 1300 | \item $\sqrt{4 x^{3} y^{5}}=2 x y^{2} \sqrt{x y}$ 1301 | \item $\sqrt{8 x^{3} y^{4}}=2 x y^{2} \sqrt{2 x}$ 1302 | \item \[\begin{split} 1303 | x y \sqrt{\left(\frac{1}{x}+\frac{1}{y}\right)(x+y)} 1304 | &=x y \sqrt{\frac{(x+y)^{2}}{x y}}\\ 1305 | &={x} y \sqrt{\frac{(x+y)^{2} x y}{x^{2} y^{2}}}\\ 1306 | &=(x+y) \sqrt{x y} 1307 | \end{split}\] 1308 | \end{enumerate} 1309 | 1310 | \end{solution} 1311 | 1312 | 如果几个二次根式化成最简根式以后,被开方数 1313 | 相同,那么这几个二次根式叫做\textbf{同类根式}. 1314 | 1315 | 例如:$2\sqrt{5}$、$\frac{1}{3}\sqrt{5}$、$a\sqrt{5}$等都是同类根 1316 | 式;而$\sqrt{2}$与$\sqrt{3}$、$\sqrt{x}$与$\sqrt{y}$、$\sqrt{a}$与$\sqrt{7a}$等都不 1317 | 是同类根式. 1318 | 1319 | \begin{example} 1320 | 把下列各根式化为最简根式,并指出哪些 1321 | 是同类根式? 1322 | \[\sqrt{2},\qquad \sqrt{75},\qquad \sqrt{\frac{1}{50}},\qquad \sqrt{\frac{1}{27}},\qquad \sqrt{3}\] 1323 | \[\frac{2}{3}\sqrt{8ab^3}\quad (b>0),\qquad 6b\sqrt{\frac{a}{2b}}\quad (b>0)\] 1324 | \end{example} 1325 | 1326 | 1327 | \begin{solution} 1328 | 由于: 1329 | \[\begin{split} 1330 | \sqrt{75}&=\sqrt{25\x 3}=5\sqrt{3}\\ 1331 | \sqrt{\frac{1}{50}}&=\frac{1}{50}\sqrt{25\x 2}=\frac{1}{10}\sqrt{2} \\ 1332 | \sqrt{\frac{1}{27}}&=\frac{1}{27}\sqrt{9\x 3}=\frac{1}{9}\sqrt{3} \\ 1333 | \frac{2}{3}\sqrt{8ab^3}&=\frac{2}{3}\x 2b\sqrt{2ab}=\frac{4b}{3}\sqrt{2ab} \\ 1334 | 6b\sqrt{\frac{a}{2b}}&=\frac{6b}{2b}\sqrt{2ab}=3\sqrt{2ab} \\ 1335 | \end{split}\] 1336 | 因此:$\sqrt{2}$与$\sqrt{\frac{1}{50}}$、$\sqrt{75}$与$\sqrt{\frac{1}{27}}$与$\sqrt{3}$、$\frac{2}{3}\sqrt{8ab^3}$ 1337 | 与$6b\sqrt{\frac{a}{2b}}$ 分别是同类根式. 1338 | \end{solution} 1339 | 1340 | 同类根式与同类项一样,可以进行合并,通常称 1341 | 为\textbf{合并同类根式}.其方法也与合并同类项类似,只要 1342 | 先化为最简根式,再运用分配律把同类根式各根号前 1343 | 的因式相加即可. 1344 | 1345 | \begin{example} 1346 | 合并下列各式中的同类根式: 1347 | \begin{enumerate} 1348 | \item $\sqrt{5}+\frac{1}{2}\sqrt{5}+\frac{1}{3}\sqrt{5}+\frac{1}{6}\sqrt{5}$ 1349 | \item $\sqrt{12}-\sqrt{\frac{3}{4}}-\sqrt{\frac{1}{3}}\sqrt{18}$ 1350 | \item $5\sqrt{ab}-x\sqrt{ab}+y\sqrt{ab}$ 1351 | \end{enumerate} 1352 | \end{example} 1353 | 1354 | \begin{solution} 1355 | \begin{enumerate} 1356 | \item \[\begin{split} 1357 | \text{原式}&=\left(1+\frac{1}{2}+\frac{1}{3}+6\frac{1}{6}\right)\sqrt{5}\\ 1358 | &=2\sqrt{5} 1359 | \end{split}\] 1360 | \item \[\begin{split} 1361 | \text{原式}&=2\sqrt{3}-\frac{1}{2}\sqrt{3}-\frac{1}{3}\sqrt{3}+3\sqrt{2}\\ 1362 | &=\left(2-\frac{1}{2}-\frac{1}{3}\right)\sqrt{3}+3\sqrt{2}\\ 1363 | &=\frac{7}{6}\sqrt{3}+3\sqrt{2} 1364 | \end{split}\] 1365 | \item \[\text{原式}=(5-x+y)\sqrt{ab}\] 1366 | \end{enumerate} 1367 | \end{solution} 1368 | 1369 | \begin{ex} 1370 | \begin{enumerate} 1371 | \item 把下列各根式化成最简根式: 1372 | \begin{enumerate} 1373 | \item $\sqrt{72}$ 1374 | \item $\sqrt{17^{2}-8^{2}}$ 1375 | \item $\sqrt{9 a^{3} b c^{3}}\quad (a \ge 0, c \ge 0)$ 1376 | \item $\sqrt{\frac{1}{3}+\frac{1}{4}}$ 1377 | \item $\sqrt{\frac{y^{2}}{x}}(y \ge 0)$, 1378 | \item $\sqrt{\frac{(a+b)^{4}}{(a-b)^{8}}}$ 1379 | \item $\sqrt{x^{3}-2 x^{2}+x}\quad (x>1)$; 1380 | \item $a b \sqrt{\frac{1}{a}+\frac{1}{b}}$ 1381 | \end{enumerate} 1382 | \item 下列各组根式是不是同类根式? 1383 | \begin{enumerate} 1384 | \item $5 \sqrt{125}$ 和 $3 \sqrt{45}$ 1385 | \item $2 \sqrt{a^{3} b^{3} c},\quad \frac{1}{2} \sqrt{4 a b^{5} c}$ 和 $3 \sqrt{\frac{c}{a b}}$ 1386 | \item $\sqrt{2 x},\quad \sqrt{2 a^{2} x^{3}}\;\; (a>0)$ 和 $\sqrt{50 x y^{2}}\;\;(y>0)$. 1387 | \end{enumerate} 1388 | \item 合并下列各式中的同类根式: 1389 | \begin{enumerate} 1390 | \item $6 \sqrt{3}+\sqrt{0.12}+\sqrt{48}$ 1391 | \item $\frac{1}{2} \sqrt{a}- 2 \sqrt{b}+4 \sqrt{a}+3 \sqrt{b}-\frac{3}{2} \sqrt{a}$ 1392 | \item $\sqrt{125}+\sqrt{\frac{2}{3}}-4 \sqrt{216}+3 \sqrt{\frac{1}{5}}$ 1393 | \item $2 a \sqrt{3 a b^{2}}-\left(\frac{b}{5} \sqrt{27 a^{3}}-2 a b \sqrt{\frac{3a}{4}}\right)\quad (b>0)$ 1394 | \end{enumerate} 1395 | \end{enumerate} 1396 | \end{ex} 1397 | 1398 | \subsection{二次根式的运算} 1399 | 二次根式的运算与整式的运算类似. 1400 | 1401 | \subsubsection{加法和减法法则} 1402 | 1403 | 先把根式化成最简根式,再合并同类根式. 1404 | 1405 | \begin{example} 1406 | 计算 $\left(\sqrt{24}-\sqrt{0.5}-2 \sqrt{\frac{2}{3}}\right)-\left(\sqrt{\frac{1}{2}}-\sqrt{6}\right)$ 1407 | \end{example} 1408 | 1409 | \begin{solution} 1410 | \[\begin{split} 1411 | \text{原式}&= 2 \sqrt{6}-\frac{1}{2} \sqrt{2}-\frac{2}{3} \sqrt{6} -\frac{1}{2} \sqrt{2}+\sqrt{6} \\ 1412 | &=\left(2-\frac{2}{3}+1\right) \sqrt{6}-\left(\frac{1}{2}+\frac{1}{2}\right) \sqrt{2} \\ 1413 | &=\frac{7}{3} \sqrt{6}-\sqrt{2} 1414 | \end{split}\] 1415 | \end{solution} 1416 | 1417 | 1418 | \begin{example} 1419 | 计算 $\frac{2 x}{3} \sqrt{18 x}+12 x \cdot \sqrt{\frac{x}{8}}-x^{2} \sqrt{\frac{2}{x^{3}}}$ 1420 | \end{example} 1421 | 1422 | \begin{solution} 1423 | \[\begin{split} 1424 | \text{原式}&=2 x \sqrt{2 x}+3 x \sqrt{2 x}-\sqrt{2 x} \\ 1425 | &=(2 x+3 x-1) \sqrt{2 x}\\ 1426 | &=(5 x-1) \sqrt{2 x} 1427 | \end{split}\] 1428 | \end{solution} 1429 | 1430 | \begin{ex} 1431 | \begin{enumerate} 1432 | \item 计算下列各式 1433 | \begin{enumerate} 1434 | \item $\sqrt{0.2}+\sqrt{125}$ 1435 | \item $2\sqrt{\frac{1}{27}}-\frac{2}{3}\sqrt{\frac{1}{3}}+\sqrt{1\frac{1}{3}}$ 1436 | \item $\sqrt{3 a x^{2}}-\sqrt{3 a^{3} x^{2}}$ 1437 | \item $\sqrt{\frac{y}{3 x^{2}}}-\frac{1}{3} \sqrt{\frac{y}{3}}$ 1438 | \item $\sqrt{2 a b}-2 b \sqrt{\frac{a}{2 b}}$ 1439 | \item $2 \sqrt{25 a}-3 \sqrt{a^{2}b}+5 \sqrt{36 a}-2 \sqrt{a^{2} b}$ 1440 | \item $\left(\sqrt{32}+\sqrt{0.5}-2 \sqrt{\frac{1}{3}}\right)-\left(\sqrt{\frac{1}{18}}-\sqrt{48}\right)$ 1441 | \item $(5 \sqrt{a}-3 \sqrt{25 a})+(2 \sqrt{35 a}+2 \sqrt{9 a})$. 1442 | \end{enumerate} 1443 | 1444 | \item 下列计算是否正确? 为什么? 1445 | \begin{enumerate} 1446 | \item $\sqrt{3}+\sqrt{97}=10_{3}$ 1447 | \item $5+\sqrt{3}=5 \sqrt{3}$ 1448 | \item $\sqrt{2 x}+\sqrt{14 x}=\sqrt{16 x}=4 \sqrt{x}$ 1449 | \item $\frac{\sqrt{50}+\sqrt{8}}{2}=\sqrt{25}+\sqrt{4}=5+2=7$ 1450 | \end{enumerate} 1451 | \end{enumerate} 1452 | \end{ex} 1453 | 1454 | 由二次根式的基本性质3、4,反过来运用,就 1455 | 可以得到二次根式的乘、除法法则. 1456 | 1457 | \subsubsection{乘法和除法法则} 1458 | \[\sqrt{a}\cdot \sqrt{b}=\sqrt{ab}\quad (a\ge 0,\; b\ge 0),\qquad \frac{\sqrt{a}}{\sqrt{b}}=\sqrt{\frac{a}{b}}\quad (a\ge 0,\; b>0) \] 1459 | 并化为最简根式. 1460 | 一般地,还可以写成以下公式: 1461 | \[\begin{split} 1462 | m\sqrt{a}\cdot n\sqrt{b}&= mn\sqrt{ab}\\ 1463 | m\sqrt{a}\div n\sqrt{b}&=\frac{m\sqrt{a}}{n\sqrt{b}}=\frac{m}{n}\sqrt{\frac{a}{b}} 1464 | \end{split}\] 1465 | 并将结果化为最简二次根式. 1466 | 1467 | \begin{example} 1468 | 计算下列各式: 1469 | \begin{enumerate} 1470 | \item $\sqrt{4x^3}\cdot \sqrt{\frac{3x}{2}}$ 1471 | \item $3\sqrt{5a}\cdot 4\sqrt{10b}$ 1472 | \item $\left(\sqrt{xy}+2\sqrt{\frac{y}{x}}-\sqrt{\frac{x}{y}}+\sqrt{\frac{1}{xy}}\right)\cdot \sqrt{xy}$ 1473 | \end{enumerate} 1474 | \end{example} 1475 | 1476 | \begin{solution} 1477 | \begin{enumerate} 1478 | \item $\sqrt{4x^3}\cdot \sqrt{\frac{3x}{2}}=\sqrt{4x^3\cdot \frac{3x}{2}}=\sqrt{6x^4}=x^2\sqrt{6}$ 1479 | \item $3\sqrt{5a}\cdot 4\sqrt{10b}=3\x 4\sqrt{50ab}=12\x \sqrt{2ab}=60\sqrt{2ab}$ 1480 | \item $\left(\sqrt{xy}+2\sqrt{\frac{y}{x}}-\sqrt{\frac{x}{y}}+\sqrt{\frac{1}{xy}}\right)\cdot \sqrt{xy}=xy+2y-x+1$ 1481 | \end{enumerate} 1482 | \end{solution} 1483 | 1484 | 1485 | \begin{example} 1486 | 计算下列各式: 1487 | \begin{enumerate} 1488 | \item $\left(3\sqrt{2}-2\sqrt{3}\right)\left(7\sqrt{2}+5\sqrt{3}\right)$ 1489 | \item $\left(6\sqrt{3}+3\sqrt{6}\right)^2$ 1490 | \item $\left(x\sqrt{a}-y\sqrt{b}\right)^2$ 1491 | \end{enumerate} 1492 | \end{example} 1493 | 1494 | \begin{solution} 1495 | \begin{enumerate} 1496 | \item $\text{原式}=42-14\sqrt{6}+15\sqrt{6}-30=12+\sqrt{6}$ 1497 | \item $\text{原式}=\left(6\sqrt{3}\right)^2+2\cdot 6\sqrt{3}\cdot 3\sqrt{6}+\left(3\sqrt{6}\right)^2=108+108\sqrt{2}+54=162+108\sqrt{2}$ 1498 | \item $\text{原式}=ax^2+by^2-2xy\sqrt{ab}$ 1499 | \end{enumerate} 1500 | \end{solution} 1501 | 1502 | 1503 | \begin{example} 1504 | 计算下列各式: 1505 | \begin{multicols}{2} 1506 | \begin{enumerate} 1507 | \item $\left(\sqrt{7}+\sqrt{3}\right)\cdot \left(\sqrt{7}-\sqrt{3}\right)$ 1508 | \item $\left(\sqrt{xy}-\sqrt{ab}\right)\cdot \left(\sqrt{xy}+\sqrt{ab}\right)$ 1509 | \item $\left(4\sqrt{\frac{a}{2}}+6\sqrt{\frac{b}{4}}\right)\cdot \left(\sqrt{8a}-3\sqrt{b}\right)$ 1510 | \end{enumerate} 1511 | \end{multicols} 1512 | \end{example} 1513 | 1514 | \begin{solution} 1515 | \begin{enumerate} 1516 | \item $\text{原式}=\left(\sqrt{7}\right)^2-\left(\sqrt{3}\right)^2=7-3=4$ 1517 | \item $\text{原式}=\left(\sqrt{xy}\right)^2-\left(\sqrt{ab}\right)^2=xy-ab$ 1518 | \item $\text{原式}=\left(2\sqrt{2a}+3\sqrt{b}\right)\cdot\left(2\sqrt{2a}-3\sqrt{b}\right)=\left(2\sqrt{2a}\right)^2-\left(3\sqrt{b}\right)^2=8a-9b$ 1519 | \end{enumerate} 1520 | \end{solution} 1521 | 1522 | 由例6.45中,我们发现:计算结果都不再含有根 1523 | 式. 1524 | 1525 | 两个含有根式的式子相乘,如果它们的乘积中不 1526 | 再含有根式,那么,这两个式子就叫做\textbf{互为有理化因 1527 | 式}. 1528 | 1529 | 例如,$\sqrt{7}+\sqrt{3}$与$\sqrt{7}-\sqrt{3}$;$\sqrt{x+y}$与$\sqrt{x+y}$;$a+2\sqrt{b}$与$a-2\sqrt{b}$;$\sqrt{xy}+\sqrt{ab}$与 1530 | $\sqrt{xy}-\sqrt{ab}$;$2\sqrt{2a}+3\sqrt{b}$与$2\sqrt{2a}-3\sqrt{b}$等 1531 | 等,都是互为有理化因式. 1532 | 1533 | \begin{ex} 1534 | 计算下列各题: 1535 | \begin{enumerate} 1536 | \item \begin{multicols}{2} 1537 | \begin{enumerate} 1538 | \item $\sqrt{6x}\cdot \sqrt{2x}$ 1539 | \item $2\sqrt{2a}\cdot \sqrt{24ab}$ 1540 | \item $\frac{3}{4}\sqrt{\frac{5a}{2}}\cdot \sqrt{\frac{0.4}{a}}$ 1541 | \item $\left(\sqrt{10}-2\sqrt{15}\right)\cdot \sqrt{5}$ 1542 | \item $\left(x\sqrt{y}-y\sqrt{x}\right)\cdot \sqrt{xy}$ 1543 | \end{enumerate} 1544 | \end{multicols} 1545 | \item \begin{enumerate} 1546 | \item $\left(4\sqrt{3}+5\sqrt{2}\right)\cdot \left(\sqrt{3}-\sqrt{2}\right)$ 1547 | \item $(a+\sqrt{ab})\cdot (b+\sqrt{ab})$ 1548 | \item $\left(\sqrt{mn}+\sqrt{\frac{m}{n}}-m\right)\cdot \left(\sqrt{\frac{m}{n}}-n\right)$ 1549 | \end{enumerate} 1550 | 1551 | \item \begin{enumerate} 1552 | \item $\left(7+3\sqrt{2}\right)\cdot \left(\sqrt{18}-7\right)$ 1553 | \item $\left(2\sqrt{ax}+5\sqrt{by}\right)\cdot \left(2\sqrt{ax}-5\sqrt{by}\right)$ 1554 | \item $\left(\sqrt{x+3}-\sqrt{2x}\right)\cdot \left(\sqrt{x+3}+\sqrt{2x}\right)$ 1555 | \item $\left(\frac{-1+\sqrt{3}}{2}\right)^2$ 1556 | \item $\left(\sqrt{\frac{a}{b}}+\sqrt{\frac{b}{a}}\right)^2$ 1557 | \item $\left(\sqrt{a}+\sqrt{b}\right)^2+\left(\sqrt{a}-\sqrt{b}\right)^2$ 1558 | \item $\left(1+\sqrt{x}-\sqrt{y}\right)\cdot \left(1-\sqrt{x}+\sqrt{y}\right)$ 1559 | 1560 | \end{enumerate} 1561 | 1562 | \item 写出以下各根式的有理化因式: 1563 | \[3\sqrt{7},\quad 7-\sqrt{11},\quad 5\sqrt{3}+\sqrt{10},\quad \sqrt{2x^2-1},\quad x+x\sqrt{y}\] 1564 | \[a^2-b\sqrt{a+1},\qquad \sqrt{x+2}-\sqrt{x}\] 1565 | \end{enumerate} 1566 | \end{ex} 1567 | 1568 | \begin{example} 1569 | 计算下列各式: 1570 | \begin{multicols}{2} 1571 | \begin{enumerate} 1572 | \item $\sqrt{104}\div\sqrt{13}$ 1573 | \item $9\sqrt{45}\div \frac{3}{2}\sqrt{1\frac{1}{2}}$ 1574 | \item $18\sqrt{2x^3}\div 3\sqrt{3y}$ 1575 | \item $-6\sqrt{\frac{2a-2b}{x^2}}\div \frac{4}{5}\sqrt{\frac{a-b}{2bx^2}}$ 1576 | \end{enumerate} 1577 | \end{multicols} 1578 | \end{example} 1579 | 1580 | \begin{solution} 1581 | \begin{enumerate} 1582 | \item $\text{原式}=\frac{\sqrt{104}}{\sqrt{13}}=\sqrt{\frac{104}{13}}=\sqrt{8}=2\sqrt{2} $ 1583 | \item $\text{原式}=9\x \frac{2}{3}\sqrt{45\x \frac{2}{3}}=6\sqrt{30} $ 1584 | \item $\text{原式}=\frac{18\sqrt{2x^3}}{3\sqrt{3y}}=6\frac{2x^3}{3y} =\frac{2x}{y}\sqrt{6x} $ 1585 | \item $\text{原式}=-6\x \frac{5}{4}\sqrt{\frac{2a-2b}{x^2}\cdot \frac{2bx^2}{a-b}}=-\frac{15}{2}\sqrt{4b}=-15\sqrt{b} $ 1586 | \end{enumerate} 1587 | \end{solution} 1588 | 1589 | 在二次根式的除法运算中,总是先用法则将分 1590 | 子、分母并入一个根号内,再将根号里边的分母化 1591 | 去,使结果成为最简根式. 1592 | 1593 | 在实际的运算中,有时先把分母中的根号化去再 1594 | 进行计算,较为简便,例如,计算$\frac{\sqrt{3}}{\sqrt{2}}$ 1595 | 的近似值(精确到 1596 | 0.01)时,就可以先把分母中的根号化去再计算: 1597 | \[\frac{\sqrt{3}}{\sqrt{2}}=\frac{\sqrt{3}\cdot \sqrt{2}}{\sqrt{2}\cdot \sqrt{2}}=\frac{\sqrt{6}}{2}\approx \frac{2.449}{2}\approx 1.23 \] 1598 | 1599 | 同样,在计算$\frac{1}{\sqrt{3}-\sqrt{2}}$ 1600 | (精确到0.01)的近似值 1601 | 时,也可以先将分母中的根号化去再计算,较为简 1602 | 便. 1603 | \[\begin{split} 1604 | \frac{1}{\sqrt{3}-\sqrt{2}}&=\frac{\sqrt{3}+\sqrt{2}}{\left(\sqrt{3}-\sqrt{2}\right)\left(\sqrt{3}+\sqrt{2}\right)}\\ 1605 | &=\frac{\sqrt{3}+\sqrt{2}}{3-2}=\sqrt{3}+\sqrt{2}\\ 1606 | &\approx 1.732+1.414=3.146\approx 3.15 1607 | \end{split}\] 1608 | 1609 | 把分母中的根号化去,叫做\textbf{分母有理化}.分母有 1610 | 理化的方法是:根据分式的基本性质,只要将分子、 1611 | 分母同乘以分母的有理化因式,就可以达到目的. 1612 | 1613 | 在根式的除法中,先进行有理化分母,往往是较 1614 | 简便的. 1615 | 1616 | \begin{example} 1617 | 计算: 1618 | \begin{multicols}{2} 1619 | \begin{enumerate} 1620 | \item $\left(\sqrt{a^3b}+\sqrt{ab^3}-ab\right)\div \sqrt{ab}$ 1621 | \item $\frac{a^2-b^2}{\sqrt{a+b}}$ 1622 | \item $\left(4\sqrt{3}+5\sqrt{2}\right)\div \left(\sqrt{3}-\sqrt{2}\right)$ 1623 | \item $\frac{2}{3+2\sqrt{2}}$ 1624 | \item $\frac{\sqrt{a+b}+\sqrt{a-b}}{\sqrt{a+b}+\sqrt{a-b}}\quad (a>b)$ 1625 | \end{enumerate} 1626 | \end{multicols} 1627 | \end{example} 1628 | 1629 | \begin{solution} 1630 | \begin{enumerate} 1631 | \item \[\begin{split} 1632 | \text{原式 }&=\frac{\left(\sqrt{a^{3} b}+\sqrt{a b^{3}}-a b\right) \cdot \sqrt{a b}}{\sqrt{a b \cdot \sqrt{a b}}}\\ 1633 | &=\frac{1}{a b}\left(a^{2} b+a b^{2}-a b \sqrt{a b}\right)\\ 1634 | &=a+b-\sqrt{ab} 1635 | \end{split}\] 1636 | \item \[\begin{split} 1637 | \text{原式 }&= \frac{(a^2-b^2)\sqrt{a+b}}{\sqrt{a+b}\cdot \sqrt{a+b}} \\ 1638 | &= \frac{a^2-b^2}{a+b}\cdot \sqrt{a+b} \\ 1639 | &= (a-b)\sqrt{a+b} 1640 | \end{split}\] 1641 | \item \[\begin{split} 1642 | \text{原式 }&= \frac{4\sqrt{3}+5\sqrt{2}}{\sqrt{3}-\sqrt{2}} \\ 1643 | &= \frac{\left(4\sqrt{3}+5\sqrt{2}\right)\left(\sqrt{3}+\sqrt{2}\right)}{\left(\sqrt{3}-\sqrt{2}\right)\left(\sqrt{3}+\sqrt{2}\right)} \\ 1644 | &= \frac{12+4\sqrt{6}+5\sqrt{6}+10}{(\sqrt{3})^2-(\sqrt{2})^2}\\ 1645 | &=22+9\sqrt{6} 1646 | \end{split}\] 1647 | \item \[\begin{split} 1648 | \text{原式 }&= \frac{2\cdot (3-2\sqrt{2})}{(3+2\sqrt{2})(3-2\sqrt{2})} \\ 1649 | &= \frac{6-4\sqrt{2}}{3^2-(2\sqrt{2})^2} \\ 1650 | &= \frac{6-4\sqrt{2}}{9-8}=6-4\sqrt{2} 1651 | \end{split}\] 1652 | \item \[\begin{split} 1653 | \text{原式 }&= \frac{(\sqrt{a+b}+\sqrt{a-b})^2}{(\sqrt{a+b}-\sqrt{a-b})(\sqrt{a+b}+\sqrt{a-b})} \\ 1654 | &= \frac{a+b+2\sqrt{a+b}\cdot \sqrt{a-b}+a-b}{(\sqrt{a+b})^2-(\sqrt{a-b})^2} \\ 1655 | &= \frac{2a+2\sqrt{a+b}\cdot \sqrt{a-b}}{2b}\\ 1656 | &=\frac{a+\sqrt{a^2-b^2}}{b} 1657 | \end{split}\] 1658 | \end{enumerate} 1659 | \end{solution} 1660 | 1661 | \begin{example} 1662 | 计算(精确到0.01): 1663 | \begin{multicols}{2} 1664 | \begin{enumerate} 1665 | \item $\frac{\sqrt{3}}{\sqrt{2}+\sqrt{3}+\sqrt{5}}$ 1666 | \item $2\div \left(1-\sqrt{2}+\sqrt{3}\right)$ 1667 | \end{enumerate} 1668 | \end{multicols} 1669 | \end{example} 1670 | 1671 | \begin{solution} 1672 | \begin{enumerate} 1673 | \item \[\begin{split} 1674 | \text{原式 }&=\frac{\sqrt{3}(\sqrt{2}+\sqrt{3}-\sqrt{5})}{\left[(\sqrt{2}+\sqrt{3})+\sqrt{5}\right]\left[(\sqrt{2}+\sqrt{3})-\sqrt{5}\right]} \\ 1675 | &= \frac{\sqrt{3}(\sqrt{2}+\sqrt{3}-\sqrt{5})}{(\sqrt{2}+\sqrt{3})^2-(\sqrt{5})^2} \\ 1676 | &= \frac{\sqrt{6}+3-\sqrt{15}}{2\sqrt{6}}\\ 1677 | &= \frac{(3+\sqrt{6}-\sqrt{15})\cdot \sqrt{6}}{2\sqrt{6}\cdot \sqrt{6}}\\ 1678 | &=\frac{3\sqrt{6}+6-\sqrt{90}}{12}=\frac{1}{4}\left(2+\sqrt{6}-\sqrt{10}\right)\\ 1679 | &\approx \frac{1}{4}(2+2.449-3.162)\approx 0.32 1680 | \end{split}\] 1681 | \item \[\begin{split} 1682 | \text{原式 }&= \frac{2\left[(1-\sqrt{2})-\sqrt{3}\right]}{\left[(1-\sqrt{2})+\sqrt{3}\right]\cdot \left[(1-\sqrt{2})-\sqrt{3}\right]} \\ 1683 | &= \frac{2-2\sqrt{2}-2\sqrt{3}}{(1-\sqrt{2})^2-(\sqrt{3})^2} \\ 1684 | &= \frac{2-2\sqrt{2}-2\sqrt{3}}{-2\sqrt{2}} \\ 1685 | &= \frac{1-\sqrt{2}-\sqrt{3}}{-\sqrt{2}} =\frac{(1-\sqrt{2}-\sqrt{3})\cdot \sqrt{2}}{-\sqrt{2}\cdot \sqrt{2}}\\ 1686 | &=\frac{\sqrt{2}-2-\sqrt{6}}{-2}\\ 1687 | &\approx -\frac{1}{2}(1.414-2-2.449)\approx 1.52 1688 | \end{split}\] 1689 | \end{enumerate} 1690 | \end{solution} 1691 | 1692 | 1693 | 1694 | \begin{ex} 1695 | \begin{enumerate} 1696 | \item 计算下列各式: 1697 | \begin{multicols}{2} 1698 | \begin{enumerate} 1699 | \item $\sqrt{ab}\div \sqrt{3a}$ 1700 | \item $5n\div 3\sqrt{mn}$ 1701 | \item $\sqrt{9a^2}\div \sqrt{\frac{a}{9}}$ 1702 | \item $25a^2x\div 5a\sqrt{x}$ 1703 | \item $\left(x\sqrt{y}-y\sqrt{x}\right)\div \sqrt{xy}$ 1704 | \item $\left(\sqrt{x}-\sqrt{\frac{x}{2}}\right)\div \sqrt{x}$ 1705 | \item $\left(\sqrt{mn}+\sqrt{\frac{m}{n}}{-m}\right)\div \sqrt{\frac{m}{n}}$ 1706 | \item $\frac{\sqrt{5}}{\sqrt{5}+1}$ 1707 | \item $(\sqrt{5}+\sqrt{2})\div (\sqrt{5}-\sqrt{2})$ 1708 | \item $(a+\sqrt{ab}\div (b+\sqrt{ab}))$ 1709 | \end{enumerate} 1710 | \end{multicols} 1711 | 1712 | \item 将下列各式分母有理化: 1713 | \begin{multicols}{2} 1714 | \begin{enumerate} 1715 | \item $\frac{5y^2}{\sqrt{75y}}$ 1716 | \item $\frac{1-\sqrt{3}}{2+\sqrt{3}}$ 1717 | \item $\frac{5}{2\sqrt{3}-\sqrt{2}}$ 1718 | \item $\frac{a-b}{\sqrt{a}-\sqrt{b}}$ 1719 | \item $\frac{1}{x-\sqrt{1+x^2}}$ 1720 | \item $\frac{\sqrt{2x^2+1}}{\sqrt{2x^2+1}+\sqrt{2x^2-1}}$ 1721 | \end{enumerate} 1722 | \end{multicols} 1723 | 1724 | \item 试一试:你能把下列各式的分子有理化吗? 1725 | \begin{multicols}{2} 1726 | \begin{enumerate} 1727 | \item $\frac{\sqrt{10}-\sqrt{6}}{4}$ 1728 | \item $\frac{b(\sqrt{x^2-a^2}-x)}{a}$ 1729 | \end{enumerate} 1730 | \end{multicols} 1731 | \end{enumerate} 1732 | \end{ex} 1733 | 1734 | \subsection{根式方程} 1735 | 已经学过的整式方程和分式方程统称为有理方 1736 | 程.在实际中我们还会遇到像$\sqrt{x^2-1}=2$, $\sqrt{1-x}+\sqrt{12+x}=5$, 1737 | $\frac{1}{\sqrt{x}}=7$等方程,这些\textbf{根号里含有未知 1738 | 数的方程叫做根式方程}.方程$\sqrt{2}x^2+3x-\sqrt{5}=0$, 1739 | $\frac{x}{\sqrt{2}-1}=3$, 虽然带有根号,但根号内不含有未知 1740 | 数,所以它们不是根式方程. 1741 | 1742 | 在根式方程中,由于未知数包含在根号内,因而, 1743 | 未知数只允许取使二次根式有意义的值.例如,根式 1744 | 方程$\sqrt{x-1}+\sqrt{3-x}=2$中,未知数$x$只允许在 1745 | “能使$x-1\ge 0$, 且$3-x\ge 0$”的范围内取值. 1746 | 1747 | 解根式方程主要是设法把原方程变形为有理方 1748 | 程.我们通常采用的方法是方程两边平方,逐步使含 1749 | 有未知数的根式有理化. 1750 | 1751 | 1752 | 1753 | 1754 | 1755 | \begin{example} 1756 | 解方程$\sqrt{x+7}=x-5$ 1757 | \end{example} 1758 | 1759 | \begin{solution} 1760 | 两边平方得: 1761 | \[(\sqrt{x+7})^2=(x-5)^2\] 1762 | 整理后,得方程:$x^2-11x+18=0$ 1763 | 解出: 1764 | $x_1=9,\qquad x_2=2$ 1765 | 1766 | 验根:把$x=9$代入原方程两边. 1767 | \[\begin{split} 1768 | \text{左式}&=\sqrt{9+7}=4\\ 1769 | \text{右式}&=9-5=4 1770 | \end{split}\] 1771 | $\therefore\quad x=9$是原方程的根. 1772 | 1773 | 把$x=2$代入原方程两边. 1774 | \[\begin{split} 1775 | \text{左式}&=\sqrt{2+7}=3\\ 1776 | \text{右式}&=2-5=-3 1777 | \end{split}\] 1778 | 两边的值不等. 1779 | 1780 | $\therefore\quad x=2$是原方程的增根(舍去). 1781 | 1782 | $\therefore\quad $原方程的解集是:$\{9\}$. 1783 | 1784 | 通过上例可以看出:根式方程两边平方后,就得 1785 | 到一个新的整式方程,这个新方程的根可能是原根式 1786 | 方程的根;也可能是原方程的增根.但为什么会产生 1787 | 增根呢? 1788 | 1789 | 观察原方程$\sqrt{x+7}=x-5$, 不难知道:未知 1790 | 数$x$的取值范围,既要使 1791 | $x+7\ge 0$, 又要使$x-5 1792 | \ge 0$, 这样才能使原方程中的根式有意义. 1793 | 1794 | 但是,在原方程两边平方后所得的新方程 1795 | $(\sqrt{x+7})^2=(x-5)^2$中,未知数$x$的取值范围, 1796 | 只要求使$x+7\ge 0$就能使新方程有意义.这就是说, 1797 | 新方程中未知数的取值范围扩大了. 1798 | 1799 | 事实上,方程两边平方$(\sqrt{x+7})^2=(x-5)^2$ 1800 | 可以变形为: 1801 | \[(\sqrt{x+7})^2-(x-5)^2=0\] 1802 | 即: 1803 | \[\left[\sqrt{x+7}-(x-5)\right]\left[\sqrt{x+7}+(x-5)\right]=0 \] 1804 | 这就相当于得到以下两个方程: 1805 | \[\sqrt{x+7}-(x-5)=0,\qquad \sqrt{x+7}+(x-5)=0\] 1806 | 这样一来,解整式方程:$x+7=(x-5)^2$, 就相当 1807 | 于解以上两个根式方程,但原题所给出的根式方程只 1808 | 是其中的一个.因此,解的过程中就可能产生增根. 1809 | \end{solution} 1810 | 1811 | 很明显,在例6.49中,原方程的增根$x=2$就是方 1812 | 程$\sqrt{x+7}=-(x-5)$的根. 1813 | 1814 | 由以上分析,可以得出以下结论: 1815 | 1816 | \begin{blk}{} 1817 | 方程两边平方,实际上就是方程两边乘以同一 1818 | 个含有未知数的因式,这样,未知数的取值范围就可 1819 | 能扩大,就可能产生原方程的增根.这时,就必须进 1820 | 行验根,把增根舍去. 1821 | \end{blk} 1822 | 1823 | 1824 | 1825 | \begin{example} 1826 | 解方程$\sqrt{x+10}+\sqrt{x-11}=7$ 1827 | \end{example} 1828 | 1829 | \begin{analyze} 1830 | 解根式方程时,利用“\textbf{移项规则}”可以把 1831 | 所含根式比较均匀地分列于等号两边,然后再逐步平 1832 | 方.这样比较简便. 1833 | \end{analyze} 1834 | 1835 | 1836 | \begin{solution} 1837 | \begin{align*} 1838 | \sqrt{x+10}&=7-\sqrt{x-11}\tag{移项}\\ 1839 | x+10&=49-14\sqrt{x-11}+x-11 \tag{两边平方}\\ 1840 | \sqrt{x-11}&=2\\ 1841 | x-11&=4 \tag{两边平方}\\ 1842 | x&=15 1843 | \end{align*} 1844 | 验根:把$x=15$代入原方程: 1845 | \[\begin{split} 1846 | \text{左式}&=15+10+15-11=5+2=7\\ 1847 | \text{右式}&=7 1848 | \end{split}\] 1849 | $\therefore\quad $ 1850 | 原方程的解集是$\{15\}$. 1851 | \end{solution} 1852 | 1853 | \begin{example} 1854 | 解方程$\sqrt{2x-3}+\sqrt{3x-5}-\sqrt{5x-6}=0$ 1855 | \end{example} 1856 | 1857 | \begin{solution} 1858 | \begin{align*} 1859 | \sqrt{2x-3}+\sqrt{3x-5}&=\sqrt{5x-6} \tag{移项}\\ 1860 | 2x-3+2\sqrt{2x-3}\cdot \sqrt{3x-5}+3x-5&=5x-6 \tag{两边平方}\\ 1861 | \sqrt{2x-3}\cdot \sqrt{3x-5}&=1 1862 | (2x-3)(3x-5)&=1 \tag{两边平方}\\ 1863 | 6x^2-19x+14&=0 1864 | \end{align*} 1865 | $\therefore\quad x_1=2,\quad x_2=\frac{7}{6}$ 1866 | 1867 | 把$x=2$代入原方程: 1868 | \[\begin{split} 1869 | \text{左式}&=1+1-2=0\\ 1870 | \text{右式}&=0 1871 | \end{split}\] 1872 | $\therefore\quad x=2$是原方程的根. 1873 | 1874 | 把$x=\frac{7}{6}$代入原方程,因为$\sqrt{2x-3}=\sqrt{-\frac{2}{3}}$, 1875 | 被开方数是负数,使原方程无意义,所以 1876 | $x=\frac{7}{6}$是增根(舍去). 1877 | 1878 | $\therefore\quad $原方程的解集是$\{2\}$. 1879 | \end{solution} 1880 | 1881 | 由以上例题,告诉我们: 1882 | 1883 | 解根式方程进行验根时,可以把有理方程的每一 1884 | 个根分别代入原根式方程的每一个根号内,如果代入 1885 | 后,至少有一个根号内得负数,就说明这个根使原方 1886 | 程无意义,肯定是增根,应该舍去(如例6.51);如果 1887 | 代入后,使原方程中的每一个根号内都为非负数,就 1888 | 说明这个根能使原方程中的根式有意义,但还不能保 1889 | 证能使原方程两边相等,仍然有可能是增根(如例 1890 | 6.49),所以还应该继续代入原方程两边,进行检验. 1891 | 1892 | 总之,解根式方程的一般步骤是: 1893 | \begin{enumerate} 1894 | \item 移项,使方程中含未知数的根式比较均匀的 1895 | 分列于等号的两边; 1896 | \item 方程两边同时平方,逐次化去根号,得到有 1897 | 理方程; 1898 | \item 解有理方程; 1899 | \item 验根. 1900 | \end{enumerate} 1901 | 1902 | \begin{ex} 1903 | \begin{enumerate} 1904 | \item 解下列方程: 1905 | \begin{multicols}{2} 1906 | \begin{enumerate} 1907 | \item $\sqrt{x^2-1}=\sqrt{3}$ 1908 | \item $\sqrt{x^2-3x+4}+5=x$ 1909 | \item $\sqrt{2x-4}-\sqrt{x+5}=1$ 1910 | \item $\sqrt{x-1}\cdot \sqrt{2x+6}-x=3$ 1911 | \end{enumerate} 1912 | \end{multicols} 1913 | 1914 | 1915 | \item 不解方程判别下列各方程是否有解 1916 | \begin{enumerate} 1917 | \item $\sqrt{3x-2}=-4$ 1918 | \item $\sqrt{2x+1}=-(x^2+1)$ 1919 | \item $\sqrt{2x-1}+\sqrt{x+1}=0$ 1920 | \item $\sqrt{x-1}+\sqrt{x-2}+\sqrt{x-3}=-3$ 1921 | \end{enumerate} 1922 | \end{enumerate} 1923 | \end{ex} 1924 | 1925 | \begin{example} 1926 | 解方程$x^2-2x+6\sqrt{x^2-2x+6}=21$ 1927 | \end{example} 1928 | 1929 | 1930 | \begin{solution} 1931 | 设$\sqrt{x^2-2x+6}=y$, 则$x^2-2x+6=y^2$. 1932 | 原方程可变形为$y^2+6y-27=0$ 1933 | $\because\quad y_1=3,\qquad y_2=-9$ 1934 | 1935 | 这就是$\sqrt{x^2-2x+6}=3$或$\sqrt{x^2-2x+6}=-9$ 1936 | ,其中$\sqrt{x^2-2x+6}$是算术根,所以不能等于$-9$. 1937 | 1938 | $\therefore\quad \sqrt{x^2-2x+6}=-9$无解. 1939 | 1940 | 解方程 1941 | \begin{equation} 1942 | \sqrt{x^2-2x+6}=3 1943 | \end{equation} 1944 | 两边平方: 1945 | \begin{equation} 1946 | x^2-2x+6=9 \quad \Rightarrow\quad x^2-2x-3=0 1947 | \end{equation} 1948 | $\therefore\quad x_1=3,\qquad x_2=-1$ 1949 | 1950 | 验根:把$x_1=3,\; x_2=-1$, 分别代入方程检验, 1951 | 可知这两个根都是原方程的根. 1952 | 1953 | $\therefore\quad $原方程的解集是:$\{3,\;-1\}$. 1954 | \end{solution} 1955 | 1956 | 在上题求解过程中,由方程(6.4)变形为方程(6.5) 1957 | 有产生增根的可能,所以,验根时只须代入方程(6.4) 1958 | 检验就可以. 1959 | 1960 | \begin{ex} 1961 | 解下列方程 1962 | \begin{enumerate} 1963 | \item $x^2+8x+\sqrt{x^2+8x}=12$ 1964 | \item $x^2-x+\sqrt{x^2-x+2}-4=0$ 1965 | \end{enumerate} 1966 | \end{ex} 1967 | 1968 | \begin{example} 1969 | 解方程:$\sqrt{3x-1}+\frac{2}{\sqrt{3x-1}}=\sqrt{5x+3}$ 1970 | \end{example} 1971 | 1972 | \begin{solution} 1973 | 方程两边乘以$\sqrt{3x-1}$得: 1974 | \[3x-1+2=\sqrt{5x+3}\cdot \sqrt{3x-1} \] 1975 | 即: 1976 | \[3x+1=\sqrt{(5x+3)\cdot (3x-1)}\] 1977 | 两边平方:$(3x+1)^2=(5x+3) (3x-1)$,整理得: 1978 | \[3x^2-x-2=0 \] 1979 | $\therefore\quad x_1=2,\quad x_2=-\frac{2}{3}$ 1980 | 1981 | 经检验,当$x=-\frac{2}{3}$时, 1982 | 原方程所含的根号内都 1983 | 是负数,因而$x=-\frac{2}{3}$ 1984 | 是增根,应舍去. 1985 | 1986 | $x=1$是原方程的根. 1987 | 1988 | $\therefore\quad $原方程的解集是:$\{1\}$. 1989 | \end{solution} 1990 | 1991 | \begin{example} 1992 | 解方程$\frac{1}{1-\sqrt{1-x^2}}-\frac{1}{1+\sqrt{1-x^2}}=\frac{\sqrt{3}}{x^2}$ 1993 | \end{example} 1994 | 1995 | \begin{solution} 1996 | 分母有理化得: 1997 | \[\frac{1+\sqrt{1-x^2}}{1-(1-x^2)}-\frac{1-\sqrt{1-x^2}}{1-(1-x^2)}=\frac{\sqrt{3}}{x^2}\] 1998 | 就是 1999 | \[\frac{1+\sqrt{1-x^2}}{x^2}-\frac{1-\sqrt{1-x^2}}{x^2}=\frac{\sqrt{3}}{x^2}\] 2000 | 去分母: 2001 | \[\begin{split} 2002 | 2\sqrt{1-x^2}&= \sqrt{3}\\ 2003 | 4(1-x^2)&=3\\ 2004 | 4x^2&=1 2005 | \end{split}\] 2006 | $\therefore\quad x_1=\frac{1}{2},\quad x_2=-\frac{1}{2}$ 2007 | 2008 | 经检验知,$x_1=\frac{1}{2}$和$x_2=-\frac{1}{2}$ 2009 | 都是原方程的根. 2010 | 2011 | 原方程的解集是$\left\{\frac{1}{2},\; -\frac{1}{2}\right\}$ 2012 | \end{solution} 2013 | 2014 | \begin{ex} 2015 | 解下列方程: 2016 | \begin{enumerate} 2017 | \item $\sqrt{x}+\frac{1}{\sqrt{x}}=2$ 2018 | \item $\frac{1}{x+\sqrt{1+x^2}}+\frac{1}{x-\sqrt{1+x^2}}+2=0$ 2019 | \end{enumerate} 2020 | \end{ex} 2021 | 2022 | 2023 | 2024 | \section*{习题6.2} 2025 | \addcontentsline{toc}{subsection}{习题6.2} 2026 | 2027 | \begin{enumerate} 2028 | \item 求下列各式的值: 2029 | \begin{enumerate} 2030 | \item $\sqrt{(a-b)^{2}}$, 当 $a=15,\; b=9$ 2031 | \item $\sqrt{(2 m-3 n)^{2}}$, 当 $m=4,\; n=5$ 2032 | \item $\sqrt{b^{2}-6 b+9}$, 当 $0b>0)$ 2063 | \item $\left(\sqrt{a b}-\sqrt{\frac{a}{b}}\right)-\left(\sqrt{\frac{b}{a}}-\sqrt{\frac{a}{b}}+\frac{b}{a}+2\right)$ 2064 | \end{enumerate} 2065 | 2066 | \item 计算下列各式: 2067 | \begin{enumerate} 2068 | \item $\left(\sqrt{24}-3 \sqrt{1.5}+2 \sqrt{2 \frac{2}{3}}\right) \cdot \sqrt{2}$ 2069 | \item $\left(\frac{a}{b} \sqrt{\frac{n}{m}}-\frac{a b}{n} \sqrt{m n}+\frac{a^{2}}{b^{2}} \sqrt{\frac{m}{n}}\right) \cdot a^{2} b^{2} \sqrt{\frac{n}{m}}$ 2070 | \item $10 a^{2} \sqrt{a b} \cdot 5 \sqrt{\frac{b}{a}} \cdot 15 \sqrt{\frac{a}{b}}$ 2071 | \item $(\sqrt{27}+\sqrt{28})(\sqrt{12}-\sqrt{63})$ 2072 | \item $\left(\frac{1}{4} \sqrt{a-b}+2 \sqrt{a}\right)\left(\frac{1}{4} \sqrt{a-b}-2 \sqrt{a}\right)$ 2073 | \item $\left(\sqrt{4+2 \sqrt{3}}-\sqrt{4-2 \sqrt{3}}\right)^{2}$ 2074 | \end{enumerate} 2075 | 2076 | \item 计算下列各式: 2077 | \begin{enumerate} 2078 | \item $\left(\frac{3 x}{2} \sqrt{\frac{x}{y}}-0.4 \sqrt{\frac{3}{x y}}+\frac{1}{3} \sqrt{\frac{x y}{2}}\right) \div \frac{4}{15} \sqrt{\frac{3 y}{2 x}}$ 2079 | \begin{multicols}{2} 2080 | \item $\frac{3 \sqrt{5}+\sqrt{32}}{3 \sqrt{5}-\sqrt{32}}$ 2081 | \item $\frac{a+\sqrt{a^{2}-1}}{a-\sqrt{a^{2}-1}}\quad (a>1)$ 2082 | \item $\frac{x+y+2 \sqrt{x y}}{(\sqrt{x}-\sqrt{y})^{2}}$ 2083 | \item $\frac{\sqrt{x^{2}+a^{2}}+\sqrt{x^{2}-a^{2}}}{\sqrt{x^{2}+a^{2}}-\sqrt{x^{2}-a^{2}}}\quad (x>a)$ 2084 | \item $\frac{\sqrt{3}}{\sqrt{8}-\sqrt{12}-3 \sqrt{2}}$ 2085 | \item $\frac{1}{\sqrt{2}+\sqrt{5}-\sqrt{7}}$ 2086 | \item $\frac{1-\sqrt{2}+\sqrt{3}}{1+\sqrt{2}-\sqrt{3}}$ 2087 | \end{multicols} 2088 | 2089 | \end{enumerate} 2090 | 2091 | \item 计算下列各式: 2092 | \begin{multicols}{2} 2093 | \begin{enumerate} 2094 | \item $\frac{\sqrt{2}+\sqrt{3}}{3+\sqrt{3}}+\frac{2-\sqrt{3}}{\sqrt{3}+1}$ 2095 | \item $\frac{1}{\sqrt{3}+\sqrt{2}}+\frac{1}{\sqrt{2}-1}-\frac{2}{\sqrt{3}+1}$, 2096 | \item $\frac{\sqrt{y+1}-\sqrt{y}}{\sqrt{y+1}+\sqrt{y}}-\frac{\sqrt{y-1}+\sqrt{y}}{\sqrt{y-1}-\sqrt{y}}$ 2097 | \item $\frac{1}{x+\sqrt{1+x^{2}}}\left(1+\frac{x}{\sqrt{1+x^{2}}}\right)$. 2098 | \end{enumerate} 2099 | \end{multicols} 2100 | 2101 | 2102 | \item 解下列方程: 2103 | \begin{multicols}{2} 2104 | \begin{enumerate} 2105 | \item $\sqrt{x-1} \cdot \sqrt{x+1}=\sqrt{x+5}$ 2106 | \item $\sqrt{x^{2}-3 x+1}+7=2 x$ 2107 | \item $\sqrt{1-x}+\sqrt{12+x}=5$ 2108 | \item $\sqrt{x+8}-\sqrt{5 x+20}+2=0$ 2109 | \item $\sqrt{y-1}-\sqrt{y+2}=\sqrt{5 y-1}$ 2110 | \item $\sqrt{3+\sqrt{x}}=\sqrt{9-5 \sqrt{x}}$ 2111 | \end{enumerate} 2112 | \end{multicols} 2113 | 2114 | 2115 | 2116 | \item 解下列方程: 2117 | \begin{enumerate} 2118 | \item $\frac{\sqrt{y}+\sqrt{y-3}}{\sqrt{y}-\sqrt{y-3}}=2 y-5$ 2119 | \item $\frac{\sqrt{2 x+1}+\sqrt{2 x-1}}{\sqrt{2 x+1}-\sqrt{2 x-1}}=2 x+3$ 2120 | \item $\frac{x+\sqrt{x^{2}-a^{2}}}{x-\sqrt{x^{2}-a^{2}}}-\frac{x-\sqrt{x^{2}-a^{2}}}{x+\sqrt{x^{2}-a^{2}}}-4 \sqrt{x^{2}-a^{2}}=0\quad (a \neq 0)$ 2121 | \end{enumerate} 2122 | 2123 | \item 解下列方程: 2124 | \begin{enumerate} 2125 | \item $7 x^{2}-4 \sqrt{7 x^{2}+1}-31=0$ 2126 | \item $x^{2}-3 x+\sqrt{2 x^{2}-7 x+6}=\frac{x}{2}-3$ 2127 | \item $3 x^{2}-6 x-2 \sqrt{x^{2}-2 x+4}+4=0$ 2128 | \item $\sqrt{\frac{x-1}{x+1}}+\sqrt{\frac{x+1}{x-1}}=3 \frac{1}{3}$ 2129 | \end{enumerate} 2130 | 2131 | \end{enumerate} 2132 | 2133 | 2134 | \section*{本章内容要点} 2135 | 2136 | 本章是在学习多项式(整式)的基础上,进一步学习了分式及其运算、根式及其运算和分式方程、根式方程. 2137 | 2138 | 一、多项式,分式,根式等,都是含有数字和字 2139 | 母并涉及加、减、乘、除、乘方、开平方六种代数运算的式子,这些式子统称为代数式.其中,凡只涉及字母的加、减、乘、乘方运算的式子,叫做多项式 2140 | (整式);凡涉及字母的除法运算且字母含在除式中的式子,叫做分式; 2141 | 分式与整式,又统称为有理式. 2142 | 2143 | 凡涉及字母或数字的开方运算的式子叫做根式,根号内含有字母的根式,又称为关于这个字母的无理 2144 | 式.如$\sqrt{2}$是根式,但不是无理式,$\sqrt{x}$,$\sqrt{x-1}$,$\frac{x}{\sqrt{x-2}}$ 2145 | 等都是无理式. 2146 | 2147 | 关于代数式的概念,可以列表如下: 2148 | \[ 2149 | \text{代数式}\begin{cases} 2150 | \text{有理式} & \begin{cases} 2151 | \text{多项式(即整式)}\\\text{分式} 2152 | \end{cases}\\ 2153 | \text{无理式} 2154 | \end{cases} 2155 | \] 2156 | 2157 | 二、如果有多项式$f(x),g(x)$且$g(x)$的次数大于 2158 | 零次,那么分式$\frac{f (x)}{g (x)}$有以下基本性质: 2159 | \[\frac{f (x) \cdot h (x)}{g(x)\cdot h(x)}=\frac{f(x)}{g (x)}=\frac{f (x) \div h (x)}{g(x)\div h(x)}\] 2160 | 其中,$h(x)$是非零多项式. 2161 | 2162 | 利用基本性质,可以进行分式的通分和约分.分式的四则运算和分数的四则运算是一样的. 2163 | 2164 | 三、表示平方根的式子,叫做二次根式. 2165 | 2166 | 二次根式有以下基本性质: 2167 | \begin{itemize} 2168 | \item $\left(\sqrt{a}\right)^2=1\quad (a\ge 0)$ 2169 | \item $\sqrt{a^2}=|a|$ 2170 | \item $\sqrt{ab}=\sqrt{a}\cdot \sqrt{b}\quad (a\ge 0,\; b\ge 0)$ 2171 | \item $\sqrt{a\div b}=\sqrt{a}\div \sqrt{b} \quad (a\ge 0,\; b>0)$ 2172 | \end{itemize} 2173 | 2174 | 利用基本性质,二次根式可以进行以下变形: 2175 | \begin{enumerate} 2176 | \item 因式的内移与外移,即 2177 | \[\begin{split} 2178 | m\sqrt{a}&=\sqrt{am^2}\quad (m>0)\\ 2179 | \sqrt{a^2m}&=a\sqrt{m}\quad (a>0) 2180 | \end{split}\] 2181 | \item 化去根号内的分母或化去分母中的根号——都是有理化分母的内容,即 2182 | \[\sqrt{\frac{a}{b}}=\sqrt{\frac{a\x b}{b\x b}}=\frac{\sqrt{ab}}{b}\qquad (b>0)\] 2183 | 或 2184 | \[\sqrt{\frac{a}{b}}=\frac{\sqrt{a}}{\sqrt{b}}=\frac{\sqrt{a}\cdot \sqrt{b}}{\sqrt{b}\cdot \sqrt{b}}=\frac{\sqrt{ab}}{b}\qquad (b>0)\] 2185 | \end{enumerate} 2186 | 2187 | 如果一个二次根式符合条件: 2188 | \begin{enumerate} 2189 | \item 被开方各因数的指数小于2; 2190 | \item 根号内不含分母(即分母已经有理化). 2191 | \end{enumerate} 2192 | 那么这个二次根式就叫做最简二次根式. 2193 | 2194 | 如果几个二次根式化为最简根式以后,根号内的式子相同,那么,这几个二次根式就叫做同类根式.同类根式和同类项一样可以合并. 2195 | 2196 | 二次根式的四则运算和多项式的运算很类似.只 2197 | 要注意化为最简根式和合并同类根式就行了. 2198 | 2199 | 四、分式方程与根式方程的解法要点是:设法转 2200 | 化为整式方程求解.由于它们的特点不同,转化方法也就不同. 2201 | 2202 | 分式方程的特点是:分母中含有未知数.因而,要利用分式的基本性质或等式的基本性质,两边乘以同一个整式(一般是取各分母的最低公倍式),约简后 2203 | 转化为一个整式方程. 2204 | 2205 | 根式方程的特点是:根号内含有未知数,因而,就要方程两边同次乘方(如:同平方)后,利用根式的基本性质转化为有理方程,并进而转化为整式方程. 2206 | 2207 | 但是,一定要注意,在解分式方程与根式方程的过程中,由于各自的转化方式都能引起未知数允许取值范围的扩大,所以都可能产生增根.因此,无论是解分式方程,还是解根式方程,最后的验根都是不可缺少的,验根,将起到“识别真假”“去伪存真”的作用. 2208 | 2209 | 五、已学过的方程有:整式方程、分式方程、根 2210 | 式方程,统称为代数方程.其系统可列表如下: 2211 | \begin{center} 2212 | \begin{tikzpicture}[yscale=.6] 2213 | \node at (6,3)[right]{一次方程}; 2214 | \node at (6,2)[right]{二次方程}; 2215 | \node at (6,1)[right]{高次方程}; 2216 | \node at (4,2)[right]{整式方程}; 2217 | \node at (4,0)[right]{分式方程}; 2218 | \node at (2,1)[right]{有理方程}; 2219 | \node at (2,-1)[right]{根式方程}; 2220 | \node at (0,0)[right]{代数方程}; 2221 | \draw[decorate,decoration=brace, thick](6,1)--(6,3); 2222 | \draw[decorate,decoration=brace, thick](4,0)--(4,2); 2223 | \draw[decorate,decoration=brace, thick](2,-1)--(2,1); 2224 | \end{tikzpicture} 2225 | \end{center} 2226 | 2227 | \section*{复习题六} 2228 | \addcontentsline{toc}{section}{复习题六} 2229 | \begin{enumerate} 2230 | \item 约简下列各分式: 2231 | \begin{multicols}{2} 2232 | \begin{enumerate} 2233 | \item $\frac{\poly{1,1,0,-1,-1}}{\poly{1,-1,0,1,-1}}$ 2234 | \item $\frac{\poly{1,-2,0,-2,-1}}{\poly{1,0,-2,-2,-3,-2}}$ 2235 | \end{enumerate} 2236 | \end{multicols} 2237 | 2238 | \item 判别$\frac{\poly{1,1,1,1}}{\poly{1,-1,-1,-1}}$是不是最简分式,为什么? 2239 | %$\frac{\poly{}}{\poly{}}$ 2240 | 2241 | \item 计算下列各式: 2242 | \begin{enumerate} 2243 | \item $\left(\frac{x}{y}-\frac{y}{x}\right) \div(x+y)+x\left(\frac{1}{y}-\frac{1}{x}\right)$ 2244 | \item $\frac{m^{2}+n^{2}}{m^{2}+2 m n+n^{2}}+\frac{2}{m n} \div\left(\frac{1}{m}+\frac{1}{n}\right)^{2}$ 2245 | \begin{multicols}{2} 2246 | \item $\frac{1}{1-\frac{1}{1-\frac{1}{x}}}$ 2247 | \item $1+\frac{1}{2+\frac{1}{3+\frac{1}{x}}}$ 2248 | \end{multicols} 2249 | \end{enumerate} 2250 | 2251 | 2252 | \item \begin{enumerate} 2253 | \item 已知 $a=-2, b=-1$. 求 $\left(a-\frac{a^{2}}{a+b}\right)\left(\frac{a}{a-b}-1\right)\div \frac{b^{2}}{a+b}$ 的值. 2254 | \item 已知 $x=-2$, $y=\frac{1}{3}$, 求 $\frac{4 x^{2}+12 x y+9 y^{2}-16}{4 x^{2}-9 y^{2}-4(2 x-3 y)}$ 的值. 2255 | \end{enumerate} 2256 | 2257 | \item 解下列方程: 2258 | \begin{enumerate} 2259 | \item $\frac{3}{1+3 x}\left(x-\frac{x}{1+x}\right)+\frac{x}{1+x}\left(1+\frac{1}{1+3 x}\right)=1$ 2260 | \item $\frac{1}{x(x+1)}+\frac{1}{(x+1)(x+2)}+1=0$ 2261 | \item $x^{2}+5 x-5=\frac{6}{x^{2}+5 x}$ 2262 | \end{enumerate} 2263 | 2264 | \item 解下列关于 $x$ 的方程: 2265 | \begin{enumerate} 2266 | \item $\frac{5 a^{2}}{4 x^{2}-a^{2}}=\frac{2 x}{2 x-a}-\frac{x}{2 x+a}$ 2267 | \item $\frac{x}{3 a+x}-\frac{x}{x-3 a}=\frac{a^{2}}{9 a^{2}-x^{2}}$ 2268 | \end{enumerate} 2269 | 2270 | \item 当$a$取何值时,下列各式的值最小? 2271 | \[\sqrt{8+a},\qquad \sqrt{4-a^2},\qquad \sqrt{16-a} \] 2272 | 2273 | \item \begin{enumerate} 2274 | \item 已知$x=25$, $y=15$,计算: 2275 | \[\sqrt{x^3+x^2y+\frac{1}{4}xy^2}+\sqrt{\frac{1}{4}x^3+x^2y+xy^2} \] 2276 | \item 已知$x=\frac{2ab}{b^2+1}$,其中$a,b$都是正数, 2277 | 2278 | 求 $\frac{\sqrt{a+x}+\sqrt{a-x}}{\sqrt{a+x}-\sqrt{a-x}}$的值. 2279 | \end{enumerate} 2280 | 2281 | \item 已知 $\sqrt{3} \approx 1.732$, 求 $\frac{3+\sqrt{6}}{\sqrt{3}+\sqrt{2}}$ 的值. 2282 | \item 化简下列各式: 2283 | \begin{enumerate} 2284 | \item $y=\sqrt{x+2 \sqrt{x-1}}+\sqrt{x-2 \sqrt{x-1}}\qquad (x \geqslant 1)$ 2285 | \item $\sqrt{3+2 \sqrt{5+12 \sqrt{3+2 \sqrt{2}}}}$ 2286 | \end{enumerate} 2287 | 2288 | \item 设 $x=\sqrt{3+\sqrt{5}},\quad y=\sqrt{3-\sqrt{5}}$, 2289 | 2290 | 求 $\frac{x+y}{x-y}-\frac{x-y}{x+y}$-的值. 2291 | \item 关于 $x$ 的二次方程 $(a+1)\left(x^{2}-x\right)=(a-1)(x-2)$ 2292 | 的两个根互为相反数, 取其正根求 $\sqrt{4 x^{2}-12 x+9}$ 的值. 2293 | \item 2294 | \begin{enumerate} 2295 | \item 若 $a+\frac{1}{b}=1, \quad b+\frac{1}{c}=1$. 求证 $a b c+1=0$. 2296 | \item 若 $\frac{y}{x}+\frac{x}{z}=a,\quad \frac{z}{y}+\frac{y}{x}=b,\quad \frac{x}{z}+\frac{z}{y}=c$ 2297 | 2298 | 求证 $(a+b-c)(a-b+c)(-a+b+c)=8$. 2299 | \end{enumerate} 2300 | 2301 | 2302 | \item 解方程 $\frac{x^{3}+2}{x^{2}-x+1}+\frac{x^{3}-2}{x^{2}+x+1}=2 x$ 2303 | \item 试证明: 2304 | \begin{enumerate} 2305 | \item 两个真分式的和仍是真分式; 2306 | \item 两个真分式的差也是真分式. 2307 | \end{enumerate} 2308 | 2309 | \end{enumerate} 2310 | 2311 | -------------------------------------------------------------------------------- /7.tex: -------------------------------------------------------------------------------- 1 | \chapter{代数运算的初步应用} 2 | 3 | 我们讨论了多项式的基本运算,而这些运算的应用很广泛,在第二章、第三章中解代数方程式的原理,实质上就是多项式的运算.本章就多项式运算的另一些初步应用加以讨论,以扩大同学们的视野,进一步体会代数运算的基本精神和方法. 4 | \section{求和公式} 5 | \subsection{等差数列求和} 6 | 在第一章中,我们利用运算性质,曾经简捷地计算过: 7 | \[\begin{split} 8 | 1+2+3+\cdots+100&=5050\\ 9 | 1+3+5+\cdots+99&=2500 10 | \end{split}\] 11 | 12 | 在这两串数$1, 2,\ldots,100$和$1, 3, 5,\ldots,99$中都有一个特点:从第二个数起;每个数与它前边的一个数 13 | 的差都相等.象第一个数串中,这个差为1;第二个数串中,这个差为2. 14 | 15 | 象这样有次序地排好的一串数,其中任一数与它前一个数的差都相等.我们就把这样一串数叫做\textbf{等差数列}.这个“相等的差”,叫做这个数列的\textbf{公差}.数列中的每一个数,叫做这个数列的一项.排头的一个数,叫首项,排尾的一个数叫末项. 16 | 17 | 可见,\textbf{等差数列的任一项,都应等于它前面的一项加上公差}. 18 | 19 | 若用$a_1$表示等差数列的首项,$d$表示公差,那么这个等差数列的每一项可写成: 20 | \begin{center} 21 | \begin{tikzpicture}[xscale=1.6] 22 | \foreach \x/\xtext in {0/a_1, 1/a_1+d, 2/a_1+2d, 3/\cdots, 4/a_1+99d} 23 | { 24 | \node at (\x,0){$\xtext$,}; 25 | } 26 | \node at (5,0){$\cdots$.}; 27 | \foreach \x/\xtext in {0/首项,, 1/第二项,, 2/第三项,, 3/…,, 4/第100项,, 5/….} 28 | { 29 | \node at (\x,-1){\xtext}; 30 | 31 | } 32 | 33 | \foreach \x in {0,1,2,4} 34 | { 35 | \draw (\x,-.25)--(\x,-0.75); 36 | } 37 | \end{tikzpicture} 38 | \end{center} 39 | 40 | 一般地,这个数列的第$n$项可表为: 41 | \[a_n=a_1+ (n-1) d\] 42 | 43 | 由此,一个等差数列,只要知道它的首项和公差,就可以写出它的任何一项来. 44 | 45 | \begin{example} 46 | \begin{enumerate} 47 | \item 写出首项为2,公差为5的等差数列的各项; 48 | \item 写出首项为2,公差为$-1$的等差数列的 49 | 各项. 50 | \end{enumerate} 51 | \end{example} 52 | 53 | \begin{solution} 54 | \begin{enumerate} 55 | \item 首项为2, 公差为5的等差数列各项为:$2, 7, 12, \ldots, 2+5 (n-1) ,\ldots$ 56 | \item 首项为2, 公差为$-1$的等差数列的各项为: 57 | $2, 1, 0, -1, -2, \ldots, 2+ (n-1)\cdot(-1) ,\ldots$ 58 | \end{enumerate} 59 | \end{solution} 60 | 61 | \begin{example} 62 | 如果一等差数列的首项是5,公差是2,那么它的第10项,第15项各是多少? 63 | \end{example} 64 | 65 | \begin{solution} 66 | $\because\quad $ 第$n$项为$a_n=a+(n-1)d$ 67 | 68 | 这里$a_1=5$, $d=2$, $n=10, 15$. 69 | 70 | $\therefore\quad $ 第10项应为: 71 | \[\begin{split} 72 | a_{10}&=a_1+ (10-1) d\\ 73 | &=5+9\x2=23 74 | \end{split}\] 75 | 同样,第15项应为: 76 | \[a_{15}=5+ (15-1) \x2=33\] 77 | \end{solution} 78 | 79 | \begin{ex} 80 | \begin{enumerate} 81 | \item 写出下列等差数列的公差,并求出它的第100项是多少? 82 | 如何表示它的第n项? 83 | \begin{enumerate} 84 | \item $3, -1, -5, -9, \ldots$ 85 | \item $5, 7, 9, 11, \ldots$ 86 | \end{enumerate} 87 | \item 如果等差数列的第10项是100, 公差是10, 你能知道这个 88 | 数列的首项是多少吗? 89 | \item 如果等差数列的首项是2, 第10项是20, 你能知道这个数 90 | 列的公差是多少吗? 91 | \item 如果以1为首项,2为末项,那么在中间插入9项,构 92 | 成一个等差数列.你能写出这个数列的各项来吗? 93 | \end{enumerate} 94 | \end{ex} 95 | 96 | 等差数列前$n$项的和如何求呢?下边我们就利用 97 | 数系运算的通性,导出它的求和公式. 98 | 99 | \begin{example} 100 | 求$1+2+3+\cdots+n$ 101 | \end{example} 102 | 103 | \begin{solution} 104 | 设 105 | \begin{equation} 106 | 1+2+3+\cdots+n=S 107 | \end{equation} 108 | 则利用交换律,可以改写为 109 | \begin{equation} 110 | n+\cdots+3+2+1=S 111 | \end{equation} 112 | 将等式(7.1),(7.2)相加,得 113 | \[\underbrace{(1+n)+\cdots +[(n-2)+3]+[(n-1)+2]+(n+1)}_{n\text{项}}=2S\] 114 | 即:$n(n+1)=2S$,$\qquad \therefore\quad S=\frac{n(n+1)}{2}$ 115 | \end{solution} 116 | 117 | 118 | 因此: 119 | \[1+2+3+\cdots+n=\frac{n(n+1)}{2}\] 120 | 121 | 当$n=100$时,就是: 122 | \[1+2+3+\cdots+100=\frac{100(100+1)}{2}=5050\] 123 | 124 | 一般地,对于等差数列$a_1,a_1+d,a_1+2d,\ldots,a_1+(n-1)d,\ldots$的前$n$项求和公式,可作如下推导: 125 | 126 | 设$S=a_1+(a_1+d)+(a_1+2d)+\cdots+[a_1+(n-1)d]$,又 127 | $S=[a_1+(n-1)d]+[a_1+(n-2)d]+[a_1+(n-3)d]+\cdots+a_1$,两式相加,可得: 128 | \[2S=\underbrace{[2a_1+(n-1)d]+[2a_1+(n-1)d]+\cdots+[2a_1+(n-1)d]}_{n\text{项}}\] 129 | 即:$2S=n[2a_1+(n-1)d]$,$\qquad \therefore\quad S=\frac{n[2a_1+(n-1)d]}{2}$ 130 | 131 | 这就是\textbf{等差数列前$n$项和的公式}. 132 | 133 | 其中,$a$为首项,可以是任意数;$d$为公差,可以是任意数;$n$为项数,只能是自然数. 134 | 135 | \begin{example} 136 | 求$1+3+5+\cdots+(2n-1)$ 137 | \end{example} 138 | 139 | \begin{solution} 140 | 这里$a_1=1$, $d=2$, 共$n$项,因此: 141 | \[S=\frac{n[2+(n-1)\x 2]}{2}=n^2 \] 142 | 即:$1+3+5+\cdots+(2n-1)=n^2$. 143 | \end{solution} 144 | 145 | \begin{example} 146 | 如果等差数列的首项为3,前25项的和为1000,试求这个数列的公差是多少? 147 | \end{example} 148 | 149 | \begin{solution} 150 | 将 $a_1=3$, $n=25$, $S=1000$, 都代入求和 151 | 公式: 152 | \[S=\frac{n [2a_1+ (n-1) d]}{2}\] 153 | 可得:$1000=\frac{25(6+24d)}{2}$. 154 | 155 | $\therefore\quad d=\frac{74}{24}=\frac{37}{12}$ 156 | \end{solution} 157 | 158 | \begin{example} 159 | 试求:$\frac{1}{2}+1+1\frac{1}{2}+\cdots+100$ 160 | \end{example} 161 | 162 | \begin{analyze} 163 | 这个和式中的各项,显然是一个等差数 164 | 列,其首项为$\frac{1}{2}$, 公差为$\frac{1}{2}$, 165 | 末项为100, 但一共是 166 | 多少项呢?可以由等差数列的第$n$项公式 $a_n=a_1+(n-1)d$求出来. 167 | 168 | \end{analyze} 169 | 170 | \begin{solution} 171 | 将首项$a_1=\frac{1}{2}$公差$d=\frac{1}{2}$以及末项 172 | $a_n=100$都代入公式$a_n=a_1+(n-1)d$中,得 173 | \[100=\frac{1}{2}+(n-1)\x \frac{1}{2}\] 174 | 解出$n=200$,这就是说,和式中共有200项.因此: 175 | \[\begin{split} 176 | S&=\frac{n[2a_1+(n-1)d]}{2}\\ 177 | &=\frac{200\left[2\x\frac{1}{2}+199\x \frac{1}{2}\right]}{2}\\ 178 | &=100+199\x 50=10050 179 | \end{split}\] 180 | \end{solution} 181 | 182 | \begin{ex} 183 | \begin{enumerate} 184 | \item 求 $2+4+6+\cdots+50$ 185 | \item 求 $7+5+3+\cdots+(-101)$ 186 | \item 求 $1+(1+n)+(1+2n)+\cdots+(1+n^2-n)$ 187 | \end{enumerate} 188 | \end{ex} 189 | 190 | 如果把等差数列前$n$项和的公式变形,又可以得 191 | 出: 192 | \[\begin{split} 193 | S&=\frac{n[2a_1+(n-1)d]}{2}\\ 194 | &=\frac{n[a_1+a_1+(n-1)d]}{2} 195 | \end{split}\] 196 | $\therefore\quad S=\frac{n(a_1+a_n)}{2}$ 197 | 198 | 因此,等差数列的求和公式,又可以叙述为: 199 | 等差数列前$n$项和,等于首项加末项,乘以项数,再除以2. 200 | 201 | \subsection{等比数列求和} 202 | 如果在一串按次序排好的数中,从第二个数起,每一数与它前一个数的比都相等.那么,这串数就叫做\textbf{等比数列}.这个相等的比值,叫这个数列的\textbf{公比}.其中的每一个数,叫做等比数列的一项,头一项称为 203 | 首项,最后一项称为末项. 204 | 205 | 不难看出,等比数列的任一项,都应等于它的前一项乘以公比. 206 | 207 | 若以$a_1$表示首项,$q$表示公比,则可以写出等比数列的每一项分别是: 208 | \begin{center} 209 | \begin{tikzpicture}[xscale=1.5] 210 | \foreach \x/\xtext in {0/a_1, 1/a_1q, 2/a_1q^2, 3/\cdots, 4/a_1q^9, 5/\cdots, 6/a_1q^{100}} 211 | { 212 | \node at (\x,0){$\xtext$,}; 213 | } 214 | \node at (7,0){$\cdots$.}; 215 | \foreach \x/\xtext in {0/第一项,, 1/第二项,, 2/第三项,, 3/…,, 4/第10项,, 5/…,, 6/第101项,, 7/….} 216 | { 217 | \node at (\x,-1){\xtext}; 218 | 219 | } 220 | 221 | \foreach \x in {0,1,2,4,6} 222 | { 223 | \draw (\x,-.25)--(\x,-0.75); 224 | } 225 | \end{tikzpicture} 226 | \end{center} 227 | 228 | 一般地,等比数列的第$n$项是: 229 | \[a_n=a_1\cdot q^{n-1}\] 230 | 231 | \begin{example} 232 | 写出首项是2,公比分别是2,$-1$的等比数列. 233 | \end{example} 234 | 235 | \begin{solution} 236 | 首项是2, 公比是2的等比数列是: 237 | \[2, 4, 8, \ldots , 2^n, \ldots \] 238 | 首项是2, 公比是$-1$的等比数列是: 239 | \[2, -2, 2, \ldots, (-1)^{n-1}\cdot 2,\ldots\] 240 | \end{solution} 241 | 242 | \begin{example} 243 | 一等比数列的首项为3, 公比是$\frac{1}{2}$,求它 244 | 的第10项及第20项. 245 | \end{example} 246 | 247 | \begin{solution} 248 | $\because\quad a_n=a_1q^{n-1}$. 249 | 250 | 当$n=10$时可得第10项: 251 | \[a_{10}=3\x\left(\frac{1}{2}\right)^9=\frac{3}{2^9}=\frac{3}{512}\] 252 | 253 | 当$n=20$时可得第20项: 254 | \[a_{20}=3\x\left(\frac{1}{2}\right)^{19}=\frac{3}{2^{19}}=\frac{3}{524288}\] 255 | \end{solution} 256 | 257 | 258 | \begin{ex} 259 | \begin{enumerate} 260 | \item 写出下列等比数列的公比及第100项、第$n$项. 261 | \begin{enumerate} 262 | \item $4, 12, 36, 108,\ldots$ 263 | \item $5, -10, 20,-40,\ldots$ 264 | \item $2, 1, \frac{1}{2}, \frac{1}{4},\ldots$ 265 | \end{enumerate} 266 | 267 | \item 如果给你两个数4与25,你能在它们中间插入一个数, 268 | 使这三个数成等比数列吗?它们的公比是多少? 269 | \end{enumerate} 270 | \end{ex} 271 | 272 | \begin{example} 273 | 求$3+3\cdot 4+3\cdot 4^2+\cdots+3\cdot 4^{n-1}$ 274 | \end{example} 275 | 276 | \begin{solution} 277 | 这是一个等比数列的前$n$项求和的问题,可设:$$S=3+3\x4+3\x4^2+\cdots+3\x4^{n-1}$$ 278 | 则有 279 | \[4S=3\x4+3\x4^2+\cdots+3\x4^{n-1}+3\x4^n\] 280 | 两式相减得: 281 | \[\begin{split} 282 | (4-1) S&=3\x4^n-3\\ 283 | S&=\frac{3(4^n-1)}{4-1}=4^n-1 284 | \end{split}\] 285 | \end{solution} 286 | 287 | 一般地,对于等比数列$a_1,a_1q,a_1q^2,\ldots,a_1q^{n-1},\ldots$的前$n$项求和,我们可作如下推导: 288 | 289 | 设$S=a_1+a_1q+a_1q^2+\cdots+a_1q^{n-1}$,则: 290 | \[qS=a_1q+a_1q^2+\cdots+a_1q^{n-1}+a_1q^n\] 291 | 两式相减:$(1-q)S=a_1-a_1q^n$. 292 | 293 | 当$q\ne 1$时, 294 | \[S=\frac{a_1-a_1q^n}{1-q}=\frac{a_1(1-q^n)}{1-q}\] 295 | 这就是等比数列的前$n$项求和公式. 296 | 297 | 而当$q=1$时,这个等比数列为:$a_1,a_1,a_1,\ldots,a_1,\ldots$ 298 | 299 | 显然前$n$项和 $S=na_1$. 300 | 301 | \begin{example} 302 | 求$1+\frac{1}{3}+\frac{1}{9}+\frac{1}{27}$ 303 | \end{example} 304 | 305 | \begin{solution} 306 | $\because\quad a_1=1,\quad q=\frac{1}{3},\quad n=4$ 307 | 308 | $\therefore\quad S=\frac{a_1(1-q^n)}{1-q}=\frac{1-\frac{1}{3^4}}{1-\frac{1}{3}}=\frac{\frac{80}{81}}{\frac{2}{3}}=\frac{40}{27}$ 309 | \end{solution} 310 | 311 | \begin{example} 312 | 如果等比数列前两项的和是$1\frac{1}{2}$,而这两项的差是$\frac{1}{2}$,试求这数列的前10项和. 313 | \end{example} 314 | 315 | 316 | \begin{solution} 317 | 设等比数列的首项为$a_1$,第二项为$a_1q$,则由已知得: 318 | \[\begin{cases} 319 | a_1+a_1q=1\frac{1}{2}\\ 320 | a_1-a_1q=\frac{1}{2} 321 | \end{cases}\] 322 | 两式相加得:$2a_=2,\qquad \therefore\quad a_1=1$ 323 | 324 | 两式相减得:$2a_1q=1,\qquad \therefore\quad a_1q=\frac{1}{2}$ 325 | 326 | 已知$a_1=1$,$\qquad \therefore\quad q=\frac{1}{2}$. 327 | 328 | 因此,前10项和应为: 329 | \[\begin{split} 330 | S_{10}=\frac{a_1(1-q^{10})}{1-q}&=\frac{1\left(1-\frac{1}{2^{10}}\right)}{1-\frac{1}{2}}\\ 331 | &=\frac{2^{10}-1}{2^9}=\frac{1023}{512} 332 | \end{split}\] 333 | \end{solution} 334 | 335 | 336 | \begin{ex} 337 | 求和: 338 | \begin{enumerate} 339 | \item $3+6+12+24+\cdots+3072$ 340 | \item $1-2+4-8+\cdots-512+1024$ 341 | \item $1+x+x^2+\cdots+x^{n-1}\qquad (x\ne 1)$ 342 | \end{enumerate} 343 | \end{ex} 344 | 345 | \section*{习题7.1} 346 | \addcontentsline{toc}{subsection}{习题7.1} 347 | \begin{enumerate} 348 | \item 求下列等差数列的公差及第10项、第$n$项. 349 | \begin{enumerate} 350 | \item $5,\; 11,\; 17,\; 23, \ldots $ 351 | \item $2,\; 5,\; 8,\; 11, \ldots $ 352 | \item $-13,\; -11,\; -9,\; -7, \ldots $ 353 | \item $-\frac{3}{2},\; -\frac{2}{3},\; \frac{1}{6}, \; 1, \ldots $ 354 | \item $(4 a-3 b),\; (2 a+b),\; 5 b,\; (9 b-2 a), \ldots$ 355 | \item $(a+3 m),\; (a+m),\; (a-m),\; (a-3 m) \ldots $ 356 | \end{enumerate} 357 | \item 求下列等差数列指定的部分和: 358 | \begin{enumerate} 359 | \item $2,\; 7,\; 12, \ldots $ 的前 10 项; 360 | \item $15,\; 8,\;1, \ldots $ 的前15项; 361 | \item $\frac{1}{6},\; \frac{4}{3},\; \frac{5}{2}, \ldots $ 的前 14 项; 362 | \item $(x-y),\; x,\;(x+y), \ldots$ 的前30项; 363 | \item $(x-y),\; y,\;(-x+3 y), \ldots$ 的前 25 项. 364 | \end{enumerate} 365 | 366 | \item 如果已知等差数列的首项 $a_{1}$, 项数 $n$, 末项 $a_{n}$, 试求这些 数列的和 $S_{n}$. 367 | \begin{enumerate} 368 | \item $a_{1}=3,\quad a_{13}=61$, 369 | \item $a_{1}=15,\quad a_{11}=-25$, 370 | \item $a_{1}=3.14,\quad a_{22}=5.68$ 371 | \end{enumerate} 372 | 373 | \item 在下列各题中,插入指定个数的中间项,使它们与给定两 374 | 数成等差数列. 375 | \begin{enumerate} 376 | \item 在$-9$与5之间,插入一个数; 377 | \item 在7与25之间,插入5个数; 378 | \item 在13与$-5$之间,插入4个数; 379 | \item 在28与$-26$之间,插入8个数. 380 | \end{enumerate} 381 | 382 | \item 求下列等比数列的公比及第10项,第$n$项. 383 | \begin{enumerate} 384 | \item $4,\; 4,\; 4,\; 4,\ldots$ 385 | \item $4,\; 12,\; 36,\; 108, \ldots$ 386 | \item $-24,\; 12,\; -6,\; 3,\; -\frac{3}{2},\ldots$ 387 | \item $5x^3,\; 10x^2,\; 20x,\; 40, \ldots \qquad (x\ne 1)$ 388 | \item $\frac{x}{y},\; x,\; xy,\; xy^2,\ldots\qquad (y\ne 1)$ 389 | \end{enumerate} 390 | 391 | \item 求下列等比数列的指定部分和: 392 | \begin{enumerate} 393 | \item $1,\; 3,\; 9,\; 27,\ldots$的前10项; 394 | \item $2,\; -2,\; 2,\;-2,\ldots$的前101项; 395 | \item $625,\; 125,\; 25,\; 5,\; 1,\ldots$的前10项; 396 | \item $56,\;-28,\; 14,\;-7,\ldots$的前10项. 397 | \end{enumerate} 398 | 399 | \item 试在$\frac{1}{2}$与$\frac{9}{8}$之间,插入一个数,使它们成等比数列.并 400 | 求出它们的公比. 401 | 402 | \item 如果有一数列是:$100,\; 95,\; 90,\; 85,\ldots$ 403 | 你能知道这一数列的前多少项和最大?并求出这个最大值. 404 | 405 | \end{enumerate} 406 | 407 | \section{待定系数法} 408 | 待定系数法是一种重要的数学方法.在本册书的前几章内容中,已经利用待定系数法解决过许多问题,例如:多项式除法,分解因式,寻求根与系数的关系等.本节将在此基础上,进一步说明待定系数法的意义和原理,以及它在代数里的其它应用. 409 | 410 | \subsection{待定系数法及其根据} 411 | 先从我们已经熟悉的具体例子谈起. 412 | 413 | 414 | \begin{example} 415 | 试求$f(x)=\poly{4,7,6,2}$除以$g(x)=x^2+1$的商式和余式. 416 | \end{example} 417 | 418 | \begin{solution} 419 | 由多项式除法可知,其商式必定是一次 420 | 式,其余式至多是一次式.因而可设 421 | 商式$Q(x)=ax+b$, 余式$R(x)=cx+d$.由除法恒等式,可得 422 | \[ 4x^3+7x^2+6x+2= (ax+b) (x2+1)+ (cx+d)\] 423 | 即:$4x^3+7x^2+6x+2=ax^3+bx^2+(a+c)x+b+d$ 424 | 425 | 比较两边同类项系数,得 426 | \[\begin{cases} 427 | a=4\\ 428 | b=7\\ 429 | a+c=6\\ 430 | b+d=2 431 | \end{cases}\] 432 | 解这个方程组,得 433 | \[a=4,\quad b=7,\quad c=2,\quad d=-5\] 434 | 435 | 因此,$f(x)$除以$g(x)$的商式$Q(x)$、余式$R(x)$分别为: 436 | \[Q (x) =4x+7,\qquad R (x) =2x-5\] 437 | \end{solution} 438 | 439 | \begin{example} 440 | 已知多项式$ax^3+bx^2+cx+d$能被$x^2+p$整除,求证:$ad=bc$. 441 | \end{example} 442 | 443 | \begin{analyze} 444 | 只要根据已知条件,设法建立恒等关系,从中找出已知系数$a,b,c,d$之间的关系,就可达到目的. 445 | \end{analyze} 446 | 447 | \begin{proof} 448 | 由于三次多项式$ax^3+bx^2+cx+d$, 能被二次式$x^2+p$整除,因而,其商式必为一次式,不妨设商式为$mx+n$ ($m,n$为待定系数).这样,可以得出恒等式: 449 | \[ax^3+bx^2+cx+d= (mx+n) (x^2+p)\] 450 | 即:$ax^3+bx^2+cx+d=mx^3+nx^2+pmx+pn$ 451 | 452 | 比较等式两边同类项的系数,得 453 | \begin{numcases}{} 454 | a=m\\ 455 | b=n\\ 456 | c=pm\\ 457 | d=pn 458 | \end{numcases} 459 | 利用这个方程组,消去待定系数$m,n$和已知系数$p$, 就可以找出$a,b,c,d$的关系. 460 | 461 | 将(7.3), (7.4)分别代入(7.5), (7.6);再由(7.5), (7.6)可得: 462 | \[\frac{c}{a}=m=\frac{d}{b} \] 463 | $\therefore\quad ad=bc$ 464 | \end{proof} 465 | 466 | 象以上例题的解题方法,叫做待定系数法(或叫未定系数法),这个方法的特点是引进待定系数(未知的),列出一个含有待定系数的恒等式,然后根据多项式恒等的性质,比较等式两边的同类项系数,得出一个方程组.解这个方程组,求出待定系数,或消去待定系数而找出原来已知系数之间所存在的关系,使问题得以解决. 467 | 468 | 这个方法的主要根据是两个多项式恒等的性质,即 469 | 470 | \begin{blk}{定理} 471 | 如果两个多项式恒等,那么,这两个多项式的同类项系数都一定对应相等. 472 | \end{blk} 473 | 474 | 也就是说,如果 475 | $$a_nx^n+a_{n-1}x^{n-1}+\cdots+a_1x+a_0\equiv b_nx^n+b_{n-1}x^{n-1}+\cdots+b_1x+b_0$$ 476 | 那么,$a_n=b_n,\; a_{n-1}=b_{n-1},\ldots, a_1=b_1,\; a_0=b_0$ 477 | 478 | \begin{proof} 479 | 由于$a_nx^n+a_{n-1}x^{n-1}+\cdots+a_1x+a_0\equiv b_nx^n+b_{n-1}x^{n-1}+\cdots+b_1x+b_0$ 480 | 481 | 移项,合并同类项可得: 482 | \[(a_n-b_n)x^n+(a_{n-1}-b_{n-1})x^{n-1}+\cdots + (a_1-b_1)x+(a_0-b_0)\equiv 0\] 483 | 所以,$a_n=b_n,\; a_{n-1}=b_{n-1},\ldots, a_1=b_1,\; a_0=b_0$ 484 | \end{proof} 485 | 486 | \begin{ex} 487 | \begin{enumerate} 488 | \item 多项式$f(x)=x^4-5x^3+11x^2+mx+n$如果能被二次三项式$x^2-2x+1$整除,试求$m,n$的值. 489 | \item 如果$ax^2+bx+c$能被$px+q$整除.试求$a,b,c$和$p,q$之间应具有什么关系? 490 | \end{enumerate} 491 | 492 | \end{ex} 493 | 494 | \subsection{待定系数法的应用} 495 | 待定系数法在代数上有许多应用,除我们已经学习过的“求商式及余式、分解因式、寻求方程的根与系数的关系”等内容外,以下将学习另外一些应用.从中进一步领会这个方法的要点和重要. 496 | 497 | \subsubsection{把多项式表示成另一个多项式的各次幂的形式} 498 | 499 | 在代数中,有时需要将一个多项式,表示成次数较低的另一个多项式的各次幂的形式.例如,在习题4.3第9题中,就是把多项式$3x^3-10x^2+13$表示成$x-2$的各次幂的形式的: 500 | \[3x^3-10x^2+13=A (x-2)^3 +B (x-2)^2+ C (x-2) +D\] 501 | 当时,采用的是逐次使用综合除法的方法,其实这类问题也可以用待定系数法解决. 502 | 503 | 504 | \begin{example} 505 | 试用$(x-1)$的各次幂表示出多项式$2x^3-x^2+2x+3$ 506 | \end{example} 507 | 508 | 509 | \begin{solution} 510 | 设$2x^3-x^2+2x+3=a(x-1)^3+b(x-1)^2+c(x-1)+d$ 511 | 因此: 512 | \[\begin{split} 513 | &2x^3-x^2+2x+3\\ 514 | &\qquad =ax^3-3ax^2+3ax-a+bx^2-2bx+b+cx-c+d 515 | \end{split}\] 516 | 即:$2x^3-x^2+2x+3 =ax^3+(b-3a)x^2+(3a-2b+c)x-a+b-c+d$ 517 | 518 | 比较两边同类项系数,得 519 | \[\begin{cases} 520 | a=2\\ 521 | b-3a=-1\\ 522 | 3a-2b+c=2\\ 523 | -a+b-c+d=3 524 | \end{cases}\] 525 | 解这个方程组,得 526 | \[a=2,\quad b=5,\quad c=6,\quad d=6\] 527 | 因此, 528 | $2x^3-x^2+2x+3 =2(x-1)^3+5(x-1)^2+6(x-1)+6$ 529 | \end{solution} 530 | 531 | 应该指出,待定系数法在解以上这种问题时,并不是最简便的.使用习题3.4第9题所提示的综合除法要简便一些. 532 | 533 | 其实,这类问题如果用换元法变形,会更简便.这就是: 534 | 535 | 设$x-1=y$, 则 536 | $x=y+1$, 代入原多项式中, 537 | 得 538 | \[\begin{split} 539 | 2x^3-x^2+2x+3&=2(y+1)^3-(y+1)^2+2(y+1)+3\\ 540 | &=2y^3+6y^2+6y+2-y^2-2y-1+2y+2+3\\ 541 | &=2y^3+5y^2+6y+6 542 | \end{split}\] 543 | 再将原设$x-1=y$代入上式右边,得: 544 | $$2x^3-x^2+2x+3=2(x-1)^3+5(x-1)^2+6(x-1)+6$$ 545 | 546 | \begin{ex} 547 | 用三种方法解下列各题: 548 | \begin{enumerate} 549 | \item 把$3x^3-10x^2+13$表示成$x-2$的各次幂的形式. 550 | \item 用$(x+1)$的各次幂表示多项式$x^4-1$. 551 | \end{enumerate} 552 | \end{ex} 553 | 554 | \subsubsection{求多项式与求方程的解} 555 | 556 | 如果给出$n$次多项式在$x$取$n+1$个不同数值时所 557 | 对应的值,就可以用待定系数法求出这个多项式的表 558 | 达式,进而还可以求出其它值. 559 | 560 | 561 | 解: 562 | 563 | 564 | \begin{example} 565 | 已知$f(1)=-1$, $f(2)=4$, $f(3)=-3$, 566 | 试求二次多项式$f(x)$的表达式以及$f(10)$. 567 | \end{example} 568 | 569 | \begin{solution} 570 | 设$f(x)=ax^2+bx+c$, 571 | 则由已知条件可知: 572 | \[\begin{cases} 573 | f(1)=a+b+c=-1,\\ 574 | f(2)=4a+26+c=4,\\ 575 | f(3)=9a+36+c=-3. 576 | \end{cases}\] 577 | 解这个方程组,得$a=-6,\quad b=23,\quad c=-18$. 578 | 579 | 因此,所求二次多项式为 580 | \[\begin{split} 581 | f(x)&=-6x^2+23x-18\\ 582 | f(10)&=-388 583 | \end{split}\] 584 | \end{solution} 585 | 586 | 如果给出一元三次或四次方程的一个或两个根, 587 | 那么用除法可以得到一个一元二次方程,进而求出其 588 | 余的两个根.这样的方程也可以利用待定系数法来 589 | 解. 590 | 591 | 592 | \begin{example} 593 | 已知方程$x^4-9x^2+12x-4=0$有两个根 594 | 1与2,试求这个方程的另两个根. 595 | \end{example} 596 | 597 | \begin{analyze} 598 | 由于1与2是已知方程的两个根,根据余 599 | 式定理的推论可知, 600 | $x^4-9x^2+12x-4$ 601 | 含有因式 602 | $(x-1)(x-2)$. 又因为$x^4-9x^2+12x-4$的首项系数 603 | 是1, 所以可设它的另一个因式是$x^2+ax+b$. 其中 604 | $a,b$是待定系数. 605 | \end{analyze} 606 | 607 | \begin{solution} 608 | 由已知可设 609 | $x^4-9x^2+12x-4=(x-1)(x-2)(x^2+ax+b)$ 610 | 611 | 在以上恒等式中,分别取$x=0,-1$, 得: 612 | \[\begin{cases} 613 | -4=26\\ 614 | -24=6(1-a+b) 615 | \end{cases}\] 616 | 617 | 解这个方程组,得 $b=-2,\quad a=3$. 618 | 619 | 再解方程 $x^2+3x-2=0$, 得:$x=\frac{-3\pm\sqrt{17}}{2}$ 620 | 621 | 因此,原方程的另两个根是$\frac{-3+\sqrt{17}}{2}$,$\frac{-3-\sqrt{17}}{2}$. 622 | \end{solution} 623 | 624 | 如果给出一元三次或四次方程的根有某种给定的 625 | 关系,那么,利用方程的根与系数间的关系,余式定 626 | 理的推论等,就可以解所给的方程,这样的方程也可 627 | 以用待定系数法来解. 628 | 629 | \begin{example} 630 | 已知方程:$2x^3-3x^2-8x+12=0$有两个 631 | 互为相反数的根.试求这个方程的所有的根. 632 | \end{example} 633 | 634 | \begin{solution} 635 | 由已知,可设所给方程的根为:$a,-a,b$.根据余式定理推论,则有 636 | \[\begin{split} 637 | 2x^3-3x^2-8x+12&=2(x-a)(x+a)(x-b)\\ 638 | &=2x^3-2bx^2-2a^2x+2a^2b 639 | \end{split}\] 640 | 比较等式两边同类项的系数,得 641 | \begin{numcases}{} 642 | -3=-2b\\ 643 | -8=-2a^2\\ 644 | 12=2a^2b 645 | \end{numcases} 646 | 由(7.7), (7.8)解出:$a=\pm 2,\quad b=\frac{3}{2}$. 647 | 代入(7.9)都 648 | 能够适合.所以,原方程的根是$2, -2, \frac{3}{2}$. 649 | \end{solution} 650 | 651 | \begin{ex} 652 | 用待定系数法解下列各题: 653 | \begin{enumerate} 654 | \item 已知$f(1)=-4$, $f(0)=-5$, $f(-1)=-14$, $f(2)=1$. 655 | 试求三次多项式$f(x)$的表达式及$f(10)$. 656 | \item 已知$g(1)=g(2)=g(3)=0$, $g(0)=6$, $g(-1)=12$. 657 | 试求四次多预式$g(x)$表达式及它的另一个根. 658 | \item 已知方程$x^3-x^2-8x+12=0$有两个根相等,试解这个方 659 | 程. 660 | \item 已知方程$x^4-6x^3+8x^2+8x-16=0$有两个根都是2, 661 | 试求这个方程的另两个根. 662 | \end{enumerate} 663 | \end{ex} 664 | 665 | \subsubsection{把一个分式化为部分分式} 666 | 在分式的运算和变形中,有时需要把一个真分式 667 | 化为另外几个真分式的代数和的形式,例如: 668 | \[\frac{5x-4}{(x-1)(2x-1)}=\frac{1}{x-1}+\frac{3}{2x-1}\] 669 | 其中,两个比较简单的真分式 670 | $\frac{1}{x-1}$,$\frac{3}{2x-1}$ 671 | 就叫做 672 | 原真分式$\frac{5x-4}{(x-1)(2x-1)}$ 673 | 的\textbf{部分分式}. 674 | 675 | 把一个分化成部分分式的代数和,对今后学习 676 | 高等数学很有用途.以下将举例说明如何用待定系数 677 | 法,化分式为部分分式的和. 678 | 679 | 因为一个假分式都可以化成一个整式与一个真分 680 | 式的和,所以只要研究真分式的情形就可以了. 681 | 682 | 首先我们分析一下,$\frac{5x-4}{(x-1)(2x-1)}$ 683 | 是怎样化成$\frac{1}{x-1}+\frac{3}{2x-1}$ 684 | 的? 685 | 686 | 687 | 因为原分式的分母是互质的两个多项式$x-1$与 688 | $2x-1$的乘积,因此它就是$x-1$和$2x-1$的最低公倍 689 | 式.所以原分式一定是这样两个真分式 690 | $\frac{a}{x-1}$与$\frac{b}{2x-1}$ 691 | 的代数和,这里$a,b$是待定的常数. 692 | 693 | 如果恒等式 694 | \begin{equation} 695 | \frac{5x-4}{(x-1)(2x-1)}=\frac{a}{x-1}+\frac{b}{2x-1} 696 | \end{equation} 697 | 成立,那么就一定可以求出$a,b$的值. 698 | 699 | 由(7.10) 700 | \[ \frac{5x-4}{(x-1)(2x-1)}=\frac{a(2x-1)+b(x-1)}{(x-1)(2x-1)}\] 701 | 702 | 由于两个相等的分式的分母相等,因此它们的分 703 | 子也一定相等. 704 | 705 | $\therefore\quad 5x-4=a(2x-1)+b(x-1)$, 706 | 即: 707 | \[5x-4=(2a+b)x-(a+b)\] 708 | 比较等式两边同类项的系数,得到: 709 | \[\begin{cases} 710 | 2a+b=5\\ a+b=4 711 | \end{cases}\] 712 | 解得:$a=1$, $b=3$.代入等式(7.10)得到 713 | \[\frac{5x-4}{(x-1)(2x-1)}=\frac{1}{x-1}+\frac{3}{2x-1}\] 714 | 715 | 一般地说,如果$P$、$Q$是互质的两个因式,那么 716 | 真分式$\frac{A}{PQ}$ 717 | 可以化为形如 718 | $\frac{B}{P}$与$\frac{C}{Q}$ 719 | 两个真分式的和,即 720 | $\frac{B}{P}$与$\frac{C}{Q}$为$\frac{A}{PQ}$ 721 | 的部分分式. 722 | 723 | \begin{example} 724 | 化分式$\frac{23x-11x^2}{(2x-1)(9-x^2)}$为部分分式. 725 | \end{example} 726 | 727 | \begin{analyze} 728 | 因为原分式分母中的因式$2x-1$与$(3+x)(3-x)$是互质的因式.所以,原分式可以化成下面 729 | 两个真分式的和,即 730 | \[\frac{a}{2x-1}+\frac{ex+f}{(3+x)(3-x)}\] 731 | 这里$a,e,f$都是待定系数. 732 | 733 | 这两个分式都是真分式,也就是分子的次数小于 734 | 分母的次数.第一个分式的分母是一次,所以分子可以用常数$a$表示,第二个分式的分母是二次,所以分 735 | 子应用一次式$ex+f$表示. 736 | 737 | 但由前面分析知道等式 738 | $\frac{ex+f}{(3+x)(3-x)}=\frac{b}{3+x}+\frac{c}{3-x}$可以成立. 739 | 740 | 因此$\frac{23x-11x^2}{(2x-1)(3+x)(3-x)}$可以化成 $\frac{a}{2x-1}+\frac{b}{3+x}+\frac{c}{3-x}$的形式. 741 | \end{analyze} 742 | 743 | \begin{solution} 744 | 设: 745 | \begin{equation} 746 | \frac{23x-11x^2}{(2x-1)(3+x)(3-x)}=\frac{a}{2x-1}+\frac{b}{3+x}+\frac{c}{3-x} 747 | \end{equation} 748 | $\therefore\quad $有恒等式 749 | \begin{equation} 750 | 23x-11x^2=a(3+x)(3-x)+b(2x-1)(3-x)+c(2x-1)(3+x) 751 | \end{equation} 752 | 753 | \begin{itemize} 754 | \item 令$x=\frac{1}{2}$,代入恒等式(7.12)得:$\frac{35}{4}=a\cdot \frac{35}{4}$ 755 | 756 | $\therefore\quad a=1$ 757 | \item 令$x=-3$,代入恒等式(7.12)得:$-168=(-42)b$ 758 | 759 | $\therefore\quad b=4$ 760 | \item 令$x=3$,代入恒等式(7.12)得:$-30=45c$ 761 | 762 | $\therefore\quad c=-\frac{2}{3}$ 763 | \end{itemize} 764 | $\therefore\quad \frac{23x-11x^2}{(2x-1)(3+x)(3-x)}=\frac{1}{2x-1}+\frac{4}{3+x}-\frac{2}{3(3-x)}$ 765 | \end{solution} 766 | 767 | \begin{example} 768 | 化分式$\frac{4x^2+3x-1}{(x-1)(x^2+x-2)}$为部分分式. 769 | \end{example} 770 | 771 | \begin{analyze} 772 | 因为原分式的分母中的因式 773 | $x-1$与 774 | $x^2+x-2$不是互质的,所以先把原分式变形为: 775 | $\frac{4x^2+3x-1}{(x-1)^2(x+2)}$ 776 | 这里$(x-1)^2$与$x+2$是互质的. 777 | 778 | 原分式可以化成这样两个真分式的和:$\frac{a}{x+2}+\frac{bx+m}{(x-1)^2}$ 779 | 780 | 又由于 781 | \[\begin{split} 782 | \frac{bx+m}{(x-1)^2}&=\frac{b(x-1)+b+m}{(x-1)^2}\\ 783 | &=\frac{b(x-1)}{(x-1)^2}+\frac{b+m}{(x-1)^2}\\ 784 | &=\frac{b}{x-1}+\frac{b+m}{(x-1)^2} 785 | \end{split}\] 786 | 787 | 因为$b,m$都是待定常数,所以$b+m$也是待定常 788 | 数,不妨用$c$表示$b+m$. 789 | 790 | 这样原分式就可以化成:$\frac{a}{x+2}+\frac{b}{x-1}+\frac{c}{(x-1)^2}$的形式. 791 | \end{analyze} 792 | 793 | \begin{solution} 794 | $\frac{4x^2+3x-1}{(x-1)(x^2+x-2)}=\frac{4x^2+3x-1}{(x-1)^2(x+2)}$ 795 | 796 | 设$\frac{4x^2+3x-1}{(x-1)^2(x+2)}=\frac{a}{x+2}+\frac{b}{x-1}+\frac{c}{(x-1)^2}$ 797 | 798 | 所以,$4x^2+3x-1=a(x-1)^2+b(x-1)(x+2)+c(x+2)$. 799 | 800 | 因为这是恒等式,$x$可以任意取值,所以,我们 801 | 不妨: 802 | \begin{itemize} 803 | \item 令$x=1$, 得$6=3c$, $\therefore\quad c=2$ 804 | \item 再令$x=-2$, 得$9=9a$, $\therefore\quad a=1$ 805 | \item 再令$x=0$, 得$-1=a-2b+2c$, $\therefore\quad b=3$ 806 | \end{itemize} 807 | 因此: 808 | \[\begin{split} 809 | \frac{4x^2+3x-1}{(x-1)(x^2+x-2)}&=\frac{4x^2+3x-1}{(x-1)^2(x+2)}\\ 810 | &= \frac{1}{x+2}+\frac{3}{x-1}+\frac{2}{(x-1)^2} 811 | \end{split}\] 812 | \end{solution} 813 | 814 | \begin{example} 815 | 化$\frac{2x^2-x+1}{(x-1)^3}$为部分分式. 816 | \end{example} 817 | 818 | \begin{solution} 819 | 设 $2x^2-x+1=a(x-1)^2+b(x-1)+c=[a(x-1)+b](x-1)+c$ 820 | 821 | 累次作综合除法可以求得$a$、$b$、$c$: 822 | \begin{center} 823 | \begin{tikzpicture}[yscale=1.2, >=latex] 824 | \foreach \x /\xtext in {0/2,1/-1,2/+1} 825 | { 826 | \node at (\x,2)[left] {$\xtext$}; 827 | } 828 | 829 | \foreach \x /\xtext in {1/+2,2/+1} 830 | { 831 | \node at (\x,1.5)[left] {$\xtext$}; 832 | } 833 | 834 | \foreach \x /\xtext in {0/2,1/+1,2/+2} 835 | { 836 | \node at (\x,1)[left] {$\xtext$}; 837 | } 838 | \foreach \x /\xtext in {0/2,1/+3} 839 | { 840 | \node at (\x,0)[left] {$\xtext$}; 841 | } 842 | 843 | \node at (1,.5)[left]{$+2$}; \node at (2.5,2){1}; 844 | \draw (-1,.25)--(1.25,.25)--(1.25,1.25); 845 | \draw (.2,.25)--(.2,-.25)--(1.5,-.25); 846 | \draw (-1,1.25)--(2.25,1.25)--(2.25,2.5); 847 | \draw (1.25,-.25+1)--(2,-.25+1); 848 | \draw [->] (3,1)node[right]{$c$}--(2,1); 849 | \draw [->] (2,0)node[right]{$b$}--(1,0); 850 | \draw [->] (-1.5,0)node[left]{$a$}--(-.5,0); 851 | \end{tikzpicture} 852 | \end{center} 853 | 854 | $\therefore\quad 2x^2-x+1=2(x-1)^2+3(x-1)+2$ 855 | 856 | 因此: 857 | \[\begin{split} 858 | \frac{2x^2-x+1}{(x-1)^3}&= \frac{2(x-1)^2+3(x-1)+2}{(x-1)^3}\\ 859 | &=\frac{2}{x-1}+\frac{3}{(x-1)^2}+\frac{2}{(x-1)^3} 860 | \end{split}\] 861 | 862 | 注意: 863 | \begin{itemize} 864 | \item $\frac{2x^2-x+1}{(x-1)^3}$可以化成$\frac{a}{x-1}+\frac{b}{(x-1)^2}+\frac{c}{(x-1)^3}$的形式. 865 | \item 也可以用待定系数法解,但并不简便. 866 | \end{itemize} 867 | \end{solution} 868 | 869 | \begin{example} 870 | 化$\frac{42-19x}{(x-4)(x^2+1)}$为部分分式. 871 | \end{example} 872 | 873 | \begin{note} 874 | $x^2+1$在实数范围内已不能分解因式,它 875 | 与$x-4$是互质因式. 876 | \end{note} 877 | 878 | \begin{solution} 879 | 设$\frac{42-19x}{(x-4)(x^2+1)}=\frac{a}{x-4}+\frac{bx+c}{x^2+1}$ 880 | 881 | $\therefore\quad 42-19x=a(x^2+1)+(bx+c)(x-4)$是一个恒等式. 882 | 883 | 令$x=4$,得$a=-2$. 884 | 885 | 再把$a=-2$代入上式,整理得到: 886 | \[42-19x=(b-2)x^2+(c-4b)x-(4c+2)\] 887 | 比较等式两边同类项的系数,得 888 | \begin{numcases}{} 889 | b-2=0\\ 890 | c-4b=-19\\ 891 | -(4c+2)=42 892 | \end{numcases} 893 | 由(7.13), (7.14)解出$b=2,\quad c=-11$.代入(7.15) 894 | 都能够适合.因此: 895 | \[\frac{42-19x}{(x-4)(x^2+1)}=-\frac{2}{x-4}+\frac{2x-11}{x^2+1}\] 896 | \end{solution} 897 | 898 | 通过以上各例,可以归纳出以下结论: 899 | \begin{blk}{} 900 | 任何含有一个未知数的真分式,它的分母分解成 901 | 不可约因式后,都可以化成部分分式的代数和. 902 | \end{blk} 903 | 904 | 把一个真分式化为部分分式的问题,我们在例题 905 | 中已经学过了三种类型: 906 | 907 | \begin{enumerate} 908 | \item 分母中如果含有因式$ax+b$的一次幂,那 909 | 么,原真分式就对应有一个部分分式$\frac{A}{ax+b}$ 910 | ($A$是常数,$a\ne 0$); 911 | \item 分母中如果含有因式$(ax+b)^n\quad (n>1)$, 912 | 那么,原真分式就对应有$n$个部分分式的代数和,即 913 | \[\frac{A_1}{ax+b}+\frac{A_2}{(ax+b)^2}+\cdots+\frac{A_n}{(ax+b)^n}\] 914 | 其中:$A_1,A_2,\ldots,A_n$都是常数); 915 | \item 分母中如果含有因式 $ax^2+bx+c$的一次 916 | 幂,且$b^2-4ac<0$, 那么,原真分式就对应有一个 917 | 部分分式$\frac{mx+n}{ax^2+bx+c}$ 918 | ($m,n$都是常数,$a\ne 0$). 919 | \end{enumerate} 920 | 921 | 922 | 另外,还有一种类型,就是“分母中如果含有因 923 | 式$(ax^2+bx+c)^n\quad (n>1)$, 且$b^2-4ac<0$, 那么, 924 | 原真分式就对应有$n$个部分分式的代数和,即 925 | \[\frac{m_1x+n_1}{ax^2+bx+c}+\frac{m_2x+n_2}{(ax^2+bx+c)^2}+\cdots+\frac{m_nx+n_n}{(ax^2+bx+c)^n}\] 926 | 其中,$m_1,n_1,m_2,n_2,\ldots,m_n,n_n$ 927 | 都是常数($a\ne 0$)”. 928 | 但运算较繁,这里就略去了. 929 | 930 | \begin{ex} 931 | \begin{enumerate} 932 | \item 已知下列各恒等式,求$A$、$B$、$C$的值. 933 | \begin{enumerate} 934 | \item $\frac{4x+1}{x(x+1)}=\frac{A}{x}+\frac{B}{x+1}$ 935 | \item $\frac{46+13x}{12x^2-11x-15}=\frac{A}{3x-5}+\frac{B}{4x+3}$ 936 | \item $\frac{4-7x}{(x^2+1)(x-2)}=\frac{Ax+B}{x^2+1}+\frac{C}{x-2}$ 937 | \end{enumerate} 938 | \item \begin{enumerate} 939 | \item 已知恒等式$x^3-6x^2-4x+8=A(x-1)^3+B(x-1)^2+ 940 | +C(x-1)+D$.求$A,B,C,D$; 941 | \item 用$x-2$的各次幂表示$3x^3-8x^2+10$. 942 | \end{enumerate} 943 | 944 | \item 化下列各式为部分分式: 945 | \begin{multicols}{2} 946 | \begin{enumerate} 947 | \item $\frac{2x+11}{(x-2)(x+3)}$ 948 | \item $\frac{6x-1}{(2x+1)(3x-1)}$ 949 | \item $\frac{x}{x^2-2x-3}$ 950 | \item $\frac{x^2-3x-1}{(x-2)^2}$ 951 | \item $\frac{x^2+x-1}{(x^2+1)(x-2)}$ 952 | \item $\frac{x^2+6x-1}{(x-3)^2(x-1)}$ 953 | \end{enumerate} 954 | \end{multicols} 955 | \end{enumerate} 956 | 957 | \end{ex} 958 | 959 | \subsubsection{求算术平方根$\sqrt{c+2\sqrt{b}}$} 960 | 在有些计算中,需要将算术平方根$\sqrt{c+2\sqrt{b}}$ 961 | 表示为两个二次根式的代数和.利用待定系数法,也可 962 | 以解决这类问题. 963 | 964 | \begin{example} 965 | 计算$\sqrt{8+2\sqrt{12}}$ 966 | \end{example} 967 | 968 | \begin{solution} 969 | 设$\sqrt{8+2\sqrt{12}}=\sqrt{x}+\sqrt{y}$($x,y$都是正整数). 970 | 971 | 则两边平方后可得: 972 | \[8+2\sqrt{12}=\left(\sqrt{x}+\sqrt{y}\right)^2=(x+y)+2\sqrt{xy}\] 973 | 以上恒等式要成立,必须满足: 974 | \begin{numcases}{} 975 | x+y=8\\ xy=12 976 | \end{numcases} 977 | 将(6.16)代入(6.17)得:$x^2-8x+12=0$ 978 | 979 | 解二次方程得:$x_1=2,\quad x_2=6$; 980 | 981 | 再由(6.16), 得出$y_1=6,\quad y_2=2$ 982 | 983 | $\therefore\quad \sqrt{8+2\sqrt{12}}=\sqrt{2}+\sqrt{6}(=\sqrt{6}+\sqrt{2})$ 984 | 985 | 用同样的方法,可以求得:$\sqrt{8-2\sqrt{12}}=\sqrt{6}-\sqrt{2}$ 986 | (注意:$\sqrt{8-2\sqrt{12}}=\sqrt{2}-\sqrt{6}$是错误的,因 987 | 为$\sqrt{2}$小于$\sqrt{6}$, 其差小于零,但所求算术根 988 | $\sqrt{8-2\sqrt{12}}$是不能小于零的.) 989 | \end{solution} 990 | 991 | 一般来说,要求形如$a\pm 2\sqrt{b}$的数的算术平方根 992 | ($a,b$都是正整数且$b$不是完全平方数),就可以引 993 | 进未定数$x$、$y$, 使它们满足$\sqrt{a\pm 2\sqrt{b}}=\sqrt{x}\pm\sqrt{y}$ 994 | ($x$、$y$都是正整数,且$x>y$). 995 | 996 | 两边平方,得 $a\pm 2\sqrt{b}=(x+y)\pm 2\sqrt{xy}$,比较等式两边相应的部分,得 997 | \begin{numcases}{} 998 | a=x+y\\ b=xy 999 | \end{numcases} 1000 | 解这个含有未定数$x,y$的方程组,得 1001 | \[\begin{cases} 1002 | x=\frac{a+\sqrt{a^2-4b}}{2}\\ y=\frac{a-\sqrt{a^2-4b}}{2} 1003 | \end{cases}\] 1004 | 1005 | 由这里还可以看出,形如$a\pm 2\sqrt{b}$的数,只有当 1006 | $a^2-4b>0$, 且是一个完全平方数时,才能有形如 1007 | $\sqrt{x}\pm \sqrt{y}$ 1008 | 的算术平方根. 1009 | 1010 | 1011 | 1012 | \begin{example} 1013 | 求下列各数的算术平方根(精确到0.01): 1014 | \begin{multicols}{3} 1015 | \begin{enumerate} 1016 | \item $3+2\sqrt{2}$ \item $9+4\sqrt{5}$ \item $6-\sqrt{20}$ 1017 | \end{enumerate} 1018 | \end{multicols} 1019 | \end{example} 1020 | 1021 | \begin{solution} 1022 | \begin{enumerate} 1023 | \item $\text{原式}=\sqrt{(\sqrt{2}+1)^2}=\sqrt{2+1}\approx 2.41$ 1024 | 1025 | 因为,$x+y=3$且$xy=2$,可观察得出$x=2,\quad y=1$. 1026 | \item $\because\quad 9+4\sqrt{5}=9+2\sqrt{20}$ 1027 | 1028 | $\therefore\quad $设 $\sqrt{9+2\sqrt{20}}=\sqrt{x}+\sqrt{y}$,两边平方,得: 1029 | \[9+2\sqrt{20}=(x+y)+2\sqrt{xy}\] 1030 | 比较两边相应部分,得 1031 | \[\begin{cases} 1032 | x+y=9\\ 1033 | xy=20 1034 | \end{cases}\] 1035 | 解这个方程组,得:$x=5,\quad y=4$ 1036 | 1037 | $\therefore\quad \sqrt{9+4\sqrt{5}}=\sqrt{5}+\sqrt{4}\approx 2.24+2=4.24$ 1038 | 1039 | \item 设$\sqrt{6-\sqrt{20}}=\sqrt{6-2\sqrt{5}}=\sqrt{x}-\sqrt{y}$ 1040 | 1041 | 两边平方,得:$6-2\sqrt{5}=(x+y)-2\sqrt{xy}$ 1042 | 1043 | $\therefore\quad x+y=6,\quad xy=5$ 1044 | 解得$x=5,\quad y=1$ 1045 | 1046 | $\therefore\quad \sqrt{6-2\sqrt{5}}=\sqrt{5}-1\approx 1.24$ 1047 | 1048 | \end{enumerate} 1049 | \end{solution} 1050 | 1051 | \begin{ex} 1052 | 求下列各式的值(精确到0.01): 1053 | \begin{multicols}{2} 1054 | \begin{enumerate} 1055 | \item $\sqrt{5+2\sqrt{6}}$ 1056 | \item $\sqrt{10-2\sqrt{21}}$ 1057 | \item $\sqrt{10+4\sqrt{6}}$ 1058 | \item $\sqrt{11-\sqrt{120}}$ 1059 | \end{enumerate} 1060 | \end{multicols} 1061 | \end{ex} 1062 | 1063 | 1064 | 1065 | 1066 | 1067 | 1068 | \subsubsection{其它数列求和} 1069 | 除常见的等差,等比数列外,我国古代的“垛积 1070 | 术”中,还有一些较复杂的数列求和问题,这些问题 1071 | 也有重要应用. 1072 | 1073 | 1074 | \begin{example} 1075 | 某仓库中,存放的罐头堆成锥形垛.顶上 1076 | 放一桶,第二层有四桶,以下各层,每个桶均由四个 1077 | 桶顶着(如图)一垛共有五层.问这一垛共有多少桶? 1078 | \end{example} 1079 | 1080 | \begin{figure}[htp] 1081 | \centering 1082 | \includegraphics[scale=.7]{1.png} 1083 | \end{figure} 1084 | 1085 | \begin{solution} 1086 | 显然,这一垛共有:$(1+4+9+16+25)$桶. 1087 | 即:$1^2+2^2+3^2+4^2+5^2=55$桶. 1088 | 1089 | 这就是说,前五个自然数的平方 1090 | 和等于55. 1091 | 1092 | 一般地,如果由前n个自然数的 1093 | 平方组成的数列$1^2,2^2,3^2,\ldots,n^2$. 1094 | 如何求出它们的和呢?能不能导出一 1095 | 个通用的公式呢? 1096 | 1097 | 以下我们将先对这个数列的前$n$项和的特点进行 1098 | 分析,然后应用待定系数法导出它的求和公式. 1099 | 1100 | 设$1^2+2^2+\cdots+n^2=S(n)$,在$S(n)$中,$n$代表项数.显然有: 1101 | \[\begin{split} 1102 | S(0)&=0\\ S(1)&=1\\ S(2)&=1^2+2^2=5\\ S(3)&=1^2+2^2+3^2=14\\ 1103 | \cdots \cdots&\cdots\cdots\\ 1104 | S(n)&=1^2+2^2+\cdots+n^2\\ 1105 | S(n+1)&=1^2+2^2+\cdots+n^2+(n+1)^2 1106 | \end{split}\] 1107 | 1108 | 而且还可以看出:无论项数$n$取几,总有 1109 | \[S(n+1)-S(n)=(n+1)^2=n^2+2n+1\] 1110 | 可见,$S(n)$很可能是一个关于项数$n$的多项式, 1111 | 它只要满足两个性质: 1112 | \begin{enumerate} 1113 | \item $S(0)=0$ 1114 | \item $S(n+1)-S(n)=n^2+2n+1$ ($n$的二次式) 1115 | \end{enumerate} 1116 | \end{solution} 1117 | 1118 | 这就启示我们,如果能求出一个多项式$S(x)$, 能 1119 | 满足以上两条性质,即$S(0)=0$, $S(x+1)-S(x)$是 1120 | 一个二次式.那么,所求数列的前$n$项和,就是当$x 1121 | =n$时,多项式$S(x)$的值$S(n)$. 1122 | 1123 | 但是,满足以上两条性质的多项式$S(x)$应该是 1124 | 几次多项式呢? 1125 | 1126 | 我们不妨设$S(x)$是$m$次多项式,即 1127 | \[S(x)=ax^m+bx^{m-1}+\cdots+cx+d\quad (a\ne 0)\] 1128 | 则:$S(x+1)=a(x+1)x^m+b(x+1)x^{m-1}+\cdots+c(x+1)x+d$ 1129 | 由乘法公式,不难知道: 1130 | \[\begin{split} 1131 | S(x+1)&= a(x^m+mx^{m-1}+\cdots +1)+b[x^{m-1}+(m-1)x^{m-2}+\cdots+1]\\ 1132 | +\cdots + c(x+1)+d\\ 1133 | &=ax^m+(b+am)x^{m-1}+\cdots +(c+\cdots)x+(a+b+\cdots +c+d)\\ 1134 | \end{split}\] 1135 | $\therefore\quad S(x+1)-S(x)=(b+am-b)x^{m-1}+\cdots =amx^{m-1}+\cdots$ 1136 | 1137 | 由于$am\ne 0$, 显然$S(x+1)-S(x)$是$m-1$次多 1138 | 项式,因此,我们可以得出: 1139 | 1140 | \begin{blk}{} 1141 | 当$S(x)$是一个$m$次多项式时,$S(x+1)-S(x)$必定是一个$m-1$次多项式. 1142 | \end{blk} 1143 | 1144 | 也就是说,多项式$S(x+1)-S(x)$的次数比多项 1145 | 式$S(x)$的次数低一次. 1146 | 1147 | 这样,由于在我们以上所求问题中,$S(x+1)-S(x)$是二次多项式,所以, 1148 | \textbf{$S(x)$必定是一个三次多项式}. 1149 | 1150 | 基于以上分析,以下我们就可以用待定系数法导 1151 | 出前$n$个自然数平方的求和公式. 1152 | 1153 | 1154 | \begin{example} 1155 | 试求$1^2+2^2+\cdots+n^2$. 1156 | \end{example} 1157 | 1158 | 1159 | \begin{solution} 1160 | 设$1^2+2^2+\cdots+n^2=S(n)$ 1161 | 1162 | 由分析知$S(n)$是一个关于项数$n$的三次式, 1163 | 且 $S(0)=0$, $S(1)=1$, $S(2)=5$, $S(3)=14$. 1164 | 其中,0是$S(n)$的一个根. 1165 | 1166 | $\therefore\quad $由余式定理的推论,得: 1167 | \[S(n)=n(an^2+bn+c)\] 1168 | 这里$a$、$b$、$c$都是待 1169 | 定系数. 1170 | 1171 | 将$S(1)=1$, $S(2)=5$, $S(3)=14$分别代入上 1172 | 式,即可得出 1173 | \[\begin{cases} 1174 | 1=a+6+c\\ 1175 | 5=2(4a+26+c)\\ 1176 | 14=3(9a+36+c) 1177 | \end{cases}\Rightarrow\quad \begin{cases} 1178 | a+b+c=1\\ 1179 | 4a+2b+c=\frac{5}{2}\\ 1180 | 9a+3b+c=\frac{14}{3} 1181 | \end{cases}\] 1182 | 解这个方程组,得$a=\frac{1}{3},\quad b=\frac{1}{2},\quad c=\frac{1}{6}$ 1183 | 因此: 1184 | \[\begin{split} 1185 | S(n)&=n\left(\frac{1}{3}n^2+\frac{1}{2}n+\frac{1}{6}\right)\\ 1186 | &=\frac{n}{6}(2n^2+3n+1)\\ 1187 | &=\frac{1}{6}n(n+1)(2n+1) 1188 | \end{split}\] 1189 | \end{solution} 1190 | 1191 | 1192 | \begin{ex} 1193 | 试用待定系数法求$1^3+2^3+\cdots+n^3$. 1194 | \end{ex} 1195 | 1196 | 1197 | 1198 | \section*{习题7.2} 1199 | \addcontentsline{toc}{subsection}{习题7.2} 1200 | \begin{enumerate} 1201 | \item 已知$x^4+4x^3+6px^2+4qx+r$能被$x^3+3x^2+9x+3$整 1202 | 除,试求$p$、$q$、$r$的值. 1203 | \item 已知$f(x)=ax^2+bx+c$是一个完全平方式,试用待定 1204 | 系数法证明:$b^2-4ac=0$. 1205 | \item 试用待定系数法把$f(x)=3x^3-10x^2+13$表示成$(x-2)$ 1206 | 的各次方幂和. 1207 | \item 用待定系数法,把$x^4-2x^2-2$表示成$x^2-x+1$的各 1208 | 次方幂和. 1209 | \item 已知$x^3-x^2-8x+12=0$有两个根相等,试解这个方 1210 | 程. 1211 | \item 已知方程$x^3-4x^2+x+k=0$有一个根是$-1$, 试求 1212 | 它的另外两个根. 1213 | \item 如果方程$x^3+px^2+qx+r=0$有两个根互为相反数, 1214 | 试求$p,q,r$应具有什么关系? 1215 | \item 化下列各分式为部分分式 1216 | \begin{multicols}{2} 1217 | \begin{enumerate} 1218 | \item $\frac{3 x-4}{x^{2}-3 x+2}$ 1219 | \item $\frac{x^{2}+x-3}{(x-1)(x-2)(x-3)}$ 1220 | \item $\frac{x^{2}-2}{x^{3}-3 x^{2}+2 x}$ 1221 | \item $\frac{x^{3}-6 x^{2}+4 x+8}{(x-3)^{4}}$ 1222 | \item $\frac{2 x^{2}+1}{x^{3}-1}$ 1223 | \item $\frac{21 x-14}{(x-3)^{2}(2 x+1)}$ 1224 | \item $\frac{8 x}{(x+1)\left(x^{2}-1\right)}$ 1225 | \item $\frac{x^{2}+x+1}{\left(x^{2}+1\right)\left(x^{2}+2\right)}$ 1226 | \end{enumerate} 1227 | \end{multicols} 1228 | 1229 | \item 求下列各算术平方根: 1230 | \begin{multicols}{2} 1231 | \begin{enumerate} 1232 | \item $\sqrt{5+2 \sqrt{6}}$ 1233 | \item $\sqrt{15-2 \sqrt{56}}$ 1234 | \item $\sqrt{11+4 \sqrt{7}}$ 1235 | \item $\sqrt{26-8 \sqrt{10}}$ 1236 | \end{enumerate} 1237 | \end{multicols} 1238 | 1239 | \item 求下列数列的前 $n$ 项和: 1240 | $1^{4}, 2^{4}, 3^{4}, \ldots , n^{4}, \ldots$ 1241 | 1242 | \end{enumerate} 1243 | 1244 | 1245 | 1246 | 1247 | 1248 | 1249 | \section*{本章内容要点} 1250 | 1251 | 本章的主要内容是两种常见数列的求和及待定系数法与它的应用. 1252 | 1253 | 一、等差数列 1254 | 1255 | \begin{enumerate} 1256 | \item 按顺序排好的一列数中,如果从第二个数 1257 | 起,每一个数与它前一个数的差都相等,那么,这一列数叫做等差数列. 1258 | 1259 | 设等差数列的首项为$a_1$, 公差为$d$, 项数为$n$, 末项为$a_n$及前$n$项和为$S_n$, 则有以下关系式: 1260 | \[\begin{split} 1261 | a_n&=a_1+(n-1)d\\ 1262 | S_n&=\frac{n}{2}[2a_1+(n-1)d]=\frac{n}{2}(a_1+a_n) 1263 | \end{split}\] 1264 | 1265 | 如果已知$a_1,a_n,n,d,S_n$中的任意三个,就可以利用这两个公式,求出另两个. 1266 | 1267 | \item 在两个已知数$a,b$之间,插入$n$个数构成等 1268 | 差数列的问题,实际上就是已知首项$a$, 末项$b$及项数$n+2$, 要求出公差,进而可以求出插入的各项,还可以求出所有项的和. 1269 | \end{enumerate} 1270 | 1271 | \vskip 2ex 1272 | 二、等比数列 1273 | 1274 | 按顺序排好的一列数中,如果从第二个数起,每一个数与它前一个数的比都相等,那么,这一列数叫 1275 | 做等比数列. 1276 | 1277 | 设等比数列的首项为$a_1$, 公比为$q$, 项数为$n$, 末项为$a_n$, 前$n$项的和为$S_n$, 则有以下关系式: 1278 | \[\begin{split} 1279 | a_n&=a_1q^{n-1}\\ 1280 | S_n&=\frac{a_1(1-q^n)}{1-q} 1281 | \end{split}\] 1282 | 1283 | 公比$q$的取值,决定了等比数列各项的大小变化趋向:如果首项$a_1>0$(或$<0$),那么, 1284 | \begin{itemize} 1285 | \item 当$q>1$时,等比数列逐项增大(或减小); 1286 | \item 当$0